The MCAT Physics Book
The MCAT Physics Book
The MCAT Physics Book
MCAT
P~Y.=
Book
""*Y
rraw of MCAT
hysdcs
The MCAT
Physics Book
Garrett Biehle
Nova Press
Vocabulary 4000: The 4000 Words Essential for an Educated Vocabulary (160 pages)
ISBN: 1-889057-33-9
MCAT is a service mark of the Association of American Medical Colleges.
The MCAT
Physics Book
Preface
The physics portion of the Medical College Aptitude Test (MCAT) contains questions to test your knowledge of basic physics and your ability to apply that knowledge to
unfamiliac situations.
The goal of this book is to review basic physics with an emphasis on the principles
and ideas, and to help you learn to approach new situations and think about them with a
physics mindset. This book is not primarily concerned with test-taking techniques. It is
designed to help you develop an intuitive understanding of physics, so that you will
understand the MCAT questions and how to answer them.
Each chapter contains a discussion of a major physics topic, followed by problems
to help you apply the concepts. Finally, there are MCAT-style passages and questions to
help you get used to the MCAT format. All the problems have complete solutions in the
back of the book, some with tips to help you approach problems and to solve them faster.
If you work through this book, taking notes with pencil and paper by your side, and
solving the problems, you will improve your understanding of physics. And you will
improve your score on the MCAT.
The author wishes to thank James Aldridge for his comments on the manuscript.
The author wishes to thank Michelle Haller for the many hours she spent editing the
book's prose.
The author especially appreciates Andrew and Judy Cordell for their critical reading
of the book, alerting the author to subtleties in the science, helping the author approach
difficult topics, and pointing out unnecessary detail.
contents
Chapter 1
Introduction . . . . . . . . . . . . . . . . . . . . . . . . . . . . . . . . . 1
Chapter 2
Chapter 3
Laws of Motion . . . . . . . . . . . . . . . . . . . . . . . . . . . . . . . . . . 3 1
Chapter 4
......................................
Grav~tat~on
Chapter 5
Chapter 6
Chapter 7
Chapter 8
Momentum . . . . . . . . . . . . . . . . . . . . . . . . . . . . . . . . . . . .
111
Chapter 9
Energy . . . . . . . . . . . . . . . . . . . . . . . . . . . . . . . . . . . . . . . .
121
Interlude
Solving Problems . . . . . . . . . . . . . . . . . . . . . . . . . . . . . . .
149
Chapter 10
Fluids . . . . . . . . . . . . . . . . . . . . . . . . . . . . . . . . . . . . . . . . .
159
Chapter 11
185
Chapter 12
Sound . . . . . . . . . . . . . . . . . . . . . . . . . . . . . . . . . . . . . . . . . 207
Chapter 1 3
Light . . . . . . . . . . . . . . . . . . . . . . . . . . . . . . . . . . . . . . . . . . 225
Chapter 14
Electrodynamics . . . . . . . . . . . . . . . . . . . . . . . . . . . . . . . . 249
Chapter 15
Electric Circuits . . . . . . . . . . . . . . . . . . . . . . . . . . . . . . . . .
Chapter
45
279
Solutions
Index
. .
. . . . . . . . . . . . . . . . . . . . . . . . . . . . . . . . . . 323
. . . . . . . . . . . . . . . . . . . . . . . . . . . . . . . . Index-i
CluQter 1
Introduction . . . . . . . . . . . . . . . . . . . . . . . . . . . . . . . . . . 1
A . Rilosophy of the Book . . . . . . . . . . . . . . . . . . . . . . . . . . . . . . . . . . . . . . . . . . . . . . I
8 . Reading this Book . . . . . . . . . . . . . . . . . . . . . . . . . . . . . . . . . . . . . . . . . . . . . . . . . . 1
C.Unib . . . . . . . . . . . . . . . . . . . . . . . . . . . . . . . . . . . . . . . . . . . . . . . . . . . . . . . . . .
D.Equations . . . . . . . . . . . . . . . . . . . . . . . . . . . . . . . . . . . . . . . . . . . . . . . . . . . . . . .
Chapter 1 Problems . . . . . . . . . . . . . . . . . . . . . . . . . . . . . . . . . . . . . . . . . . . . .
Chapter
The Language of M o t i o n
..........................
13
A.lntroduction . . . . . . . . . . . . . . . . . . . . . . . . . . . . . . . . . . . . . . . . . . . . . . . . . . . .
13
B.Forcc . . . . . . . . . . . . . . . . . . . . . . . . . . . . . . . . . . . . . . . . . . . . . . . . . . . . . . . . .
13
C.Mass . . . . . . . . . . . . . . . . . . . . . . . . . . . . . . . . . . . . . . . . . . . . . . . . . . . . . . . . . 13
D.Vcctors . . . . . . . . . . . . . . . . . . . . . . . . . . . . . . . . . . . . . . . . . . . . . . . . . . . . . . . 14
I.
J.
18
Uniform Acceleration . . . . . . . . . . . . . . . . . . . . . . . . . . . . . . . . . . . . . . . . . . . . . . . 2 1
Kinematic Equations for Constant Acceleration . . . . . . . . . . . . . . . . . . . . . . . . . . . . . . 22
Chapter 2 Problems . . . . . . . . . . . . . . . . . . . . . . . . . . . . . . . . . . . . . . . . . . . . . .
Chapter 3
24
Laws of M o t i o n . . . . . . . . . . . . . . . . . . . . . . . . . . . . . . . . . . 31
D. Force Diagrams . . . . . . . . . . . . . . . . . . . . . . . . . . . . . . . . . . . . . . . . . . . . . . . . . .
36
Chapter 3 Problems . . . . . . . . . . . . . . . . . . . . . . . . . . . . . . . . . . . . . . . . . . . . . .
39
Chapter 4
Gravitation . . . . . . . . . . . . . . . . . . . . . . . . . . . . . . . . . . . . . . . - 4 5
A .The Law of Gravitation . . . . . . . . . . . . . . . . . . . . . . . . . . . . . . . . . . . . . . . . . . . . . 45
8.
. . . . . . . . . . . . . . . . . . . . . . . . . . . . . . . . . . . . . . . . . . . . . . . 47
C.FneFall . . . . . . . . . . . . . . . . . . . . . . . . . . . . . . . . . . . . . . . . . . . . . . . . . . . . . . . 47
D. Horizontal and Vertical Motion . . . . . . . . . . . . . . . . . . . . . . . . . . . . . . . . . . . . . . . . 4 9
Chapter 4 Problems . . . . . . . . . . . . . . . . . . . . . . . . . . . . . . . . . . . . . . . . . . . . .
Chapter
54
5 Problems . . . . . . . . . . . . . . . . . . . . . . . . . . . . . . . . . . . . . . . . . . . . . . 70
Chapter 6
A.lntroduction
....................................................
...................................................
D.SticWslip . . . . . . . . . . . . . . . . . . . . . . . . . . . . . . . . . . . . . . . . . . . . . . . . . . . . . .
E. Air resistance . . . . . . . . . . . . . . . . . . . . . . . . . . . . . . . . . . . . . . . . . . . . . . . . . . . .
B.Statichction
77
C.wcfrictian
80
Chapter
6 Problems . . . . . . . . . . . . . . . . . . . . . . . . . . . . . . . . . . . . . . . . . . . . . .
83
83
86
Chapter 7
A.lntroduction . . . . . . . . . . . . . . . . . . . . . . . . . . . . . . . . . . . . . . . . . . . . . . . . . . . .
93
93
8 . Lnguage of Rotation. . . . . . . . . . . . . . . . . . . . . . . . . . . . . . . . . . . . . . . . . . . . . . .
C.Torque . . . . . . . . . . . . . . . . . . . . . . . . . . . . . . . . . . . . . . . . . . . . . . . . . . . . . . . .9 4
D.Equilibrium . . . . . . . . . . . . . . . . . . . . . . . . . . . . . . . . . . . . . . . . . . . . . . . . . . . . . 9 7
E.
Solid Properties . . . . . . . . . . . . . . . . . . . . . . . . . . . . . . . . . . . . . . . . . . . . . . . . .
100
Chapter 7 Problems . . . . . . . . . . . . . . . . . . . . . . . . . . . . . . . . . . . . . . . . . . . . . 1 0 3
Chapter 8
Momentum . . . . . . . . . . . . . . . . . . . . . . . . . . . . . . . . . . . .
1 11
111
112
Chapter 8 Problems . . . . . . . . . . . . . . . . . . . . . . . . . . . . . . . . . . . . . . . . . . . . .
116
Chapter 9
121
Energy . . . . . . . . . . . . . . . . . . . . . . . . . . . . . . . . . . . . . . . .
114
A . Introduction . . . . . . . . . . . . . . . . . . . . . . . . . . . . . . . . . . . . . . . . . . . . . . . . . . . 1 2 1
B.Wo& . . . . . . . . . . . . . . . . . . . . . . . . . . . . . . . . . . . . . . . . . . . . . . . . . . . . . . . . 191
C.EnergyofMotion . . . . . . . . . . . . . . . . . . . . . . . . . . . . . . . . . . . . . . . . . . . . . . . . 1 2 4
D. Potential Energy and Conservative Forces . . . . . . . . . . . . . . . . . . . . . . . . . . . . . . . . . 1 2 6
E. Conservation of Energy . . . . . . . . . . . . . . . . . . . . . . . . . . . . . . . . . . . . . . . . . . . . 127
F. Efficiency of Energy Conversion . . . . . . . . . . . . . . . . . . . . . . . . . . . . . . . . . . . . . . . 1 2 9
G.Power . . . . . . . . . . . . . . . . . . . . . . . . . . . . . . . . . . . . . . . . . . . . . . . . . . . . . . . 1 3 0
H.Pulleys . . . . . . . . . . . . . . . . . . . . . . . . . . . . . . . . . . . . . . . . . . . . . . . . . . . . . . . 1 3 1
Chapter 9 Problems . . . . . . . . . . . . . . . . . . . . . . . . . . . . . . . . . . . . . . . . . . . . . 1 3 4 .
interlude
A.lntroduction
149
B. General Strategy . . . . . . . . . . . . . . . . . . . . . . . . . . . . . . . . . . . . . . . . . . . . . . . .
149
C . Specific Strategies . . . . . . . . . . . . . . . . . . . . . . . . . . . . . . . . . . . . . . . . . . . . . . .
153
Chapter 10
Fluids . . . . . . . . . . . . . . . . . . . . . . . . . . . . . . . . . . . . . . . . .
A . Introduction . . . . . . . . . . . . . . . . . . . . . . . . . . . . . . . . . . . . . . . . . . . . . . . . . . .
B. Some definitions . . . . . . . . . . . . . . . . . . . . . . . . . . . . . . . . . . . . . . . . . . . . . . . . .
159
159
159
C.Buoyantforce . . . . . . . . . . . . . . . . . . . . . . . . . . . . . . . . . . . . . . . . . . . . . . . . . . 160
D. Facta about presswe . . . . . . . . . . . . . . . . . . . . . . . . . . . . . . . . . . . . . . . . . . . . . . 1 6 2
F.
Chapter 10 Problems . . . . . . . . . . . . . . . . . . . . . . . . . . . . . . . . . . . . . . . . . . . .
170
Chapter 1 1
A.lntroduction
197
E.
. . . . . . . . . . . . . . . . . . . . . . . . . . . . . . . . . . . . . . . . . . . . . . . . . . . . . . . . 211
Chapter 1 3
Light . . . . . . . . . . . . . . . . . . . . . . . . . . . . . . . . . . . . . . . . . .
225
A.lntroduction . . . . . . . . . . . . . . . . . . . . . . . . . . . . . . . . . . . . . . . . . . . . . . . . . . . 215
B . General Properties of Light . . . . . . . . . . . . . . . . . . . . . . . . . . . . . . . . . . . . . . . . . 2 2 5
C . Reflection and Refraction . . . . . . . . . . . . . . . . . . . . . . . . . . . . . . . . . . . . . . . . . . . 2 2 6
. . . . . . . . . . . . . . . . . . . . . . . . . . . . . . . . . . . . . . . . . . . . . 235
238
238
239
F. Dispersion . . . . . . . . . . . . . . . . . . . . . . . . . . . . . . . . . . . . . . . . . . . . . . . . . . . .
G. Combination of Lenses . . . . . . . . . . . . . . . . . . . . . . . . . . . . . . . . . . . . . . . . . . . . .
H. Ideal Lenses and Nonideal Lenses . . . . . . . . . . . . . . . . . . . . . . . . . . . . . . . . . . . . .
Chapter 13 Problems . . . . . . . . . . . . . . . . . . . . . . . . . . . . . . . . . . . . . . . . . . . .
Chapter 1 4
240
Electrodynamics . . . . . . . . . . . . . . . . . . . . . . . . . . . . . . . . 2 4 9
Alntroduction . . . . . . . . . . . . . . . . . . . . . . . . . . . . . . . . . . . . . . . . . . . . . . . . . . . 2 4 9
8.ElectricCharse . . . . . . . . . . . . . . . . . . . . . . . . . . . . . . . . . . . . . . . . . . . . . . . . . . 2 4 9
C . Charges and Materials . . . . . . . . . . . . . . . . . . . . . . . . . . . . . . . . . . . . . . . . . . . . . ' 2 5 0
. . . . . . . . . . . . . . . . . . . . . . . . . . . . . . . . . . . . . . . . . . . . . . . . . 252 .
D.Coulomb'sbw
E. Electric Field . . . . . . . . . . . . . . . . . . . . . . . . . . . . . . . . . . . . . . . . . . . . . . . . . . .
F. Electric Potential. . . . . . . . . . . . . . . . . . . . . . . . . . . . . . . . . . . . . . . . . . . . . . . . .
G. Magnetic Fields . . . . . . . . . . . . . . . . . . . . . . . . . . . . . . . . . . . . . . . . . . . . . . . . .
H. Electromagnetic Radiation . . . . . . . . . . . . . . . . . . . . . . . . . . . . . . . . . . . . . . . . . . .
253
958
263
964
Chapter 1 5
Electric Circuits . . . . . . . . .
.
.
. . . . . . . . . . . . . . . . . . . . . . . . . . . . . . . . . . . . . . . . . . . . . . . . .2 7 9
A . Introduction
B . Ohm's Law and the Combination of Resistors . . . . . . . . . . . . . . . . . . . . . . . . . . . . . . . 281
C . Real DC cells and Real Wires . . . . . . . . . . . . . . . . . . . . . . . . . . . . . . . . . . . . . . . . 285
D.Power . . . . . . . . . . . . . . . . . . . . . . . . . . . . . . . . . . . . . . . . . . . . . . . . . . . . . . . 2 8 6
E. Capacitance . . . . . . . . . . . . . . . . . . . . . . . . . . . . . . . . . . . . . . . . . . . . . . . . . . . . 288
F. Alternating current . . . . . . . . . . . . . . . . . . . . . . . . . . . . . . . . . . . . . . . . . . . . . . . 2 9 0
Chapter 15 Problems . . . . . . . . . . . . . . . . . . . . . . . . . . . . . . . . . . . . . . . . . . . . 999
Chapter 16
A.lntroduction
305
. . . . . . . . . . . . . . . . . . . . . . . . . . . . . . . . . . . . . . . . . 307
. . . . . . . . . . . . . . . . . . . . . . . . . . . . . . . . . . . . . . . . . . . . . . . . . . . 309
C . Energy Levels a
d Transitions
Solutions
Index
. . . . . . . . . . . . . . . . . . . . . . . . . . . . . . . . . . 323
. . . . . . . . . . . . . . . . . . . . . . . . . . . . . . . . . Index-i
He
3
4.003
10
Li
Be
Ne
6.94
11
9.01
12
10.81
13
12.011
14
14.01
15
16.00
16
19.00
17
20.18
18
Na
Mg
Al
Si
CI
Ar
22.99
19
24.31
20
21
22
23
27
28
29
30
26.98
31
28.09
32
30.97
33
32.07
34
35.45
35
39.95
36
Ca
Sc
Ti
Cr
Mn
Fe
Co
Ni
Cu
Zn
Ga
Ge
As
Se
Br
Kr
39.10
37
40.08
38
44.96
39
47.90
40
50.94
41
52.00
42
54.94
43
55.85
44
58.93
45
58.69
46
63.55
47
65.39
48
69.72
49
72.61
50
74.92
51
78.96
52
79.90
53
83.80
54
25
'
26
'
Rb
Sr
Zr
Nb
Mo
Tc
Ru
Rh
Pd
85.47
55
87.62
56
88.91
. 57
91.22
72
92.91
73
95.94
74
[98.91]
75
101.07
76
102.91
77
106.42
78
Ag
107.87
79
Cd
In
Sn
Sb
Te
Xe
112.41
80
114.82
81
118.71
82
121.76
83
127.60
84
126.90
85
131.29
86
Po
At
Rn
Cs
Ba
Hf
fa
Re
0 s
Ir
Pt
Au
132.91
87
137.34
88
138.91
89
178.49
104
180.95
105
183.84
106
186.21
107
190.23
108
192.22
109
195.08
110
196.97
111
200.59
112
Fr
Ra
Ac
Rf
Db
Sg
Bh
Hs
[269]
[272]
[277]
La.
Hg
Ti
Pb
Bi
204.38
207.2
114
208.98
Mt
[268]
[2891
[2891
[2931
Chapter 1
Introduction
B.
Reading a book about physics is completely different from reading a novel. First
of all, you must be at a desk and have paper and a pen or pencil. You should write down
every equation, making sure you understand all the symbols. You should reproduce
every diagram, working to understand, for example, why certain forces are there and no
others, and so on. You may even try working the examples before reading the solutions
and work the solutions out along side the text.
C. Units
A widely held belief is that unit analysis is the least interesting activity of the
physical sciences. Indeed, carefully carrying units through a difficult formula is
sometimes about as interesting as painting a barn. But there are several good reasons to
pay attention to units.
You can lose valuable points if you;drop units, substitute into a formula, and
forget to convert cm to m or the like. One way to guard against this type of error is to
automatically convert any number to MKS (meters, kilograms, and seconds) as you
read the passage, or at least flag the units which are nonstandard (i.e. not meters,
kilograms, and seconds). Another way is to keep track of the units any time the units in
the problem are nonstandard.
Another reason to pay attention to units is that they can alert you if you have
written an equation the wrong way. For example, you may remember that flow rate f is
the volume (m3) flowing past a point per unit time (s) and that it is related to the
velocity v and cross-sectional area A of the pipe. But how do you relate f [m3/s],
v [mls], and A [mZ]?
The only way to correctly obtain the units is to write something like
f=Av,
that is,
where we may have left out a proportionality constant. In this case the formula is
correct as written. Units may bring back to mind an equation you would have forgotten,
counting fcr valuable points.
A third reason for keeping track of units is that they sometimes guide you to an
answer without your having to use a formula or do much work, as the next example
shows.
Example: How much volume does 0.4 kg of oxygen gas take up at T = 27" C and
P = 12 atm? (Use the gas constant R = 0.0821 L atm/K mol.)
Solution: Well, to the question, "How much oxygen?', we can answer either in
kilograms or in liters. The problem gives kilograms and asks for liters, so this is a
complicated units conversion problem. We will essentially construct the ideal gas
equation using the units of the elements in the problem. We start with 0.4 kg.
(amount of 0,) = 0.4 kg 0,
In order to apply the ideal gas equation we need to convert to moles. We can do this by
including the factors
103g0,
-
(l*go2
ImoleO,
)(
32go2
(:"k
2X1rz)
~
Now we include a factor of R because it has liters in the numerator and moles in the
denominator. We obtain
(amount of 0,) = 0
(:"A
1
~
LF@L
32fi
)("-oF2m)
This leaves us with units of atm and K which we want to get rid of. In order to cancel
them, we can just put them in. This may seem strange, but it works. (Recall 27" C =
300 K.) Thus we obtain
300 L.
- 0.4~1000~0.082132.12
For MCAT problems we generally work to one digit of accuracy, so we replace 0.0821
with 0.08, so that we have
(amount of 0,) =
It is generally safe to round to one significant digit. If it happens that two choices are
close, then you can always go back and gain more accuracy.
This example involved more arithmetic than most MCAT problems, but its
purpose was to point out that attention to units can speed up the solution to a problem.
If this is the way you normally do such a problem, good. Most readers, however, would
take longer working through this type of problem, using up valuable seconds on the
MCAT. Remember that seconds can add up to points.
Ax=vdt.
It makes sense that, in a given time; we can go twice as far if we go twice as fast. Thus
Ax is proportional to v. On the other hand, for a given speed, we can go twice as far if
we navel twice as long a time. Thus Ax is proportional to dt. We would never be
tempted to write
v=AxAf,
because these equations give relationships among the quantities that we know to be
wrong. Note also that the units work out correctly only in equation (1).
Another example is the second law of motion, which we will encounter in
Section 3.B. If an object has a single force on it, then its acceleration is proportional to
the magnitude of the force and inversely proportional to its mass. Instead of words, we
simply write
Now let's think about the equation. What would we do if we forgot it? If we stop
to think about it, we could figure it out. First, we know that force, mass, and acceleration are connected somehow. If we have
two objects of the same mass, and we
apply three times as much force to the
second object as to the first, then we have
a picture like that in Figure 1-1. The
greater force causes the greater acceleration, so we can guess that they are
proportional. We write
Figure 1-1
a - F.
If we apply the same force to two
objects of different masses, then we
expect the smaller object to accelerate
more (Figure 1-2). Thus we can guess that
the acceleration is inversely proportional
to the mass, so we write
a-
-.
m
F
a=-
Figure 1-2
m'
as in equation (2).
When you take the MCAT, you really should have the equation F = ma in your
head, but if you train yourself to think this way, it will be easier to keep the formulas in
your head. This will make it possible to recover the formula if you forget it. And you
will understand physics better. Most importantly, you will be better able to apply the
concept behind the equation.
Chapter
where G is a constant, m, and m2 are masses of objects, and r is the distance between
them. How would you ever remember this equation?
Well, start with the idea that objects with more mass have a greater force of
gravity between them, so write
Fgnv - mim2.
Also, if objects are far apart, the force of gravity between them is less, so write
The only part that needs to be memorized is the "square" in the denominator, so that we
have
(4)
What happens to the area when the radius increases by a factor of 3? (Answer: It
increases by a factor of 9.) Try it with r, = 4 m and r2 = 12 m, or with some other
numbers.
Another equation is that for the surface area of a sphere:
What happens to the surface area of a sphere when the radius increases by a factor of 3?
(Answer: It increases by a factor of 9. Surprised? What about the factor of 4? Try it
with r, = 4 m and r2= 12 m.) The surface area of a sphere is an equation that you just
have to memorize. It is difficult to get an intuitive grasp why the 4ashould be there. On
the other hand, the 2 is natural in this equation. Why? (Think about units.)
Book
Chapter
A.
B.
Chapter 1 Problems
C.
D.
5.
2.
A.
B.
C.
D.
7.
It increases by a factor of 2.
It increases by a factor of 4.
It increases by a factor of 8.
It increases by a factor of 16.
The
8.
12. The length of the rod of a certain pendulum is decreased, and the period then decreases by 20%. By how
much was the rod length decreased?
A. 20%
B. 36%
C. 40%
D. 44%
where T is in s, 1is the length of the string or rod (in m), and
g is the acceleration due to gravity (m/s2).(See figure.)
'
11. If the length of the suing of a pendulum is increased by
a factor of 4, how does the period change?
A. It decreases by a factor of 16.
B. It increases by a factor of 2.
C. It increases by a factor of 4.
D. It increases by a factor of 16.
GO ONTOTHE NDCTPAGE
Chapter
passage
1
[You d o not need to have any prior knowledge of e l e c t n c i ~
to deal with this passage.]
V = Ed,
where s is the length of a side, and h is the perpendicular
height.
capacitor
&
eleclric
field
--I-
voltage
source
2.
B.
C.
D.
4.
2.
3.
If the force between the balls stays the same, but the
charge q2 is multiplied by 4, which is a possibility?
A. The charge q, is also multiplied by 4, and all else
is unchanged.
B. The separation is decreased by a factor of 2, and
all else is unchanged.
C. The separation is increased by a factor of 2, and
all else is unchanged.
D. The separation is increased by a factor of 4, and
all else is unchanged.
4.
It is twice as great.
It is four times as great.
There is no force on the helium.
C.
D.
5.
18N
48 N
Two charged balls which are near each other will exert
a force on each other: attractive if they are oppositely
charged, and repulsive if they are similarly charged. The .
magnitude of the force is given by
4.
3,4, and 5
D.
3 only
Passage 3
Passage 4
2.
A.
2 and 3
B.
3 and 4
C.
4 and 6
D.
5 and6
B.
C.
0
1
D.
2.
A.
B.
C.
D.
The
3.
Scott drives a very large 50s style car, and Laura drives
.a small 90s style car, so that every linear dimension of
Scott's car is double that of Laura's car. On the basis of
energy loss due to air resistance alone, how much more
energy would you expect Scott's car to expend gening
from Tucson to Phoenix than Laura's car?
A. Twice as much energy.
B. Four times as much energy.
C. Eight times as much energy.
D. Sixteen times as much energy.
4.
5. Julie modifies her car, so that the effective crosssectional area is reduced by 20%. How much further
can she drive and still use the same amount of energy?
A.
10% further.
B.
20% further.
C. 25% further.
D, 44% further.
STOP
1 Chapter 2
Mechanics is about the motion of things. Before we can talk about motion in
depth, we need to be able to describe motion and the things which affect it. Objects
move, and we talk of how fast they go, that is, velocity. Their velocity changes, so we
talk of acceleration. We can think of changes in acceleration, but it turns out (happily)
that we rarely need to. Mechanics is concerned mainly with changes in velocity.
In this chapter we look at the fundamental elements of mechanics: force, mass,
distance, velocity, and acceleration. Comparatively this chapter has a lot of equations
(six that you should memorize) and the least interesting physics. It is an unpleasant way
to begin, but it must be done.
A force is a push or pull, and the units for force are [Newtons = Nl. (Some
countries continue to use an archaic unit called the "pound".) A Newton is approximately the amount of force that you would exert on an apple near the Earth's surface to
keep it from falling.
Examples of forces include the force of-a horse pulling a cart, the force of a
spring pushing the chassis of a car, the force of gravity pdling you down, and the force
on your head due to pressure when you are at the bottom of a pool. . >..
We can think about mass in several ways. First, the nuzss of an object is a
measure of the total amount of material (or stuff) in the object. The amount of stuff in
an object is a fundamental property of the object. It doesn't change if you move the
object to a new place, like a mountain top or to Mars.
There is another way to think of mass. The mass of an object is a measure of how
difficult it is to get it moving at a certain velocity if it starts from rest. For example, if
John wants to set a car, initially at rest, to moving at 1 mls, he has to push hard for a
little while. We are assuming the car's motion has no friction. If John and the car were
on the Moon, his task would be equally difficult. The fundamental concept here is the
u &
mass of the car, not the astronomical body the car is on. (See ~ i ~ 2-1.)
Saying this another way, the mass of an object is a measure of how much it hurts
if your stub you toe on it. Stubbing your toe on a bowling ball is a painful proposition,
even on the Moon.
There is a wrong way to think about mass. Many people think the mass of an
object is a measure of how difficult it is to pick it up. But that definition depends on
where you are. It is easy to pick up a bowling ball on the Moon, but nearly impossible
on the surface of Jupiter. The difficulty in picking up an object is a matter of weight,
which is a force. And weight does depend on the astronomical body near by.
D. Vectors
In physics we often need to describe
direction as well as size. For example, two
forces F, and F, may both be 100 N and
be verybut
different
the crocodile's
dependexperience
acting on a will
crocodile,
I O O ~ W N
N'
t~
Chapter
2 ......... .. The
Language of Motion
Pf
1
2
A
FA= ( 3 ~ ) +' (
4 ~ =)25~N ~ ,
F,, = 5 N .
Also we write
Figure 2-5
If your trigonometry is rusty, now is a good time to relearn the definitions of sine,
cosine, and tangent.
A vector is denoted by a half-arrow on top of a letter, p, for example.
The
E.
To specify position, we must give three coordinates x, y, and z, generally measured in [meters = m]. The symbol ? stands for the coordinates (x, y, z). If an object
moves from one position to another, the vector giving the change in position is the
F. Velocity,
We can think of the velocity vector in terms of a speedometer reading with units
[meterskecond = m/s] and a direction. The magnitude of the velocity vector (that is.
just the speedometer reading) is called speed. The word "velocity" is sometimes used to
refer to the vector and sometimes to the magnitude. When in doubt, you should assume
it refers to the vector.
If an object is traveling such that its velocity vector is constant, we say it is in
uniform motion. An example is a car going a constant 30 m/s (freeway speed) west. We
can write the following equations for uniform motion in one dimension:
When you see a formula in this text, instead of speeding by it, slow down and
look at it. Ask yourself, "What is this equation telling me?'Quation (la) is just
another form of "distance equals rate times time" for an object in uniform motion.
Since v is constant, this tells you, for instance, that a car will travel twice as far if it
travels for twice the time. This makes sense.
Equation (lb) is like the first, only Ax is replaced by its definition x2- x,. Do you
see why it is this and not x, - x2 or x2 + x,?
But in some problems the velocity does change, and we must pay attention to
several velocities, that is,
v,
initial velocity,
v2 final velocity,
v,, average velocity.
and
The average velocity is defined as
Chapter
2 ...........
This is different from equation (1). Equation (2) is the definition of an average
velocity over a time interval when velocity is changing, whereas equation (I) defines a
constant velocity and only holds for time intervals when the motion is uniform.
60 m
-.-.--.
-..
60 m
50 m
b
120 m
Figure 2-6
G. Acceleration
When an object's velocity vector is changing, the object is accelerating. Examples include a car speeding up ("accelerating" in common parlance), slowing down
or braking ("decelerating", but physicists prefer to say "negatively accelerating"), and
turning. In three dimensions, we define acceleration by
The numerator for equation (3a) gives the change in the velocity vector, so there is an
acceleration if either the magnitude or the direction of the velocity vector change. We
will talk more about this in Chapter 6. In one dimension the definition of acceleration is
Example la: Take north to be positive. A car is traveling south and speeding up.
What is the sign of the acceleration?
Solution: Since the velocity vector points south and the car is speeding up, the
acceleration vector must point south. With this sign convention, acceleration is negative.
The
Example 2a: Take north to be positive. A car is traveling north and slowing for a
red light. What is the sign of the acceleration?
Solution: The velocity vector points north. Since this vector is shrinking, the
acceleration vector must point south. Thus the acceleration is negative.
Example 2b: What is the acceleration for the car in Example 2a slowing from
1 0 d s to 8 m/s in 1 s?
Solution: We write
Now we have three quantities, position, velocity, and acceleration, all related to
each other algebraically. Often it is helpful to visualize these quantities graphically. The
following principles apply
1. Given a graph of x versus t, the instantaneous slope at time t is the velocity v
at time t.
2.
Given a graph of v versus t, the instantaneous slope at time t is the acceleration a at time t.
3. Given a graph of a versus t, the area under the curve during interval At gives
the change in velocity v during that interval.
4. Given a graph of v versus t, the area under the curve during interval At gives
the change in position x during that interval.
Chapter
Figure 2-7
Av = aAt.
I
vK
11
Before you read the next example, consider an object thrown straight up. When it
reaches the top of its path, what is the direction of its velocity? What is the direction of
its acceleration?
Figure 2-9
Fire 2-10
Figure 2-11
Figure 2-12
The second point on Figure 2-10 still has a positive slope, but smaller. The third
point has a zero slope (see uppermost point in Figure 2-9). The fourth point has
negative slope, and the fifth point has a slope more negative still.
It will not come as a surprise if we draw a straight line through these points, as in
Figure 2- 11. We take the slope at three points, but it is easy to see that the slope is
constant and negative. We graph the acceleration in Figure 2-12.
Does this match your expectation? Particularly at the top of flight, did you know
that the direction of the acceleration would be down?
t(s)
Figure 2-13
6
x(m)
x (m)
'c/
31-&-+-
(in progress)
Figure 2-14
Figure 2-15
Chapter
2 ....... .. .. The
Language of Motion
I. Uniform Acceleration
If an object has a constant acceleration vector, we say it undergoes uniform
acceleration. Most MCAT problems involving acceleration will involve uniform
acceleration. For uniform acceleration, we have the following:
that is, the average velocity over a period of time is the average of the beginning and
ending velocities. This may seem like a natural definition of average velocity, but the
definition of v,, is given by equation (2), and equation (5) holds only for uniform
acceleration.
See Figures 2-7 and 2-8 for an example. The velocity v, is small, v, is large, and
v,", is exactly between them.
If we start with the definition of average velocity, we can write
Ax = vavgdfr
This is a useful equation if you do not have and do not need the acceleration (see
equation [7] below). Furthermore, if we substitute v, = v,+ adt (from equation [4]),
then we obtain
1
Ax = -(v, + (v,+ a d t ) ) d r ,
2
This is the first equation which may seem a bit arcane. You should memorize it anyway.
Working through the algebra will help you memorize it.
Example: A car is accelerating uniformly from rest. If it goes a distance din the
first second, how far will it go in the first four seconds?
Solution: We want an equation involving the quantities mentioned in the problem, a, v , = 0,Ax, and Ar,so equation (6) is it. With v, = 0,we obtain
II
J.
For uniform acceleration there are four equations you should know:
The first equation we have seen before, the modified "distance equals rate times
time" when velocity is changing. It should be easy to remember. The second equation is
just the definition of acceleration. The third equation was in the last section. The last
equation is the only one which is new, obtained by eliminating dt from equations (7)
and (8). It is useful for problems in which the time interval is neither specified nor
desired.
Example 1:A cat drops from a ledge 2 m above the ground. If he accelerates
10 m/s2 downward due to gravity, how much time does it take him to drop?
Solution: Let's choose "up" to be positive and DRAW A DIAGRAM (Figure 216). We write the quantities we know:
V0
m
=o-,
S
m
a = -10s2 '
Ay = -2m
a = -10-
A=?
We look for an equation which involves
these quantities and no others. Equation
(9) fits, so that
+
Figure 2-16
m
s2
Chdpter
Example 2: A man drops to his death from the sixth floor of a building (20 m).
As he is falling, his acceleration is a constant 10 m/s2 downward. What is his impact
velocity? (He was a bad man, and if he had not died many other nice people would
have.)
Solution: First we DRAW A DIAGRAM (Figure 2-17). The impact velocity is
the man's velocity just before he hits the
ground v,. Thus our information summary
m
is
v. =O-
The formula which contains this infotmation and nothing else is (10). so that
Figure 2-17
The
MCAT Physics
Chapter
Book
Problems
3.
4.
2.
t
1
24
A.
B.
C.
D.
Chapter
6.
9.
Language of Motion
2 ........... The
I
I
13. What is the car's average velocity for this time interval?
A. 2.5d.s
B. 3.5 m/s
C.
17.5m/s
This
cannot be determined from the information
D.
given.
14. How far does the car travel during this time?
D.
C.
D.
2.04 s
This cannot be determined from the information
given.
A.
B.
C.
D.
D.
30m
115m
130m
160m
26. A car is traveling 25 m/s when it passes kilometermarker 3000. The car accelerates at 0.02 d s 2 for the
next 500 s. What kilometer marker will the car pass at
that time?
A.
3015 km
B. 3030 km
C. I2000km
D.
18000 km
A.
B.
C.
D.
50m
2s
4s
8s
16s
26
Chapter
2 . ..........
time?
97
The
Passage 1
35. What can be said about the net velocity change Av?
A.
It is positive.
B. It is zero.
C. It is negative.
D. It is positive, except for one point.
Use the following information for questions 36 and 37:
A car backs up at constant velocity, then slows to a
stop. After it is stopped for a while, it accelerates and then
goes forward at constant velocity. Consider also the following graphs:
1.
2.
GO ON TO THE.NEXTPAGE
Chapter
5.
The Language of M o t i o n
2 ...........
1.
How far does the object fall in the time interval from
t=Oto4s?
A.
39.2 m
B. 78.4m
C.
156.8m
D. 313.6m
2.
3.
4.
5.
A.
Passage P
The
6.
B.
3Ax
C.
Ax+9
D.
9Ax
STOP
Chapter
Laws of Motion
A.
Most people think the above statement is a law of nature. Some very intelligent thinkers
thought it was a law of nature, including Aristotle (ancient Greek, no intellectual
lightweight) and more recently Descartes (famous philosopher). Because it is common
sense (right?) that if nothing pushes on an object, it eventually slows to a stop.
But sometimes closer scrutiny conflicts with common sense, and when that
happens we have to change our thinking, to retune our intuition, so that what once
seemed wrong now seems right. That can be difficult, but that's physics.
Galileo discovered this law, although it's generally called Newton's first law of
motion.
What does it mean for the forces to be balanced? Before we answer that question,
let's look at a few cases. In the following figures (Figures 3-la-f) we denote !he motion
of an object by "motion marks", so that
'((0
mea!s
right.
Case a. There are no forces. In this
case think of a rock in deep space moving
along. The velocity vector is constant,
meaning the rock continues traveling at.
constant speed to the right indefinitely.
"'0
Figure 3-la
The
Figure 3-lb
Figure 3-lc
Figure 3-ld
Figure 3-le
Fnel=F,
Fnelby
a
+ F2 +.--
Figure 3-lf
Figure 3-3
Chdpter
4
Fstop
Fgrav
a
Over the next several chapters there will be many problems to test your intuition
on the first law.
Figure 3 5
Figure 3-6
We have not proven this equation, but the discussion in the previous paragraph should
make it seem reasonable to you.
Thz
4, G;-.
Fnef
= 4 + 4 +". ,
2
Eel= m .
2
(2)
Equation (3a), for example, states that the sum of all the horizontal forces is mass times
the horizontal acceleration. We will discuss breaking vectors into vertical and horizontal components in Section 4.D.
Finally, we are able to make the connection between the units for force
and
[w
Example 1: Bruce pushes a car (500 kg) on level ground starting from rest with a
force 100 N.How long does it take to get the car rolling 1 d s ? (Assume no friction.)
Solution: We have the information m = 500 kg, v, = 0 d s , F = 100 N, and
v, = 1 m/s, and we want At. We can find acceleration from equation (I), so we obtain
a=-
lOON
500 kg
Chapter 3
.-V2
-Vt
At
V2
At'
Figure 3-7
Since the problem asks about the change in time, we can solve for At to obtain
Now, F and v2 stay the same, but rn is five times larger for wagon B, so At is five times
larger. The answer is 5t.
The third law of motion is not so much a law about motion as it is a rule of thumb
about pairs of forces. It is usually stated thus:
g, on object 1
42=-F21
'
of Motion
FsE
*
Earth
Figure 3-8
FsE=
p21
-L
Figure 3-9
Notice that the magnitude of the force of the player on the ball is the same as the that of
the ball on the player. But the player moves hardly at all, while the ball springs toward
another player. Why is the basketball affected more than the player? (Hint: Look at
equation 1.)
D. Force Diagrams
,
I
Already in this chapter we have seen a number of force diagrams. In this section
we discuss some rules for drawing force diagrams. There are two types of force
diagrams:
1. a diagram in which all the objects appear and the forces come in third-law
pairs, and
2. a diagram featuring one object and all its forces (or maybe several objects, but
not all the objects in the situation).
EE,
Figure 3-11
Figure 3-12
FvE=
gravitational force of vase on
the Earth,
Figure 3-13
The
Did your diagram look like Figure 3-15 A or B? If so, you have not yet tuned
your intuition about the first law of motion. Just because the skate is going to the left
does not mean there is a force to the left. Only if the skate were speeding up to the left
would we be forced to conclude that there was a force to the left.
Figure 3-14
A
Figure 3-15
In this chapter we studied Newton's laws of motion. In a sense, the first law of
motion is the most subtle. If an object is moving at a constant velocity, then the forces
on the object add to zero, and if the vector sum of the force vectors for an object is
zero, then the object moves at constant velocity. Constant velocity means constant
speed in a straight path. No force is required to keep an object moving.
The second law of motion concerns objects whose force vectors' sum is not zero:
The acceleration of such an object is in the same direction as the total force, proportional to its magnitude and inversely proportional to the object's mass. That is
a'= $&. Do you see why we use equations?
The third law states that forces come in pairs: If object 1 pushes object 2, then
object 2 pushes object 1 in the opposite direction.
Pay especial attention to Section D on force diagrams. In solving problems; we
are always interested in the forces on an object at a given instant in time. These include
gravity, usually, and forces due to things touching the object at that moment. No other
forces need to be included. In particular, do not include a force in a direction just
because the object in moving in that direction.
Chapter
C.
Problems
D.
Section
Section A
5.
FB
A.
B.
C.
D.
2.
4.
A.
B.
Case 1
A.
B.
C.
D.
Case 2
Case 3
B.
C.
D.
40N
50N
120N
1200 N
The
15. A rocket ship (500 kg) is firing two jets at once. The
two jets are at right angles, with one firing to yield a
force of 5000 N and the other to yield a force of
12000 N. What is the magnitude of the acceleration of
the rocket ship?
A.
24 m/s2
B.
26m/s2
C.
34 m/s2
I
I
1
16. A girl shoves a 4-kg toy cart across the level floor with
a speed of 15 m/s (so it is going 15 m/s when it leaves
her hand). It slides to a rest in 5 s. Assuming a constant
force slowing the cart, what is the magnitude of the
force?
A.
0.75 N
B.
1.33N
C.
12N
D.
18.75 N
Chapter
C.
D.
Section
3 .. . .. . . . ... .. . . . . . . . . . Laws of
Motion
It is slowing down.
It is speeding up or staying the same speed.
A.
B.
C.
D.
120 kg
3300 kg
10,100kg
15,400 kg
B.
C.
D.
4t
2t
tl4
D.
GO ON TO WE NEXT PAGE
The
Passage
force vector C?
The first law of motion states that objects which
A.
are motionless have balanced forces.
B . The first law of motion states that an object in
motion will remain in motion unless acted upon
by an unbalanced force.
C. The second law of motion states that a force on an
object and acceleration of the object are proportional.
D. The third law of motion states that forces come in
equal and opposite pairs.
27. A car's engine has died, and the car is slowing down as
it coasts. What forces are acting on the car?
A.
Gravity, down.
Gravity, down; and the road's force, up.
B.
C. Gravity, down; the road's force, up; and friction,
backwards.
D. Gravity, down; the road's force, up; friction,
backwards; and a forward force.
28. An arrow is shot into the air. When the arrow is in the
air, what forces are acting on the arrow?
A. There are no forces.
B. There is the force of gravity.
C. There is the force of gravity and an upward
normal force.
D. There is the force of gravity and a forward force.
'
is uniform?
A. The entries for x are nonnegative and increasing.
B. The entries for v are nonnegative and increasing.
C. The entries for v are always greater than x.
D. Any interval Av is proportional to the interval dt.
2.
Chdpter
C.
D.
4.
Motion
B.
C.
D.
6.
0.0 m/s
0.1 m/s
0.15ds
0.3 m/s
2.
3.
4.
STOP
Chapter 4
Gravitation
When a person does a great deal of work in a scientific field, it often happens that
that person eventually receives credit for almost everything done by anybody (see
Matthew 13:12 in the Christian Bible). For instance, Newton is given credit, at least in
popular accounts, for almost every interesting thing that happened in science during the
Renaissance. In fact, then as now, science is the activity of a community, with many
people contributing to the revolution in thinking. For example, the essentials of
Newton's first law of motion were discovered by Galileo, and Robert Hooke surmised
the essential parts of the law of gravitation.
Newton's genius lay in his ability to see a simple underlying law for very
different phenomena and to synthesize diverse branches of science. An example of this
is his realization that both the motion of the Moon and the motion of a falling apple
could be explained by the same force, the force of gravity. In this chapter we will study
the physics of the gravitational force.
Newton's law of gravity states that any two objects exert an attractive force on
each other given by
Here F,, is the magnitude of the gravitational force between two objects, m, and m2 are
the masses of the objects, d is the distance between the centers of the objects, and G is a
universal constant 6.67 x lo-" m 3 k g s2.
Do not memorize G, but do remember the
equation. We discussed it in Chapter 1.
Figure 4-1
Thc M C A T Physics ~ o o k
Solution: Let's assume we have a spherical cow (Figure 4-1). We calculate
Example 2: What is the motion of an apple (0.1 kg) which has let go of its tree?
Solution: First, we DRAW A DIAGRAM (Figure 4-2) showing all the forces on
the apple while it is falling. There is only the force of gravity (nothing else is touching
it), so we write
= Fnet
L
Figure 4-2
So the apple accelerates downward at 9.8 rn/s2.
Equation (1) is easy to use in two types of problems:
1. obtaining the force between two planets (d is much larger than the radii of
the planets), and
2. obtaining the force between a planet and a small object on its surface (d is
essentially the radius of the planet).
The previous examples illustrated this second use.
where we use a calculation from the previous section. This number 9.8 mls2 comes up
so often in introductory physics that we have given it a name: The acceleration due to
gravity is
Fpv = mg.
(3)
C. Free Fall
An object is said to be infree fall when nothing is touching it, so that the only
force on it is gravity. Such an object is called a projectile. The simplest problem in free
fall involves dropping objects near the surface of the Earth. We want to know which
falls faster, a heavy object or a light one? Things become complicated if the object is
too light, like a leaf fluttering to the ground, so at first we will consider two objects for
which air resistance is only a small consideration. (We discuss air resistance in Chapter
6.) Let us start by doing a pair of examples.
Example 1: How long does it take a small rock (0.02 kg) to fall from rest
2 meters to the ground?
Solution: First, we DRAW A DIAGRAM (Figure 4-3). There is only one force
since nothing touches the rock and we are neglecting air resistance. Second, we find the
net force
Figure 4-3
where the negative sign reminds us that gravity points down.
Book
F, = ma,
Ay = -2 m,
so that
At = 0.63 s.
I
I
Figure 4-4
Example 2: How long does it take a medium-sized rock (0.2 kg) to fall 2 meters
from rest?
Solution: The rock is larger, and so is the force of gravity (and the force arrow,
see Figure 4-4). We write
=-2N.
The acceleration is
F,,
-2N
a=-=-=-107.
M 0.2 kg
m
s
'Q
Chapter
4 ............................
Figure 4-7
What is going on in the previous example? Just after the grapefruit is released
from the hand on the second ship, it still has its horizontal motion. If air resistance does
not affect it, then it maintains its same horizontal motion from start to finish. The
vertical motion, on the other hand, proceeds on schedule regardless of the horizontal
motion. At t = 0.2 s, both grapefruits have moved vertically 0.05 m (the second has
moved horizontally as well). As time goes
on, the second grapefruit keeps up with
the ship, and both grapefruits hit the deck
at the same time.
If you do not believe the figure, try
the experiment yourself. While walking at
a constant speed, release an apple.above
your head (and a little forward). It will
fall in front of your face and land at your
Figure 4-8
feet. (See Figure 4-8.)
Gravitation
The
MCAT Physics B m k
..
.
The following box shows how this principle gets translated into equations:
(F,)y
=m
y .
1
dY = -(
2 v,, + v 2 , ) 4
V Z y = v,, + a, A,
1
Ay = vlyAt+-ayAt2,
v,,2 = v:,
+ 2ayAy.
Chapter
Objects in free fall experience only the force of gravity, so we can say more:
m
a, = O T 2
(4b)
m
where "up" is positive and we use the estimate 10s2 .
Example 1:A cliff stands 80 m above a flat plane. One cannonball is dropped,
and another is fired horizontally at
120 rnls at the same time. How far from
the first ball will the second ball land?
Solution: The first ball falls stnight
down, of course. Let's DRAW A DIAF,,
GRAM for the second ball while it is in
I"
flight (Figure 4-10). Note that the cannon
exerts a force on the cannonball while it is
Figure 4-10
in the cannon, but after the ball leaves the
cannon, the only force is gravity.
We record the information we have. We do not know the mass of the ball, but
from Section B we know we do not need it. The acceleration vector is 10 d s 2 . down.
vertical
VlY
horizontal
m
v,, = 1205
=0
Ax = 480 m
We solved the vertical problem first because we had more vertical information
than horizontal information. The time dt = 4 s was the connection between the horizontal and vertical parts. You should work through this example yourself without looking at
the book.
51
The
:
30"
A
Vlr
Figure 4-12
VlY
..---a-----.--.
-.
&io: -
-----.
Figure 4-11
Interruption: We need to know the
horizontal and vertical components of the
initial velocity v',. We need to find a
horizontal vector v',, and a vertical vector
v',, so that their sum is the original vector
v', (see Figure 4-12).We can find the
magnitudes of v',, and v',,, using simple
trigonometry. (You may need to review
trigonometry at this point.)
v,, = 150&m
S
Soiution: The force diagram is the same as in Figure 4-10. The cannonball rises
and then falls to the same height from which it started, so we have Ay = 0 m.
vertical
Ay=Om
Ax=?
m
s
v,, = 150-
horizontal
We have been talking about grapefruits and cannonballs so far. Objects with a
more complicated shape obey the same rules, as long as we use the center of mass to
talk about the position of the object.
Figure 4- 13 shows a baseball bat fired
/\-----/
*
+ /
from
in
a parabola,
a cannon.just
The
like
center
the cannonball
of mass moves
in
the previous example, even though the bat
is rotating. In fact this is a definition of
the center of mass. If an object is set to
fnely rotating, the center of mass is the
point which refuses to rotate. The
gravitational force acts as if it were
exerted only at the center of mass.
Figure 4-13
m
= 10 T .
S
When any object near the Earth's surface has only gravity acting on it (freefall), it
m
has a downward acceleration vector of magnitude 10-. This curious result comes
s2
from the fact that the force of gravity is proportional to the mass, while the second law
of motion states that the acceleration in inversely proportional to the mass.
We also explored the principle that horizontal and vertical motion are independent. This allows us to solve problems involving projectiles, that is, objects with only
gravity acting on them. We will see more of this principle in the following chapter.
Chapter 4 Problems
4.
C.
D.
24 kg
72 kg
Sections A-C
5.
B.
C
D.
C.
D.
2 m/s2
4m/s2
The Earth and the Moon attract each other with the
force of gravity. The Earth's radius is 3.67 times that of the
Moon, and the Earth's mass is 8 1 times that of the Moon.
The acceleration due to gravity on the surface of the Moon
is one sixth the acceleration due to gravity on the Earth's
surface.
9.
D.
6.4 N
GO ON TOTHENEXTPAGE
The
Section
D. 5.2 m/s
17. What vertical distance has the can opener fallen in the
two seconds?
A. 3 m
B. 20m
C. 20.2m
D. 23m
B.
C.
D.
1.5m/s
5.0m/s
5.2d s
B.
20m
Chapter
23. When the ball is in midair (point B), what is the net
force on the ball?
A.
B.
C.
D.
1.96 N
3.92 N
9.8 N
19.6 N
C.
D.
A.
B.
C.
D.
0.26 s
0.51 s
0.83 s
1.01s
29. When the coins are in midair, how does the acceleration
of one of Alice's coins compare with the acceleration of
one of Barbara's?
A.
It is one fourth as large.
B.
C.
D.
It is the same.
It is four times as large.
It depends on the height at which the acceleration
is recorded.
30. How does the time to reach the ground for one of
Alice's coins compare with the time of fall for
Barbara's?
A.
B.
C.
D.
Gravitation
28. When the coins are in midair, how does the gravitational force on one of Alice's coins compare with the
force on one of Barbara's?
A.
B.
0.76 m
1.47 m
1.25 m
2.25 m
A.
B.
C.
D.
31. Just before Alice's coin reaches the ground, it has speed
s,. Just before Barbara's coin reaches the ground, it has
speed s,. How does s, compare with s,?
A. The speed s, is less than s,.
B. The speed s, is the same as s,.
C. The speed s, is greater than s,.
D. It depends on the height of the building.
Use the following informationfor questions 32-36:
A woman (50 kg) is pulling a wagon behind her. In
the wagon is her daughter by her first marriage; the daughter
and the wagon are 60 kg. (See figure.) The woman pulls the
handle with a tension 200 N, and the handle makes a 30"
angle with the horizontal. There is a horizontal force of
friction, and the wagon moves at a constant 2 m/s. (Use
g = 10 ds2.)
'coin B
The
MCAT Physics B o o k
A.
B.
C.
D.
D.
ON
1OOON
5000N
9000 N
600N
800 N
I
58
4.
B.
C.
D.
2.
D.
13m/s
Passage 2
The
ing:
G = 6.67 x lo-" N m2/kg2
M,,,= 2.0 x
kg
(volume of a sphere)
, = 4n
~ 2~
Acutie = n 4
(area of a circle)
1.
2.
Je
times stronger.
B.
C.
D.
STOP
Chapter 5
Planes and Circles
Again
In the last chapter we solved problems with gravity as the only force. Well,
gravity is a fine force indeed, but we need to understand problems in which other forces
are present. That is the goal of this chapter.
In the last chapter we discussed the independence of venical and horizontal
motion for objects in freefall, but it turns out the principle works when other forces are
acting as well:
LLY
= &y + Fty +.--.
(1)
Y
a =-.F
(2)
'
+ FZx+..-,
(3)
(4)
The
k)
1
A
T
T
2 = sin 30"
= cos 30"
C Fg,
= mg
Figure 5-2
Using this and v,, = 0 rnls and At = 3 s,
we derive a horizontal velocity
vlx = v,,
+ axAt
N = Fm- Ty
=mg-Ty
= 90 N.
Notice that the normal force is not the same as the gravitational force, a mistake often
made by students. Why is the normal force not the same as gravity?
Chdpter
5 . .. .................
Note that in solving part a, we used information about forces to obtain the
horizontal acceleration a, and then the answer. In part b we reasoned the other way,
using information about the vertical acceleration a,= 0 to obtain information about the
normal force. This strategy of reasoning in both directions will be useful in many
problems.
Wizo
Fx
-.
Fgnv
Figure 5-4
and F.,
On the other hand, in physics it
is generally true that two small angles
which look congruent are congruent(!).
Y -cos 8
4rsY
~y = <rav cos e
F, = FBmv
sin 8
ex,so we write
F, = m a x ,
mgsine= ma,.
gsin8=aX,
C. Circular Motion,
Qualitative Description
FB,
Figure 5-5
Chapter 5
.
....... . . . ... . . . . . . .
In normal English, we do not say that the car is accelerating when it is turning.
But in physics language, an object that is moving at constant speed in a circle is
"accelerating toward the center of the circle", because the velocity vector is changing.
We call this centripetal acceleration, which is Latin for "toward the center". (Parenthetically, "centrifugal" means "away from the center".) The force which-provides the
centripetal acceleration is the centripital force.
bt-f)
Sun
Earth
Figure 5-7
fl
D. Circular Motion,
Quantitative Description
Tangential acceleration
Centripetal acceleration
Aspeed
aUng= -,
At
v2
a,,, = - r
(5)
(6)
Figure 5-10
Chapter
Example 2: Use the following information to find the mass of the sun:
._.._.'
-....
,
.
I
.
e+ 4
Sun
....
~ & h
...-....-..__.________.~..-.~
Figure 5-11
We use M-on
the right-hand side of the equation, because it is the Earth's acceleration we are concerned about. The Sun's acceleration is much smaller because its mass
is larger. (Recall that the force the Earth exerts on the Sun is the same as that which the
Sun exerts on the Earth.)Note that M,,cancels.
What expression shall we use for v? What is the velocity of the Earth? If the
Earth travels a full circuit in a year, then velocity is simply distance per time, where the
distance is the circumference of the circle. Thus we write
where T is 1 year. Do not simply memorize this, but take a minute to think about why
this equation is true, so it will come immediately to mind in any similar situations.
Substituting this into equation (7) and doing some algebra gives
into which we can substitute the values given in the problem, along with
hOurr)(e)
1hour
3 10' s
T = 365days(24lday
= Y
to yield
M,, = 2 x loMkg.
The importance of this example does not lie in the arithmetic. The important parts
are the method of setting two expressions for the same force equal to each other and the
use of equation (8).
Figure 5-12
Stop! Think about his question and
answer it before you look at the solution on the next page.
The most important concept in this chapter is the independence of horizontal and
vertical motion. We saw this concept for the first time in Chapter 4. In this chapter we
have used the concept to solve problems involving inclined planes and oblique forces,
that is, forces which are neither horizontal nor vertical.
We also looked at circular motion. If an object is moving in a circle, its velocity
vector is not constant, and the object must be accelerating. If the object is moving at
constant speed, then the direction of the acceleration vector is toward the center, called
, = v2/r.
centripetal acceleration, and its magnitude is a
The
Chapter 5 Problems
3,
How much time does it take the orange to reach the top
of its path?
A. 0.25 s
B.
0.4 s
C. 0.5 s
D.
2.5 s
4.
5.
Section A
Use the following information for questions 1-5:
A cannon shoots an orange (3 kg) straight up in the air
with initial velocity 5 m/s (see figure). A horizontal wind
exerts a force of 6 N on the orange while it is in the air. Use
10 m/s2 for the acceleration due to gravity.
T>F
D.
T+F=G+N
2.
A.
B.
C.
D.
ON
2N
30N
32 N
70
ChaDter
12. Which of the following is the best force diagram for the
sled after t = 0 s?
A.
B.
C.
D.
ON
10N
50N
67 N
0 m/s2
B.
0.5 m/s2
2.0 m/s2
9.8 m/s2
C.
D.
The
Section
21. Which arrow best shows the direction of the net force?
A.
B.
C.
D.
A
B
C
D
B.
C.
D.
Tension.
Friction.
Normal.
23. Which arrow best shows the direction the stopper would
go if the string were to break at the moment shown in
the diagram?
A.
B.
C.
D.
Use the following informationfor questions 19-23:
A stopper is swung 0n.a string, one end of which is
fixed at a point P. The diagram shows the stopper and string
from the top, and the stopper is swinging counterclockwise
at constant speed.
A
B
C
D
24. Consider the car as viewed from the top. Ignore the
gravitational and normal forces (which are vertical and
add to zero anyway). Which is the best force diagram?
B.
C.
D.
B
C
20. Which arrow best shows the direction of the acceleration vector?
A. A
I
I
Chapter
All Sections
D.
D.
10 rn/s2
30. The figure shows a ball (from the top view) rolling on a
table with a partial hoop. Which arrow best describes
the path of the ball after it leaves the hoop?
B* 8
C.
D.
.-
Mgsina-T
73
NIM-gcosa
TIM -gsina
The
B.
C.
D.
A.
B.
C.
D.
lOlcmls
20lcmls
100lcm/s
200lcmls
Chdpter
and Circles
1.
0
Sun
A'
B.
REarlhlTday
C.
D.
gTday
2xgTda,
21tR-lT,~
2.
3.
gm,points into
the page.
4.
75
GO ON TO M E NEXT PAGE
It would be less.
B.
C.
D.
STOP
A.
Introduction
So far in this book, we have ignored friction in order to make problems easier and
to uriderstand the basic principles behind motion. On the other hand, there cannot be
too many practical applications of such a theory without friction, since few of the
surfaces in this world are frictionless, and it is difficult to go anywhere without air
resistance (especially if you go by car).
Friction is a force which opposes the slipping of two surfaces, so it acts parallel
to the boundary between the surfaces. We generally think of friction as the force that
slows things down, but that is not the best way to consider it. When you step on the
accelerator of a car, what force makes the car go faster? None other than the friction
between the tires and the road. What happens when you try to accelerate on ice?
There are-two types of friction: staticfriction, which is relevant when the surfaces
are not slipping, and kinetic friction, which is relevant when the surfaces are slipping.
B. Static friction
Let us consider an example. Muffin the cat is trying to budge a waste-paper
basket in order to see what is under it.
Before she starts, the forces on the basket
are those shown in Figure 6- 1. She begins
to push, but the basket does not budge.
The force of friction has shown up and
exactly balances her pushing force
(Figure 6-2). When she pushes harder. the
frictional force becomes larger, frustrating
her effort (Figure 6-3).
Figure 6-1
Figure 6-2
16..
Figure 6-3
maximum
Ifthe basket is not moving, we can
surmise that thefriction is the same
magnitude as the push force.
Figure 6-4
Also
F,,,, = PsN*
where &, the coefficient of static friction, depends only on the
materials involved. It has no units and is generally less than 1.
&
Example: Beth (45 kg) has tied a rope around her brother's waist. Vincent (20
kg) is on the slippery slope of a river bank making an angle 30' with the horizontal.
The coefficient of static friction between him and the bank is 0.2.
a. If she does not pull on the rope, will he slide down into the river, which is
infested with crocodiles?
b. If so, what is the smallest force she must exert parallel to the bank in order to
keep him from slipping?
Solution: First, we need to DRAW A DIAGRAM (Figure 6-5). First, there is the
force of gravity. The bank is touching him, so there is a normal force. There is also
friction and possibly the rope. At any rate, these act along the bank, so we label them
pd, that is, the force needed to keep him from sliding.
Figure 6-5
In Figure 6-6 we divide gravity into components.
Figure 6-6
The "vertical" equation becomes
~ - m g c o s 3 0 "=(F,)" = O
because if Vincent does not move, then a,= 0. Thus we have
N = mgcos30 =
= 170 N.
The "horizontal" equation becomes
Air Resistance
F,,,
= psN = 0.2(170N) = 34 N,
C. Kinetic friction
Once the static friction maximum is exceeded, the surface involved in a problem
begins to dip, and we have a problem involving kinetic friction.
F,.= P,N,
where is the coefficient of kinetic friction, N is the normal
force, and the direction of the force is parallel to the surface in
opposition to the slipping. In generalp, is less than &, so once
an object is moving, the force of friction is less than the
maximum friction when the object is still.
I
You might have thought that, the faster an object slides, the more friction it
experiences. This is not true. (It is true for
air resistance, but not friction.)
Also, you might have thought that
there
would
be more friction for an object
Fk
Pprrh
2kg
with more surface touching (see the
second picture in Figure 6-7), but again,
this is not true. The friction depends only
Fprw
on the coefficient of friction and the
normal force.
- A
Figure 6-7
Chapter
Example 1: Brad pushes a stove (100 kg) in a straight path across the level floor
at constant speed 0.2 mls. The coefficient of kinetic friction is 0.3 for the stove and the
Boor. What is the force that Brad must apply?
Solution: First, we DRAW A
DIAGRAM (Figure 6-8a). The words
"constant speed and "straight path"
should send bells off in our head. There is
no acceleration, so the vertical equation
becomes
N=mg=(lOOkg)
Figure 6-8a
Equation (2) gives the friction
Figure 6-8b
Figure 6-8c
F i r e 6-8d
Resistance
N = 1.15 N.
The horizontal equation becomes
Figure 6-9
T, - Fk = ( F , , ) ~= o ,
Example 3: A car (1000 kg) is traveling downhill at 20 m/s in the rain. The grade
of the road is 20%, which means that for every 100 meters of road, the vertical drop is
20 meters. The driver sees Bambi in the road and slams on the brakes. The coefficient
of kinetic friction between the tires and the road is 0.5. How much time does it take the
car to skid to a halt? (Hint: If 8 is the angle between the horizontal and the road, then
cos 8 = 0.98 and sin 8 = 0.2. Also g = 10 rn/s2.)
Solution: Here we will merely sketch a solution. You should try to work out the
details. Figure 6-10 shows the force diagram. Working out the vertical equation with
(F,)
= 0 gives
N = 9800 N.
(F,)~ = 2 W N ,
m
a, = 2.9s2 .
Figure 6-10
Using the acceleration, the initial velocity, and the final velocity v2,= 0, we obtain
At=7s.
(Bambi was unscathed, but only because he jumped off the road in time.)
Chapter
The fact that &is generally less than &means that, in many situations, the
friction will switch back and forth from kinetic to static. Picture pulling a potato with arubber band, so static friction initially prevails. After the rubber band stretches enough,
the potato moves and kinetic friction takes over. But then the rubber band has contracted again, so the potato stops, and static friction prevails. By this time you are
pulling again. This is called sticWslip, for obvious reasons.
The sticWslip phenomenon is responsible for the squeal of bus brakes. It is also
responsible for the eh-eh-eh-eh-eh sound when you rub your hair after a shower or your
dishes after washing. These are some very practical applications of physics.
E. A i r resistance
We have neglected air resistance thus far mainly for one reason, and that is that
air resistance makes problems more difficult. Air resistance, or drag, depends on the
velocity that an object is going, and not always in a simple way. Thus it is difficult to
work problems without a computer.
There are a few things we can say about drag. Consider an object moving through
a substance, such as air or water. It is reasonable that a larger object would experience
more drag than a smaller one. We might also expect that the drag would be larger for a
faster moving object. Finally it is reasonable to expect that a dense fluid would exert
more drag than a "thin" fluid. It will not be a surprise, then, that the formula for drag is
Fdrilg
= CPAV*,
where C is a constant equal to about 0.2, p is the density of the fluid, A is the crosssectional area of the moving object, and v is its velocity. This equation is reasonably
accurate if the fluid is "roughed up" a bit by the object's passing through. For cars
driving and people walking through air and for fish swimming through water, this is a
reasonable assumption.
You do not have to memorize this equation, but do know how to use it. The force
is placed in the force diagram like all the other forces, except it points necessarily
opposite the direction of motion.
Example 1: A dog Nikki (7 kg) falls five stories (3.3 meters each) down from a
roof with his stomach pointing down. He is 0.2 meters tall, and 0.15 meters wide and
0.4 meters from nose to tail. The density of air is 1.29 kg/m3.
a. If we ignore air resistance,
what is the terminal velocity of
the dog?
b. What is the air resistance by the
time the dog gets to the
bottom?
c. Is air resistance a small or large
effect in this problem?
Solution: a. First, we DRAW A
DIAGRAM (Figure 6- 11). We are hoping
that the force of air resistance is small. We
have
Resistance
We use the kinematic equation that does not involve At, so we have
v; = v:
+ 2ady,
b. The cross-sectional area presented by the dog is (0.4 m)(O. 15 m) = 0.06 m2. It
does not matter how tall Nikki is. Thus
c. Air resistance is about 10% of the dog's weight (70 this implies that the
dog's weight is the dominant force all the way down, and we are justified in ignoring
air resistance. Our answer is good with no more than about 10% error.
Nikki is a stunt dog. He was unharmed.
Example 2: A rubber ball (radius 2 cm, mass 5 grams) is dropped from a height
of 50 meters. What is its velocity when it reaches the ground?
Solution: If we work out the problem as in Example l a above, we run into
trouble. Ignoring air resistance yields v, = 32 m/s. (You should work this out.) The
cross-sectional area is the area of a circle d,
not the total surface area of a sphere. If
we calculate the force due to air resistance at the bottom of flight, we obtain
whereas the weight of the ball is F, = mg = 0.05 N. The calculated force F,,is not
small at all in comparison. In fact, it is much larger than the force of gravity. Any
assumption that air resistance is neglible is not valid. We need a new idea in order to
solve this.
The new idea follows. If the ball has
fallen so far that it has stopped accelerating then the gravity force down and the
air resistance force up are balanced. Then
the force diagram would look like the one
in Figure 6- 12. The force equation
becomes
Figure 6 1 2
Chdpter
As the ball falls, its velocity increases until it begins to get close to 12 rnls. The
air resistance increases until it balances the force of gravity. Force balance occurs when
v = 12 mls. This velocity is called terminal velocity.
Example 3: A man (60 kg) falls from a very tall building (200 m). The crosssectional area for his fall is 0.3 m2.
a. What equation governs the fall?
b. What is his velocity when he
reaches the bottom?
Solution: a. First, we DRAW A
DIAGRAM (Figure 6- 13). The force
equation becomes
F~c-mg=ma,
CPAV'- mg = ma.
Fprav
In this chapter we looked at friction and air resistance. Friction is a force which
opposes the slipping of surfaces, and it always acts parallel to the surface. Situations
involving friction fall into two categories: static and kinetic. If the surfaces are not
slipping, then static friction is the force which maintains the status. We solve for the
frictional force using force balance. The static friction cannot be larger than the
maximum, given by F,,, = pJ.
If the surfaces are slipping, then kinetic friction opposes the slipping. Its magnitude is given by F, = pJV.
The MCAT does not have many problems involving air resistance. Generally you
just need to know that air resistance is a retarding force which depends on the surface
area, density of the medium, and velocity of the object.
Resistdnce
Book
Chapter 6 Problems
D.
5.
B.
C.
D.
2.
0
Tcosa
Tsina
T
A.
B.
C.
D.
B.
C.
D.
&mg
Tcosa
8.
mg
mg - Tcosa
mg + Tcosa
mg - Tsina
Rolling friction.
Tsina
Chdptei
15. How hard would the man have to push to get the washer
moving?
A.
600N
B.
700 N
C.
D.
B.
C.
D.
1000 N
A.
80N
ON
0.04N
0.08N
0.4N
the card?
A. O N
B. 0.04N
C. 0.08N
D. 0.4N
C.
13. What is the normal force on the washer?
A. 600N
B. 700N
C. 800N
D. 1000 N
mgsin9
Book
B.
mg
mgsin 13
c.
PkN
A.
D.
25. What is the net force on the car if the turn is successful?
A. 6400 N
B. 7000N
C. 9000N
D. 10,000 N
20. What is the net force on the car during the skid?
A. 0
B.
C.
D.
@
@+mgsinO
@-mgsin6
21. What forces are acting on the car besides the gravitational force (down) and the normal force (up)?
A. A force toward the turn axis.
B. A force away from the turn axis.
C. A force in the direction the car is traveling and a
force toward the turn axis.
D. A force in the direction the car is traveling and a
force away from the turn axis.
Passage 1
A.
B.
6400N
7000 N
C. 9000N
D.
10,000N
--
88
Chapter
3.
After the floor drops, what force provides the cenuipeta1 force?
A. The normal force.
B. Friction.
C. Gravitation.
D. Tension.
After the floor drops, which direction does the acceleration vector point?
A.
Toward the center of rotation.
B. Away from the center of rotation.
C. In the direction of the rider is moving.
D. The acceleration is zero.
4.
5.
Mg
B.
C.
D.
w g
&Iv2/r
phlv2/r
Passage 2
p (kg/m3)
1.29
1.0 lo3
1.36 x lo4
7 (kglm s)
1.8 x lo-'
1.0 x lo5
1.6 x low3
D.
9m2
GO ON TO ME NEXT PAGE
4.
5.
6.
A.
B.
C.
D.
A.
B.
C.
D.
2.
0.46 m
0.92 m
1.84 m
176.4 m
A.
3 x 1 0 ~ ~
I only.
B.
~ x ~ o - ~ N
IIandIII.
Iand III.
I, 11, and 111.
C.
0.7 N
D.
50N
3.
Chapter
4.
5.
. . . .. . . . . . Friction
STOP
Chapter 7
Torques and Properties of Solids
A.
Introduction
In the first six chapters, we have talked as if all things were points or boxes which
move by sliding or gliding. In this chapter we will discuss the ability of real objects to
rotate, stretch, and bend. These topics are slightly more complicated than previous
topics, and their study quickly moves beyond the scope of the MCAT. For this reason,
we will cover only the main points and the simpler problems. Even so, you may find
this chapter difficult. So work carefully, and if you do not understand everything the
first time, work through the rest of the book before coming back to it.
B.
Language of Rotation
smaller moment
of inertia
C. Torque
In order to calculate a torque, we
always have a pivot Po(where the axis is)
and a force acting at another point P,. For
example, in Figure 7-3, the pivot is at the
crocodile's belly, and the force F acts at
his snout at P,. The torque is defined by
Theforce produces a
torque about point Po.
Figure 7-3
I
I
(
I
(
where 7is the torque, r is the distance from Poto PI, F is the size of the force, and $ is
the angle between the direction of the force and the line Poand PI.But is 4 the big angle
or the little angle? Well, it turns out it doesn't matter, since we are taking the sine, and
the sines of supplementary angles are the same. Recall that
sin 0" = 0,
(4)
sin 90" = 1,
sin 180" = 0.
Also the convention is that
counterclockwise = positive torque,
clockwise
-=
negative torque.
Up until this chapter, we have drawn force vector arrows anywhere as long as the
tail of the arrow sat on the object the force acted on. In doing torque problems, we must
be more careful to put the arrows in the right place.
Chapter
. .. Torques a n d Properties
Figure 7-4
The angle $is shown in the two places in the diagram (corresponding angles with
the parallel lines). The sine of @canbe obtained by looking at the portion of the
diagram shown in Figure 7-5:
sin i$ =
opposite - 0.4 m
= 0.8.
hypoteneus 0.5 m
r= rFsin$
=(0.5 m)(1.5 N)(0.8)
= 0.6 Nm.
Figure 7-5
n +
II .
(
01 Solids
I
1,
In this example, this trick is the equivalent of sliding the force up the string to the
meter stick. In fact, the example is exactly equivalent to Example 2.
a.
T = 20 Nm.
b.
T = -20 Nm.
c.
.r=ONm.
Example 1, revisited solution: We
can slide the 1-5-N force down the edge of
the card to the x-axis. In this case we have
r = 0.4 m and sine = 1, so we have
7 = (0.4
?.
0.4 m
t
m.
i, Fold
0.3 m
Fnw
w
Po
Figure 7-8
p1
Chdpter
Torques are useful in problems involving rolling and spinning, although most
such problems lie outside the scope of the MCAT. Torques are also useful in solving for
forces in structural problems, even if there is no motion. For these problems we use the
( F , , , ) ~= 0,
Pa)
(FKJY
= 0'
(5b)
and
7, = 0,
(5~)
Equations (5a) and (5b) assure translational equilibrium, and equation (5c)
assures mtafional equilibrium. The following examples illustrate methods for calculating forces in static equilibrium.
Figure 7-9
met= 0,
z,- r2= 0,
m,=6 kg.
For problems involving torque balance, the following methods often work:
1. DRAW A DIAGRAM.
2. Label all forces acting on the system.
3. Choose a pivot.
4. Calculate torques and set T,,=,,,=
0.
5. Use (F,,,), = 0 and (F,,J,= 0, or choose another pivot.
In the following example, it will be
helpful for you to work out the example
as you read along.
Figure 7- 11
I.
Figure 7-10
What is the tension in the wire
a.
from B to C in terns of m , , m,,g, L,
and 8?
What is the vertical force exerted
b.
by the wall on the pole?
Solution: First, we DRAW A
DIAGRAM with all the forces on the pole
(Figure 7-1 1). We know m,g and m g but
not T, F,, and F,.Let us make a chart
showing the toques about A and C.
force
torque about A
m1g
--L2 mlg
torque about C
Make sure you understand why these two entries are correct. Then fill in the rest
of the chart on your own. The completed chart is shown on the next page.
Chapter
toraue about A
force
7 .. . Torques and
Properties of Solids
toraue about C
In the second column the entries for the torque due to Fxand Fyabout A are zero,
since r = 0 for these entries. In the third column, the entry for the torque due to Fxabout
C is zero because sin$ = 0.The torque due to T about C is zero because r = 0.
a. In order to find T, we can take torques about point A, since that eliminates F,
and Fy.The net torque must be zero because the system is in equilibrium, so we have
(taking the sum of the first column)
b. In order to find Fy,we can use one of two methods. Using torques and choosing
point C as a pivot yields
On the other hand, we could add up all the vertical forces and use (F,,), = 0,so
that
Fy+TsinO-m,g-m2g=0,
Fy= m , g + ~ g - T s i n O
=*~g+Qg-
m1g + 2m2g
sin 8
- 1
- -m,g.
2
This was a longer solution, so using torques is clearly the way to go.
Book
Figure 7-12
Figure 7-13
sin a
cos a
b. In order to find F,, we set the sum of vertical forces to zero, so that
(Fnedy
=0,
The negative sign tells us that we drew the F, vector the wrong way. (Perhaps you
knew this already.) The force is compressional.
E. Solid Properties
We have also been pretending that sticks and strings and such things are absolutely rigid, but we know that solid objects do stretch and bend and sometimes break.
Solids bend when you exert different forces on two sides of them. To keep things
simple, we will look at how a solid cylinder reacts to forces placed on it.
Chapter
Figure 7-14
AL
F =Ay-. L
-
lr-$A
(6)
1i
F A X
-=s-.
A
(7)
1-01
F
A
plastic region
klastic limit
In this chapter we looked at using
torques in order to solve for forces in
-elastic
region
certain static structures. In problems of
this type we begin by drawing a force
AL.
diagram, as we have always done, but
L
now we need to be careful to locate the
force at the right place. Generally, in a
Figure 7-18
given problem, there will be forces we do
not know and do not need, and it will be
possible to choose a pivot solhat the torques of all such forces are zero. Then if we
write down the torque balance equation ,z, = 0,we will be able to obtain the magnitude
of the desired forces.
We also looked at the static properties of solids. It is helpful to think in terms of a
stress (force per area) being applied to a solid, and this stress causes a strain (displacement per length). For many materials stress and strain are proportional. Just realizing
this proportionality is the key to solving some problems.
Chapter
Chapter 7 Problems
Section A-C
Use the following information for questions 1-3:
A student nails a meter stick to a board at the meter
stick's 0.0-m mark. A force of 10 N acts at the 0.5-m mark
perpendicular to the meter stick as shown in the figure.
Force of 5 N acts at the end of the meter stick, making a
30' angle, as shown. Force ? of 20 N acts at the same point
providing tension but no shear. (Use counterclockwise to be
positive.)
4.
I
2.
Section
3.
7.
The
104
Chdpter
bl
bone
elbow
muscle
bone
A.
-1 mg , to the left
B.
d
mg ,to the left
1
mg, to the right
D.
d
mg, to the right
1
hand
A.
B.
C.
D.
3Nm
4Nm
6Nm
8Nm
d
1
A.
A.
B.
C.
D.
1
mg,to the left
d
B.
d
mg, to the left
1
C.
3Nm
4Nm
6Nm
8Nm
105
22. If the torque due to the tension Tin the wire BC about
point A is 2, what is the ratio 2: T ?
(r,the follo,,,ing
A.
B.
C.
0.3333
0.5
2
Chapter
Section E
Use the following information for questions 29-33:
Consider a bone B of given size and shape (length 1,
cross-sectional area A, see figure). A force Facts to compress the bone, causing the bone to shorten by some tiny
length dl.
.. .
31. Bone D is similar to bone B, except that the circumference is 4 times that of bone B. If the same force F is
applied to it, by how much would the bone shorten?
A. 0.062541
B. 0.2541
C. 0.541
D. 4.041
32. How does the stress on bone D (see previous question)
compare with the stress on bone B?
A. It is one sixteenth as large.
B. It is one quarter as large.
C. It is half as large.
D. It is four times as large.
All9
B.
C.
D.
6113
dl
341
The
MCAT Physics
Book
Passage
2.
Chapter 7
... Torques
4.
5.
A.
B.
W'J100
Wm,/10
c. wm,
STOP
Momentum
A.
The 1970s saw the popularity of a certain physics toy, a toy in which five steel
balls hang in a line from threads (Figure 8-1, called Newton's Cradle). If you swing the
left ball to the left and let go, it swings down and hits the other four balls. Then the
right ball swings up and rises to almost the height of the initial release.
Now if you think about it, this is
totally amazing. There are five balls
undergoing collisions, and each collision
involves a rapidly changing force between
two balls (Figure 8-2). You might think we
would need a supercomputer to deal with
Desk toy or physics machine?
the problem. The very simple outcome
(one ball swinging to the right) hints that
Figure 8-1
there is some very simple underlying
physics as well.
before:
The idea that emerged from
experiments like this one was that moving
click click
objects contained a certain amount of
during:
"motion quality" (or "movage"), which is
dick c lck
a constant in any situation. In Figure 8-2
the movage begins in the left ball,
after:
CXXX)
transfers through the three balls, and ends
up in the right ball.
Figure 8-2
Can we write a formula for this
"movage"? Well, it is clear that a moving
Mack truck has more movage than a Tonka truck, so mass must be involved. And a
sprinter has more of it than a walker, so velocity must be involved. But is it
movage = mv
0
'
,
no
II
or
movage = mv2
'
or what? It turns out there are two kinds of movage, and we will study them in this
chapter and in Chapter 9.
iii
--
p'=
(1)
mc.
+p', +--.,
where p",,
..., are the momenta of the objects.
$tot=$,
(2)
z,,
'T
B.
Conservation of Momentum
Ptefore = Prtcer.
(3)
10-m
before:
r [ G
20-m
S
FMP
dU"ng:
after:
Momentum is conserved in collisions.
F i r e 8-3
Chapter 8
(.okgl(
") +
10S
= (10,000 kg)v, ,
Notice we used v,= -20 .ds, with the negative sign since it was going south and we
chose north to be positive. We have to pay attention to signs because momentum is a
vector quantity.
. Example 2: During the collision in Section A, the external forces are the tensions
in the threads and gravity, and these are balanced. The internal forces are all the
complicated forces among the balls. So momentum is conserved. We do not yet know
enough to show why exactly one ball jumps off the right end. Momentum would be
conserved, for example, if all five balls headed to the right at one fifth the impact
velocity of the left ball.
Figure 8-4
Figure 8-5
Figure 8-6
1x10'"
*
,
.,---- -PIO?
S
C
3 x 10'
k
e
S
Figure 8-8
C.
before:
after:
Figure 8-7
FM =mi;
(second law)
(definition of acceleration)
=-&
At'
Chapter
The change in momentum.& is called the impulse. So the impulse and the net force on
a system are related by
Ap = F",,At,
&
or in one dimension
T
Ap = F,,,,At
(5)
Whenever you see a problem involving force and time, you should think about momentum and write this equation.
The
Chapter 8 Problems
5.
7.
Assuming A and B collide first and C is still independent, what is the total momentum of the system just
after A and B collide?
A. -l.lkgm/s
B. 0.1 kg m/s
C. 0.9 kg m/s
D. There is not enough information to answer this
question.
ChaDter
I
12. What is the recoil velocity of the rifle, that is, the
velocity of the rifle just after firing?
A. 0.75 m/s
B.
1.5mls
C.
120 mls
butt
19. A ball (0.3 kg) hits a wall and rebounds. Initially the
ball is going 7 mls, but after the rebound it is going
5 mls. What is the impulse imparted to the ball by the
wall?
A. 0.6 kg mls
B.
1.8 kg mls
C. 2 . 1 k g d s
D. 3.6 kg m/s
A.
B.
C.
D.
One: gravity.
Two: gravity and tension.
Three: gravity, tension, and a normal force.
Three: gravity, tension, and a force to the right.
Chapter 8
24. An apple drops from a tree. Which statement is true
dilring the fall?
The momentum of the apple is conserved.
A.
B. The momentum of the apple and the Earth is
conserved.
C. Momentum is not conserved whenever gravity is
not balanced by another force.
D. The impulse received by the apple is zero.
C.
D.
B.
C.
D.
2.
Momentum
5.
. ............ ...............
STOP
A. Introduction
To understand politics, it is said, you need to follow the money. To understand
physics, it is said, you need to follow the energy. If we understand where energy is
coming from, how it flows, and where it ends up in any physical situation, then we
understand a lot about the physics of the situation.
"What is energy?'you may ask. Actually energy is harder to explain than you
may think. In the history of physics, the concept of energy did not suddenly arrive as a
mature concept out of Newtonian theory, like Athena emerging from Zeus's head.
Rather, it began as a hazy idea which grew in richness and clarity during the 1800s. In
this chapter we can do no better, so we will introduce the concept slowly.
In standard English, energy is defined as the capacity for performing something
useful. We can obtain energy from various places and then we can use it usefully or
squander it, and once it is gone, it is gone.
In physics also, energy is the capacity for doing useful things. On the other hand,
energy is a thing which cannot be created from nothing or destroyed, only transformed
from one form to another. Following energy through its forms is what much of physics
is all about.
As you can see, the physics understanding is distinct from the popular understanding, and we will want to pay attention to the differences between the two.
it. When a force (which can be due to a pair of hands, a rope, gravity, or anything) acts
on an object which moves a distance &, the work done by the force on the object is
where is the angle between the direction of the force and the direction of the
displacement &.The units for work are [ kg m2/s2= Nm = Joule = Jl.
?he total work done on an object is
where F, is the magnitude of the net force and # is the angle between pwand
z.
The
(F,), = 0.
From Figure 9-2 we obtain
F, = mg sin 8.
We can find AK by trigonometry. If we
look at the large triangle in Figure 9-2,
then we have
Figure 9-1
TN'
h
sin8 = --,
&
&sin 8 = h,
&=- h
sin 0 '
The vectors Fwand 2 point in the
same direction, so cos @ = 1. Thus the
work done by the woman on the cart is
h
sin 0 '
Figure 9-2
The energy a woman requires to push a cart from one point to a point which is
height h higher is mgh, even if the path is complicated. See Figure 9-3.
Figure 9-3
b. What is the total work done? Well, since the speed and direction are constant,
the acceleration is zero, and F,,, = 0,so that
w,, = 0.
What? The poor woman works from dawn till dusk, and the total work done is
zero? Where did the energy go? Well, if we were to figure out the work done by gravity,
we would find that it comes to -mgh, so the work done by gravity cancels the work
done by the woman. It seems like a sad story, perhaps, but this will not be the final
word on gravity (see Section D).
- - __------__---.-.
a&
r' ,:
,
,
Solution: F i t , we DRAW A
DIAGRAM (Figure 9-4). Once we draw
the diagram, the answer is clear. The
vector FIn,is perpendicular to the
Figure 9-4
The
C. Energy of Motion
So how much energy do we put into an object if we push it for a while? Let's try
another simple example, this time in one dimension.
=-v,At.
2
W=FAx
Wow! This is just the kind of expression we saw near the beginning of the
Chapter 8. We have completed the circle. But now we have to find out how to use this
expression.
Work and change in kinetic energy are related by the following expression.
W,
=A&
(4)
2
A
Example 1: What is the change in kinetic energy for the woman's cart in the
previous section?
Solution: We calculated W,= 0, which tells us that the change in kinetic energy
is zero. The cart is going the same speed at the end of the problem as at the beginning.
Thus we have dE, = 0. So this result is consistent with the above equation.
Example 2: What is the change in kinetic energy of the Earth in one day?
Solution: According to the previous section, W,o,=0, and indeed the Earth's
speed is constant from one day to the next. The kinetic energy change is zero. Here we
are assuming a circular orbit, which is almost correct.
Example 3: A bullet of mass 20 grams is fired from a gun, so that its speed is
700 mis in air. The bullet enters a tree stump and embeds 2 meters inside. What is the
average force exerted by the stump on the bullet? (Ignore gravity.)
Solution: First, we DRAW A DIAGRAM (Figure 9-5). At first this problem
looks like a momentum conservation problem, because of the collision and crunching
of wood. If we try to apply conservation of momentum, however, we just do not get
anywhere.
The key is to notice that force and
distance are both mentioned in the
problem. Immediately we think of energy.
We know the change in kinetic energy of
the bullet
AEK= Em - EKL
):
= 0J - i(.020
2
kg)(700
= -4.9 x lo3J.
This is also W,,,and we can set W, to
Fdxcos#, where cos# is -1. Thus
The
Y o , = *E,
(the same formula as before).
i
Remember the woman in Section B? She pushed a cart to a new height h, doing
work W = mgh. It would make sense to define gravitational potential energy as
E, = mgh ,
(5)
where m is the mass of the object in question, g is the acceleration due to gravity, and h
is the height. The height is measured relative to some standard, such as sea level or
street level. It does not matter what the standard is, because we are always interested in
changes in height or changes in potential energy. This formula works for all situations
near the surface of the Earth.
Any force with an associated potential energy is called a potential force or a
conservativeforce. Examples include the forces due to springs and the electrostatic
force (see Chapter 14). The force the woman exerts on the cart and magnetic forces are
not conservative forces.
Example: We are now in a position to FOLLOW THE ENERGY for the woman
and the cart. It is clear that the energy ends up as potential energy, How does the energy
start? Kinetic? No, because the cart is hardly moving both before and after its trip. The
energy starts in her muscles, where it was stored as chemical energy. Thus the flow of
energy is chemical to gravitational potential energy. -
E. Conservation of Energy
Sometimes physicists use strange words to mean normal things (They say
"scalar" when they mean "number", for example.) Sometimes they use normal words to
mean normal things, but they mean it a little differently; for example, force and energy.
Sometimes they use normal words to mean something completely different from the
standard meaning, and this leads to much confusion.
In common parlance, "conservation of energy" means frugal use of energy, a
responsibility of good citizens.
In physics, "conservation of energy" means that energy, by decree of Nature,
cannot be created from nothlng nor destroyed, but it can flow from one form to another
or from one place to another. If we calculate the total energy in a closed system at one
time, the total energy some time later will be the same. Energy is conserved.
Energy Conservation
The energy in a closed system is conserved, that is,
constant in time.
In the table are listed some of the energy forms which may appear on the MCAT.
type of energy
kinetic
potential
gravitational potential
mechanical
chemical
electrical
nuclear
sound
light
heat
description
bulk motion
object's position
object's position in gravity
MCAT word for kinetic + potential
batteries, muscles, etc.
moving electrons
energy in the nucleus, radioactivity, fission reactor
pressure waves
electric, magnetic field waves
random motion of particles
The principle of energy conservation in the previolis box is the Grand Statement,
almost too grand to be useful in most problems. For doing problems it is better if the
number of kinds of energy considered are few, like two: kinetic and potential.
(6)
Use this principle in problems in which gravity does all the work.
127
The
1
O + r n g h = - m ~ , +0.
~
2
Solving for v, yields
Vz
=&K.
Example 2: A pendulum of length 0.7 meters is pulled so that its bob (0.2 kg) is
0.1 meters higher than its resting position. From that position it is let go.
a. What is its kinetic energy at the bottom of the swing?
b. What is its velocity at the bottom of the swing?
c. What is the work done by the string tension during the swing from start to
the bottom?
Solution: a. First, we DRAW A DIAGRAM (Figure 9-7). We check the forces.
The tension is perpendicular to the direction the bob is moving at every moment. This
is true even though the bob is moving in an arc and the tension is changing direction
during the swing. Thus tension does no work. Gravity is a potential force, so
equation (6) applies and we can write
EKI+ EpI = EK2 + Epz*
0 + mgh, = EK2+ 0 ,
E, = mgh,
.-.
Sm)
--iL
0.1 m
Figure 9-7
from
1
2
-mvz2= mgh, ,
v2
=a
m
=1 . 4 ~ .
128
F.
Often we have energy in one form and we want to convert it into another form,
for example, from chemical energy in gasoline to kinetic energy of a car. It may be the
case that energy cannot be destroyed, but it can end up in an inconvenient.form such as
heat. In this case we define the efficiency of energy conversion as follows:
Efficiency =
(7)
i
Example 1: A car (800 kg) goes slowly up a hill from the base to a height of
300 meters. It uses 245 grams of fuel in the form of 2,2,4-trimethylpentane. The
following overall reaction occurs in the car:
kcal
mol
AHRac~o"
= -1 3 10-.
What is the efficiency of the engine? Assume no energy loss due to air resistance. Use
1 kcal = 4184 J.
Solution: First we calculate the energy in desired form, which is the potential
energy:
Ed, = mgh
Figure 9-8
b. Now we need to know the kinetic energy both before and after the collision.
before:
1
EK2= -(2500 kg)
2
= 3.1 x lo4J.
Thus the efficiency is
after:
Figure 9-9
G. Power
Power is the rate at which energy is produced, consumed, or transformed, that is,
Example 1: A car (1000 kg) traveling 55 mph has a forward cross-sectional area
of about 4 m2. What is the power dissipated by air resistance? (Use 1 mph = 0.45 mls.
Recall that the formula for air resistance is F,,= C ~ A Vwhere
~ , p, the density of air, is
1.29 kg/m3and C = 0.2.)
Fengin=
t
Figure 9-10
P = -F,,v = - ( c P ~ v 2 ) v
= -1.6
x lo4W.
The minus sign indicates that the energy is dissipated by the force.
Example 2: The same car is traveling 65 mph. What is the power dissipated by
air resistance?
Solution: P = -2.6 x lo4W.
Pulleys are somewhat tricky, but with some practice, problems including pulleys
become simpler. There are two underlying principles:
1. The tension in a single rope is the same all along the rope,
even if it goes over and under pulleys.
2. If a rope is pulled at a constant rate by a hand, the work done
by the hand on the rope is the same as the work done by the
rope on some load.
Example 1: A person is hanging in air by grabbing the two ends of a rope which
is draped atound a pulley. If the person is 60 kg, what is the tension in the rope?
The
Figure 9-11
w,= w2,
Figure 9-12
Figure 9-13
la
Figure 9-14
T = mg.
So m = 25 kg.
Another way to obtain the above
equation is to DRAW A DIAGRAM
showing the forces on both masses (Figure
9-15).
Figure 9-15
In this chapter we explored the concept of energy. Whenever you read about a
force and a distance through which the force acts, you should think immediately of
work W = Fdrcos 4. This will be the key to answering some of the questions, even if no
numbers are involved.
If a net force acts on an object, the kinetic energy E, = -mv2 of the object is
2
changed according to W,,,,
= 4.
The total work gives the size of the energy flow into
an object. Another form of energy is gravitational potential energy given by E, = mgh.
It is important to keep track of the energy flow because energy is conserved. That is,
energy cannot be created from nothing or destroyed, but it can be transferred from one
form to another.
The rate at which energy is transformed is called power P = MAt.
Book
Chapter 9 Problems
Section
6.
7.
1.
2.
3.
4.
8.
9.
B.
C.
D.
5.
Section
150 Joules
260 Joules
3000 Joules
A.
B.
C.
D.
-3000 Joules
-260 Joules
0 Joules
3000 Joules
16. If the force of the road on the car during the stop is
constant, what is that force?
A.
500 Newtons
B.
3000 Newtons
C. 5000Newtons
D. 8000 Newtons
Section
A.
B.
C.
D.
100 Joules
200 Joules
2 x lo4Joules
There is not enough information to answer this
question.
D and E
C.
7.7 m/s
D.
is given by
AH,
J
mole
= - 2 . 2 2 ~lo6
Section F
Cart A (1 kg) and Cart B (2 kg) run along a frictionless level one-dimensional track. Cart B is initially at rest,
and Cart A is traveling 0.5 rnls toward the right when it
encounters Cart B.After the collision, Cart A is at rest.
21. Which of the following is true concerning the collision?
A.
Momentum is conserved.
B. The collision is an elastic collision.
C. The collision is a completely inelastic collision.
D. Kinetic energy is conserved.
22. What is the final velocity of Cart B?
A. 0.25 m/s
B. 0.35 mls
C. 0.5 m/s
D. 1.0 mls
136
Section
C.
D.
18 meters
180 meters
30. How long would it take to bring the mass to the same
height if the current and the potential difference were
both doubled? (Assume constant efficiency.)
A.
15 seconds
B.
30 seconds
C. 60 seconds
D.
120 seconds
Section H
31. In the pulley system shown, the angle a is the angle the
rope makes with the horizontal.
29. If the motor is run for 60 seconds, how high does the
mass rise?
A.
1.8 meters
B. 5.4 meters
A.
amg
1
B. -mg
3
T, = 2T,
T, = T,
A.
B.
All sections
D.
force meter
1
(Take 10 m/s2 for the acceleration due to gravity.)
A.
B.
C.
D.
0 Newtons
30 Newtons
150 Newtons
300 Newtons
first 3 strides?
A. 2.7 Newtons
B. 32 Newtons
C.
130 Newtons
D.
C.
D.
GO ON TO THE N M PAGE
The
46. How high does the block (and bullet) swing on the
strings before it comes to rest?
A.
B.
C.
D.
6cm
11 cm
22.5 cm
225 cm
1
1
(
The bullet embeds itself in the block of wood.
Immediately after the bullet embeds in the wood, the wood
and bullet are moving 1.5 mls.
The event can be divided into two parts:
1) In a very little time the bullet embeds itself into the
wood. Gravity can be ignored in this part.
2) The wood block with the bullet, which is suspended
by the strings, swings upward by height h.
C.
D.
9 Joules
18 Joules
A.
B.
C.
D.
A.
B.
C.
D.
A.
B.
C.
D.
A.
B.
C.
D.
B.
C.
D.
A rock of mass m, less than M, slides without friction
from the same height h along a slope of the same angle a. It
takes time t, to reach the bottom, at which time it is traveling
a velocity v,.
141
1.
B.
C.
D.
2.
Passage 1
valve
4.
5.
2H2(B)
%(g)
-)
2HzO(g)
mol
'
A.
B.
C.
D.
GO ON TO M E ND(T PAGE
6.
1.
7.
B.
C.
D.
111 only
I and III only
I, I1 and I n
2.
3.
4.
Passage 2
D.
143
3 ~by ,a factor of 9
5.
B.
C.
D.
2.
3.
4.
Passage 3
27C
given by
I= MR~,
where M is the mass of the flywheel and R is the radius.
Ideally the kinetic energy of the car would be transferred to the flywheel as the car comes to a stop. When the
driver wants to go again, the energy would be transferred
back to forward kinetic motion. Unfortunately, the efficiency of the two transfers will be less than 10096, so energy
will be lost to heat. This energy can, of course, be made up
by conventionaI means, such as burning gasoline.
For the following questions, use the notation:
M, is mass of the car
M is mass of the flywheel
R is radius of the flywheel
w = 2zf = angular frequency of the flywheel
(
144
0.
dR
GO ON TO THE NEXT PAGE
5.
.
B.
abv
1.
C.
D.
MgH,
2MgH,
2.
3.
4.
145
The
5.
Passage 5
A.
6.
7.
What force pulls the blood to the rider's feet when the
car is at point F?
A.
The centripetal force.
B.
The force of gravity.
C. The normal force.
D. None of the above.
explosive /
8.
9.
Beginning in the 1500s, gunners began using largegrained explosive in order to decrease the rate of burning.
Faster burning charge creates a large pressure very quickly,
thus creating stress on the cannon and creating a risk of
failure. The slower burning charge ensures that the pressure
behind the ball stays more nearly constant as the ball travels
the length of the barrel.
For the following questions, consider a cannon which
is 2.3 meters long with a bore (hole) of radius 5 centimeters.
Assume the pressure inside the cannon after the explosive
has been set off is constant. While the ball is in the cannon,
the forces due to the gases are so much greater than the
force of gravity that the force of gravity can be ignored.
1.
3.
4.
5.
6.
when it leaves the muzzle of the cannon. What additional piece of information would be sufficient to allow
the calculation of the height to which the cannonball
would travel?
A. The mass of the cannonball.
B. The kinetic energy of the ball upon just leaving
the barrel.
C. The force the cannonball experiences in the
barrel.
D. No more information is needed.
STOP
I Interlude
A. Introduction
"Yes, yes," I hear the gentle reader saying, "this physics is all very nice, but I
want to know how to pass the MCAT." That is what this chapter is about.
In this chapter we will discuss the general strategy for studying for the MCAT,
for reading passages, and for approaching unfamiliar questions. In general, however,
the more physics you understand, the better you will do on the MCAT. The more
physics problems you solve beforehand, the more questions you will answer correctly
on the exam. If you concentrate on accuracy and understanding first, you will gain
speed later on.
When I began to write this minichapter, I worked through a number of published
MCAT practice problems, making notes of the skills and methods which proved useful.
You should do the same thing. As you work through practice problems, ask yourself,
how should I have approached this problem? What clues indicate that I should have
used a given method? What other types of problems can be solved with this method?
How should I approach similar problems?
Some questions on the MCAT are simply a matter of comprehending the material
in the passage. You must find your own trouble spots regarding comprehension. Do you
have trouble finding the main ideas? Practice this skill. Do you miss important details?
Learn to slow down. Do you misread questions? Learn to read them more slowly. Do
you spend too much time reading unnecessary information in the passage? Practice
scanning the passages more quickly. No book can diagnose these problems better than
yoursel'f.
Again, the best way to study is to begin studying. The more passages you have
seen, the more likely it is that you will recognize and understand the information in an
MCAT passage.
B. General Strategy
When you approach an MCAT passage, scan it quickly to note the main ideas.
Underline any sentences or phrases that seem to state a main idea or further an argument. Underline any numbers that are embedded in the passage. When you read the
question section you can always go back and read a portion of the passage more
carefully.
When you read a question which seems puzzling,
Don't panic.
Ask yourself, does this remind me of a problem I have done
The
Example 1: Police in Arizona found a piece of wreckage embedded in a mountain rock near a freeway. Study of the site indicated that the wreckage was not that of an
airplane but of a car.
Apparently a man had connected a Jet Assisted Take Off unit (JATO) to his car. A
JATO unit is a solid fuel rocket normally used for assisting in the take off of heavy
planes on short runways. The man and his Chevy (2000 kg) started accelerating
approximately from rest 5000 m from the crash site, quickly reaching terminal speed of
170 m/s. The car traveled in a straight path more or less, except that it became airborne
for the last 2000 m and hit the cliff face 40 m above the ground, creating a crater 1 m
deep in the rock.
1. What is the approximate force the cliff exerted on the car during the final crash?
(Assume that force of collision is approximately constant.)
A. 3 x 1 0 ~ ~
B. 3 x 1 0 ~ ~
C. 2 x 1 0 6 N
D. 3 x 1 0 ~ ~
2.
If the car traveling 170 m/s had crashed into a stationary 6000-kg truck and stuck,
how fast would the combined vehicles be going just after the collision?
A.
170 m/s
B.
85m/s
C. 57 m/s
D. 42 m/s
:
I
cliff
w,1=
= 0 - 1/2 mv2
=-3 x 1 0 7 ~ .
The displacement of the car during the
crash is 1 m, so the approximate force is
3 x 1 0 7 ~ / 1 m =x3 1o7N.
Figure 1-1
Chdpter
before:
Pbeforc= Pa~tcr*
after:
Figure 1-2
P + - pv2 = const.
2
The wing is constructed so that the air flowing over the top flows faster than the
air flowing under the bottom of the wing. The implication is that the air flowing over
the top has a smaller pressure than the air at the bottom of the wing. The force on a
piece of surface area of the wing is given by
F = PA,
so the cumulative effect is an upward force on the wing, or lift.
As the wings of the airplane slice through the air, some energy is lost to the air in
the form of turbulence and heat. This energy comes from the motion of the airplane, so
that the net effect is a drag due to air resistance. The drag is given (very approximately)
by
Fdng=
CPAV',
where C (= 0.2) is a constant, p (= 1.3 kg/m3) is the density of air, A is the crosssectional area of the airplane as viewed from the front, and v is its velocity relative to
the air.
,I:&
Fdng
,A
I
A
F,,, = 0
Figure 1-3
Even in questions which do not involve numbers, the answer often becomes clear
only when you
Write an equation.
As you read a passage, you should be thinking of the connections among the
quantities mentioned. For instance, we have noted that whenever you read about a force
and a time interval, you should immediately think "momentum" and write the momentum equation Ap = F A .
Example 3: One of the new safety features included in cars is the inflatable
airbag. In the event of a head-on collision, the airbag inflates in milliseconds. The
airbag de8ates.a~the driver or passenger presses against it.
1. Which of the following gives an explanation for the safety provided by an airbag?
A.
B.
C.
The airbag decreases the time of collision, thus decreasing the force.
The kinetic energy of the driver is converted to potential energy of the airbag.
The increased distance of the collision of the driver decreases the force he
experiences.
D.
The decreased distance of the collision of the driver decreases the force he
experiences. .
Chapter
I . ..
. .. . . .
. ... . . . . . . . . . . . .. . . . . . . . Interlude
C. Specific Strategies
In this section we look at a few other hints that work on certain classes of
problems.
Some problems involve a large number of quantities related to each other in
various ways, sometimes specified in the passage and sometimes not. For such passages
it is often valuable to
Draw a variable map.
To draw a variable map, connect related quantities by lines that meet at a vertex.
This strategy is especially good for questions which ask, ''What additional information
is necessary to determine . . . ?" An example will help make this clear.
B.
C.
D.
W
k F p f - l
JAX
JP
nx T v
F i r e 1-6
k
Ax
Figure 1-5
(Figure 1-5). We can extend the variable
map to include the volume and length of
the cylinder, using the ideal gas equation
PV = nRT (Figure 1-6). We can place a
check by Ax and P to indicate that we
know these quantities. According to the
variable map, we also need to know force
in order to determine work.
Figure 1-7
where dl, is the amount of stretching, F is the force applied to both ends, L is the length
of the wire, A is its cross-sectional area, and Y is Young's modulus, a constant for a
given material. The following data were obtained for a wire 1 m long of cross-sectional
area lo4 m2.
Experiment
1
2
m (kg)
0
0.5
fi (mm)
0
0.025
I . .. . . .. . . . .. . .. . . . . . . . . .. . . .... . . Interlude
Chapter
column to the table F = mg. Actually we only need to choose one experiment, so let's
choose Experiment 4. The table looks like the following.
Experiment
(kg)
dL (mm)
0.5
0.025
3
4
0.1
10
0.5
F(N)
100
FL
AY'
For questions which ask for an extrapolation from a chart, it is often helpful to
Pick a convenient point, and exmpolate using the given formula.
Figure 1-9
Experiment
m (kg)
1 (m)
2
. r (m)
dr (m)
1x
I X lo-'
1 x ~ o - 2~
~ lo-'
1 x 1 0 - ~ 3x10-'
1x10-~ 4x10-~
2x
lo-'
6.3 x
lo-'
The
What would the sag be for a 2-rn horizontal circular rod (radius loA2
m) if a 12-kg
.
mass were hung in the middle of it?
A.
1.3x10-~m
8. 8 x 1 0 " m
C.
1.2 x
m
D.
1.6x10-~rn
2.
What would the sag be for a 6-m horizontal circular rod (radius lo-' m) if a 3-kg
mass were hung in the middle of it?
A.
9x104m
B. 2.7 x
rn
C.
1.8x10-~m
D.
8.1x10-~m
Experiment
rn (kg)
v, (rnls)
2
4
ti
II
1,
8 ,
0I
A.
B.
C.
D.
Experiment 1
Experiment 2
Experiment 3
Experiment 4
cannon
Chapter
2.
I . . . . . . . . . . . . . . . .. . . . . . . . . . .. . . . . .
For which experiment does the ball have the greatest kinetic energy at the height of
0.5 m?
A.
Experiment 1
B. Experiment 2
C. Experiment 3
D.
Experiment 4
m (kg)
1
1
2
4
v, (mls)
4
8
6
4
E,, (J)
8
32
36
32
m (kg)
I
1
2
4
vo(m/s)
4
8
6
4
EK,(J)
8
32
36
32
E,(J)
5
5
10
20
EK2(J)
3
27
26
12
Interlude
Chapter 1 O
Fluids
A.
Introduction
In studying mechanics, we have generally looked at one object and the forces on
it. For most people, the difficult part of mechanics is unlearning the misconception that
a moving object needs a force to maintain its motion. (Recall Example 4 in Section
5.D.)
In this chapter we study fluids. A fluid is a large number of interacting particles,
so it is somewhat more complicated. The key concept here is pressure. In any given
problem you should be thinking, "Do I know the pressure everywhere? Can I figure out
the pressure where the crocodile is?'and that sort of thing. Even if pressure is not
mentioned in the problem, often it is the concept which leads you to the answer. For
instance, when a teenager sips a soft drink through a straw, does he pull the'refreshing
liquid into his mouth? As you will soon discover, the answer is no.
For the MCAT, you need to know only a few basic principles of elementary fluid
mechanics. The tricky part is learning how to apply them in diverse situations. One way
to do this is to look at so many examples that any new situation reminds you of
something you have seen or worked before. Hence you should pay close attention to the
problems and solutions at the end of the chapter.
B.
Some definitions
Density is a measure of how packed a substance is. Its symbol is the Greek letter
rho, p, and it is defined by
where m is the mass of a piece of fluid and V is its volume. Specific gravity is the ratio
of the density of something to the density of water:
P
specific gravity = PH~O
'
(This is a misnomer, by the way, since "specific gravity" has nothing to do with
gravity.) The density of almost all biological tissue is approximately the same as the
density of water (remember this):
Pressure is like a push (that is, force), except we also consider the area over
which the push is extended. If you are barefoot and step on the sidewalk, the sidewalk
pushes up on you with about 1000 N. If you step on an upright tack, it pushes up on
you with the same 1000 N. Why then does your face look so different when it's a tack that
you stepped on? In this case, pressure is the important concept. Pressure is defined as
The
where F is a force, and A is the area over which it acts. Consider this definition carefully, so that you do not confuse force and pressure. The point of a tack has a much
smaller area than the bottom of your foot, so the pressure on that point is quite large.
The units for pressure are N/m2, and these have a name:
Pressure can also be measured in pounds per square inch, or psi. The pressure of the
atmosphere at sea level varies, but its average is given by
1 atm= 1.01 x 1 o 5 P a = 14.7 psi.
(6)
C.
Buoyant force
FB = pVgr
(7)
g,
Figure 10-1
and we replace m with pV, where p is the density of the duck, and V is its volume:
0 = P H ~ o V , , ,-~ PVg .
L
Since one third of the duck is shown above the water, the displaced volume is - V , and
3
we write
Example 2: A crown, apparently made of gold, is weighed in air, and the weight
is 50 N. The crown is weighed again by hanging it from a string and submerging it in
water. (See Figure 10-2.) What reading will the force meter give if the crown is true
gold? (Use for the density of gold
19.3 glcm3; and for water, 1.0 &m3.
Also, use g = 10 m/s2.)
Solution: First, we DRAW A
DIAGRAM with all the forces on the
crown. (See Figure 10-3.) We draw the
force of gravity first. Next, what is
touching the crown? The fluid and the
string are, so we know to draw the
buoyant force and the force of tension. It
is the force of tension that the meter
reads. (That is what a force meter does: it
provides a force and then tells you what
Figure 10-2
that force is.)
We have force balance because there is no acceleration:
O = F,+ F , - m g .
In this equation m is the mass of the crown, and mg is 50 N. Let's first solve for F,
(which we want), then replace F, and m. We obtain
F,=mg
-F,
=P A ~ V P~H , O V ~
Figure 10-3
...
Fluids
The
P,= P I + phg
This equation applies not only to this situation but to any situation involving two
vertically separated points in a fluid. The pressure is greater at point 2 because more
fluid is pressing down on top of point 2 than on top of point 1.
We can obtain pressures at other points in the fluid using a principle, discovered
by Blaise Pascal: "Pressure applied to an enclosed fluid is transmitted to every portion
of the fluid and the walls of the containing vessel in all directions." The language is a
bit obscure, but it translates into principles 2 and 3 below.
Example 1: We are standing on the fifty-first story of a hotel, where each story is
4 meters high. How much less is the pressure on the fifty-first story than the pressure at
Chdptcr
1 0 . . . . . . . . . . . . . . . . . .. . .. . . . . .r.. . . . Fluids
= 2400 Pa.
This pressure is fairly small compared to Pa,, but it is enough to make your
ears pop. Notice what is going on here. The people on the ground floor have to deal
with not only the air column on top of us at the fifty-first floor, but also they have the
air column between us and the ground floor sitting on their head.
P = 1 atm
= 1.01 x lo5Pa+2.0x loSPa
= 3.01 x I@ pa.
b and c.
P,.= 3.01 x lo5Pa.
d.
P, = P, + (10' 3 ) ( 1 0 ~ ) ( 5m)
Figure 10-5
The
Pm=PA+pgh,
PA = Pam,- pgh
= 1.01325 x lo5Pa (1.36 x 104)(9.8)(0.38) Pa
= 5.1 x lo4pa.
b. At point B, we obtain
P, = Pa, - pgh
I
I
Figure 10-6
c. At point C, we obtain
PC = Pa, - pgh.
= 1.01325 x lo5Pa (1.36 x 104)(9.8)(0.76) Pa
=OPa.
Thus the pressure vanishes at the top of the column.
This is a simple barometer. Above the mercury column is a vacuum, or, more
accurately, mercury vapor. The last calculation shows that the height of the mercury
column is proportional to the outside pressure. For this reason, the height of a hypothetical mercury column is often given as units of pressure:
1 ton = 1 mm of mercury = pressure sufficient to lift Hg 1 mm ,
(9)
760 tom = 1 atm.
These are the units used in a sphygmomanometer. But the numbers reported in
blood pressure measurements are the pressures in excess of atmospheric pressure,
called the gauge pressure. For instance, the systolic pressure of a woman with blood
pressure 110160 is actually (760 +110) torr = 870 tom (assuming 760 torr atmospheric
pressure). Thus:
E. Surface tension
The molecules in the middle of a fluid exert an attractive force on each other,
called cohesion. This is what holds the fluid together. The molecules at the surface of
the fluid, however, experience a cohesive force directed into the fluid. If the surface
becomes bent for some reason, there is a restoring force making the surface smooth or
flat. The larger the distortion, the larger the force, up to a maximum:
(1 1)
f',,
= YL,
where y is the surface tension (a function of the fluid) and L is the length of the edge of
the object in contact with the fluid. This will become c l e a r with some examples.
Chdpter 10
. ...
Figure 10-7
7
--.-..
G,---.---.-..--:.y
,:-...
.... .-.-
Fluids
The
F. Continuity
The dynamics of flowing fluids is more difficult than statics, but this is good
news. It means the MCAT will test ~ n l ysome of the basic principles.
The first principle is continuity. Figure 10-10 shows a river, in which point 1 is in
a slow, lazy flow, while point 2 is in the
rapids. Why does the water flow faster at
point 2? During one second the number
of gallons flowing past point 1 is the same
as the number flowing past point 2.
(Think about it. If the water level between
1 and 2 is constant, then the inflow must
equal the outflow.) The difference is that
point 2 has a smaller cross-sectional area.
The rate at which a fluid passes by a
point, measured in volume per time, is
called theflow rate$ In a steady state, the
flow rate is the same at each point along
Figure 10-10
the flow:
Now we can relate the flow rate to the actual speed of the fluid. Think of water
going through a garden hose. If we turn up the faucet, then the flow rate and the flow
velocity both increase. We can also obtain a greater flow rate by increasing the crosssectional area. We would guess (correctly) that we could write
f = Av.
(13)
The discussion in this section refers to any incompressible fluid, that is, liquids. It
also applies to compressible fluids, like air, if the fluids are not, in fact, compressed.
where q is the viscosity in kglm s (that is, kilograms per meter per second, not kilograms per millisecond). You should understand this equation but need not memorize it.
Just remember that viscosity is stickiness.
Viscosity often calms down a flow. Consider a stream of water flowing past a
rock or the flow of air past a weather vane. The flow can break off into wild swirls and
chaotic patterns, called turbulence. Smooth flow is called streamline or laminar. If we
pull a spoon through a bowl of molasses, the molasses is mostly undisturbed (laminar
flow around the spoon), whereas if we pull the spoon through a cup of tea with cream at
the bottom, the tea and cream undergo turbulent motion. What's the difference? The
more viscosity, the less turbulence.
An important parameter for determining the type of flow is the Reynolds number:
where 1 is the size (in m) of the obstacle in the flow (spoon or whatever), p is the
density of the fluid (in kg/m3), v is the velocity of the flow (in mls), and q is viscosity
(in kg/m s).
The flow starts getting rough when Re is around 40, and-it is usually turbulent if
Re is greater than 20.000. Do not memorize this equation. but do realize that the
presence of r7 in the denominator means that higher viscosity reduces turbulence. It
should also make sense to you that v is in the numerator.
Example: What is the Reynolds number for a spoon moving through tea with
kg/m s and estimate other values.)
cream in it? (Use q = 1.0 x
Solution: We can use information corresponding to water, so the density is
kg/m s. A good rate to stir tea is 0.1 m/s, and a
lo3 kg/m3, and the viscosity is 1.0 x
typical spoon has size 0.03 m. Thus
H. Bernoulli's Principle
For laminar flow there is an
equation which relates conditions at one
point in the flow with points downstream.
Consider two points, 1 and 2, along a
streamline. Point 1 is at some height h,
above a standard height, and point 2 is at
height h,. (SeeFigure 10-11.)
Figure 10-11
1
1 2
~ + P ~ ~ , + ~ P Y ~ ~ = ~ + P P ~ + (16)
~ P Y ~
1
Let's see if we can make sense of this equation, which can also be written
1
(17)
1
PAV + mgh + - mv2 = const.
2
Wait! This looks like an energy conservation equation, the second and third terms
being potential and kinetic energy. But what is the first term? This is a bit more
difficult, but we can make this look like an expression for work. For a moving fluid, AV
can be replaced with AAx so that PAV = PAAx = FAx. Thus, PAV is the work that one
portion of the fluid does on another portion of the fluid as it moves along.
Bernoulli's principle is an expression of energy conservation, and that is why
there are so many caveats in the statement of the principle: we are trying to make sure
energy does not leak out into heat and ruin the equation.
Example: A large barrel of water has a hole near the bottom. The barrel is filled
to a height of 4.5 meters above'the bottom of the barrel, and the hole is a circle of
radius 1 centimeter in the side of the barrel at a height 0.5 meters above the bottom.
a. What is the flow velocity v just outsidz the hole?
b. What is the flow rate f out of the hole?
Solution: a. First, we DRAW A DIAGRAM with a streamline. (See Figure
10-12). Bernoulli's principle applies, so we have
We are looking for v,. At point 1, the pressure is atmospheric pressure Pa,, and at point
3, we have P, = Pa, as well. Also, h, = 4.5 m and h, = 0.5 m.
The tricky part is realizing that v, is very, very small. This is because continuity
guarantees that Alvl = A,v,, where A, is the cross-sectional area of the barrel and A, is
the area of the hole. Thus we can set v, to zero in the above equation:
Figure 10-12
(10?)(4.~rn) = (10~)(0.5m)+-v,',
2
m2
s2
vj2 =80-,
vj =9-.
Another way to get the same result is to realize that the pressure at point 2 must
be (from Section D)
P, = P, + pgh = P, + (10'
3 1 1 0 :)(4
m)
Chapter
Then we can use Bernoulli's principle between points 2 and 3 and use h2 = h3 to obtain
b. The answer to part b we get through the definition of flow rate, so we have
f3
= A3v3
conservation of energy along the fluid flow, so we have the sum P + -pv2 + pgh being
2
a constant along a streamline, as long as energy is not lost to heat or other energy sinks.
These two principles allow you to solve most simple problems involving flowing fluids.
Chapter 10 Problems
Section
5.
A.
B.
C.
D.
0.08 N
0.12N
0.8 N
8N
Section C
A.
B.
C.
D.
0.25
0.5
0.75
2.0
**
(0.0821)(27)(1WO)
Chdpter
7.
P H ~ Pair
10. Why is it difficult for the man to breathe?
A.
His lungs are expanding against a net
lo4Pa of pressure.
B. His lungs are expanding against a net
1.01 x 1 6 Pa of pressure.
C. His lungs are expanding against a net
1.1 x lo5Pa of pressure.
His
lungs are expanding against a net
D.
2.02 x 10' Pa of pressure.
Section D
GO ON TO M E NEXT PAGE
171
14. If the force exerted by the piston is F,, what load can be
lifted?
15. If the work done by the piston on the fluid is W,, and
the work done by the fluid on the load is W,, which is
correct?
A.
W, is less than W,.
B.
W, is equal to W,.
C . W, is greater than W,.
D. There is not enough information to determine a
relationship between Wl and W,.
13. If the piston and the load are at the same height, and the
pressure of the fluid near the piston is P,, what is the
pressure of the fluid near the load?
A.
P2
17. The figure shows a simple barometer, which is essentially a U-tube with one end leading to a reservoir
whose pressure is desired and the other end open to the
atmosphere.
A.
B.
C.
D.
Section F
P,, r2v&
B.
P,, z r Z v A t
C.
Pm,2nrzvAt
D.
P,, nrlvAt
Section G
26. Which expression gives the torque which the rod must
exert?
2zrlqvlAr
Chapter 10
Fluids
Section H
I
I
Pressure
D.
L-P,""
A,,
Pg
4,e2
+L
Pool
28. Which gives the best expression for the velocity of the
GO ON TO M E NEXT PAGE
The
35. Which gives the best expression for P,, the pressure in
the narrow tube?
D.
P,,
+ pg(h + h,)
Pressure
valve
Chdptsr
All
C.
sections
A.
B.
C.
D.
0.44N
0.49N
0.54N
There is not enough information to answer the
question.
A. 0.20N
B. 1.18 N
C. 2.16N
D. There is not enough information to answer the
question.
Use the following information to answer questions 38-39:
The restaurant Bistro-an-Maine-Street is located on
the fourth floor of a certain building. Part of the restaurant's
"character" is that the beverages are served on the first floor
with a straw leading up to the fourth-floor customers
12 meters up. The joke is that customers cannot manage to
drink their beverages. One night a chemist brings along a
vacuum pump from her laboratory. Assume the pump can
draw a perfect vacuum.
Assume also that surface tension of the beverage plays
no role, and we have
P,, = 10' Pa,
p= 1d kg/m3,
D.
10 ..... . . . . . . . . . ..... ..
. . . ... . . ..
A.
B.
C.
D.
210N
420N
2100N
2.1~10~~
g = 10 rn/s2,
plir= 1.2kg/m3,
Fluids
m.
D.
42. What would be the flow through the hole if the little
Dutch boy removed his finger?
A.
4.5 x lo4 m3/s
B. 4.5 x lo2m3/s
C. 4.5 x lo4m3/s
D. There is not enough information to answer this
question.
43. What force does the little Dutch boy have to exert in
order to keep his finger in the dike?
A.
1N
B.
102N
C. 1 0 4 ~
D. There is not enough information to answer this
question.
A.
B.
C.
D.
49 N
50N
51N
There is not enough information to answer this
question.
Passage 1
Chdpter
pgh.
Pressure at P , and at P2 are both Pa,.
Pressure at P , and at P, are the same but not Pa,.
Pressure at P2 is greater than at P , by the term
pgh-
B.
C.
D.
2.
Passage 2
F = lcr2(p,, - pgh)
B.
F = 2 z r 2 ( p a , - pgh)
C.
F = zr2p,,
F = 2 z r 2 Pa,
D.
4.
5.
Model 1
pressure less
than atmoshpere
Fgrav= r2hg
B.
FBrrv= 2r2hg
C.
F,, = lcr2hpg
D.
F,,, = 21crzhpg
6.
2ry
'v
BC.
2my
D.
2zr2y/h
A.
B.
C
D.
inner
chamber
< hydroquinone
hydrogen peroxide
- 45.68
- 44.65
- 44184
- 68.32
Passage 3
outer
chamber
chamber
1.
2.
A.
(pchm - Pam )A
B.
(PC,,
+ Pam )A
4.
A.
P,
B.
n, RT
B.
C.
D.
I
A recent innovation in automobile safety is the airbag.
In the event of a collision, an airbag in front of the driver
inflates in about 0.02 seconds. As the driver's body comes to
a rapid stop, the airbag provides a soft cushion, softer than
the steering wheel in any case.
The essential mechanism of the airbag consists of a
sealed combustion chamber containing iron(II1) oxide and
sodium azide, which react to form the nitrogen gas which
fills the airbag:
C.
D.
nRT
v,m
molecules in the chamber.
nRT
<tm
+-
v*,
(2)
The
gas chamber
igniter
Lb
r
chamber
I\
3.
4.
5.
rupture
'.Db--#Y
filter
screen
The nitrogen gas fills the bag in about 20 ms, and the
whole reaction (I) continues for about 50 ms. The bag is
porous, so as the driver presses against it, the bag deflates.
Thus, it is almostly completely deflated by the time the
accident is over. After that, it is hoped that the driver can
walk away from the collision with no injuries worse than
abrasions from the inflating bag.
A.
B.
C.
D.
A.
(1)
B.
(2)
c.
(3)
All three have zero entropy increase.
B.
C.
D.
D.
reservoir
at pressure pmS
h2
2.
- -
s&&=---
Barometer 1
Barometer 2
P,?
A.
B.
C.
D.
The
STOP
I Chapter 1 1
A. Introduction
This chapter begins the study of waves. Waves govern many of the phenomena
we experience every day, such as sound and light. In this chapter we look at what these
various wave phenomena have in common, their so-called "wavelike" nature. In the
following two chapters we will look at sound and light separately.
For these chapters it will be important to have pen and paper ready to recopy
diagrams and rework problems. But it is especially important to have ready a mental
pad of paper. You need to visualize the to-and-fro motion of the medium, the shape of
waves, and the bending and reflecting of waves. Only in this way will the few formulas
and ideas become intuitive.
If you stretch a spring, the spring exerts a pull on you. The more you stretch it,
the more it pulls. If you compress a spring capable of being compiessed, then it exerts a
push. Hooke's law states that the force is proportional to the extension or compression.
t
Hooke's Law
If a spring has resting length 4, and it is stretched (or
compressed) to a length 1, + x, then it exerts a force
Fvring
= kl:
(1)
Let's be clear about the forces involved here. Figure 11-2 shows a spring attached
to a wall stretched by a hand. Figure 11-3 shows all the forces involved. The force p, is
the force the spring ejrerts on the wall, and is the force the wall exerts on the spring.
These forces are equal in magnitude because of the third law of motion. The force F3is
the force the hand exerts on the spring, and is the force the spring exerts on the hand.
The forces F3and p4are equal in magnitude because of the third law of motion as well.
The forces p2and F3 add to zero because of the second law of motion, because the
spring is not accelerating, giving force balance. All these forces are equal in magnitude
(for various reasons), but there are four forces.
F2
F4
Figure 11-2
Figure 11-1
Figure 11-3
This energy is the second type of potential energy we have encountered. In many
problems, we can treat this potential energy like gravitational potential energy.
EK + Ep = constant.
(3)
Chqter
11
. . . . . . Periodic
L;d
w,, = AE, = -4
=4
-4
Figure 11-4
This is one of those problems in which blind plugging into the formulas is to no
avail, but thinking about the energy flow is the key to success.
C.
Let's think about a mass rn connected to a horizontal spring (resting length I,,
spring constant k ) which is connected to a wall. The mass is sitting on a frictionless
floor (so we can ignore vertical forces). We stretch the spring to length 1, + A and let go.
What happens?
At first the spring pulls the mass back towards equilibrium. As the mass goes
faster, the displacement x and force F,,,, decrease. When the spring is length b, the
Because the length is I, (and thus x = O), the restoring
mass is moving velocity -v,.
force and acceleration are zero. 'lhe maximum displacement occurs when v = 0 and the
magnitude of the spring force is very large. Figure 11-5 shows the movement at five
times. Spend some time making sure you understand every entry on the table. In the
first line the negative sign of the acceleration indicates the acceleration vector points
left, the same as the force of the spring. This motion is called simple harmonic motion.
Figure 11-5
E, = E,
+ E,
=-&
(= constant)
+_my2.
The frequency is related to the spring constant and the mass as follows:
Let's see if this equation makes sense. We would guess that a system with a ~tiff~spring
would have a high frequency (think about it), so it makes sense that k should be in the
numerator. As k increases, f increases. Also a larger mass will decrease the frequency,
so it makes sense that rn is in the denominator. The square root is needed to make the
units agree.
Let's consider a similar set up. In a
pendulum we have a bob connected to a
string or a light rod, which is connected
to a ceiling of some sort. In this case, the
restoring force is provided not by a spring
but by a component of gravity. In Figure
. -A mgsine
11-6, the solid mows are the two forces
mgcose
on the bob, and the dashed arrows show
mg
the gravitational force divided into
components, along the supporting rod and
A pendulum operates by a principle
perpendicular to it. The restoring force is
similar to t h t of a spring with a mass.
the latter
:"
Figure 11-6
where sin0 is approximately equal to 0
(measured in radians) for small angles. Note the similarity of this equation to equation
(1). In both, the force is proportional to the displacement, so the motion is similar.
The frequency of a pendulum is given by
Note that the mass of the bob m does not appear in this equation, so a 10-kg mass
swings on a 3-m string with the same period as a 0.1-kg mass swinging on a 3-m string.
You need not memorize this equation, but you should be familiar with the fact that m
does not appear in it.
Chdpter
I
I
Figure 11-8 shows two pendulums with a very weak spring connecting them.
still
still
still
These are two pendula weakly connected by a spring. The energy in the
first pendulum is slowly transferred to the second then slowly back again.
Figure 11-8
Another example of resonance involves a soprano, a wine glass, and the air
between them. The soprano can tap the glass to hear its natural frequency. If she sings
that note very loudly, then the energy starts in one oscillator, her vocal cords, and
transfers to the other oscillator, the wine glass, by the weak coupling, the air. In this
case, enough energy enters the wine glass to cause it to go into a nonlinear regime, and
it bursts. (See Section 7.E.)
E. Waves,
an Introduction
rn
rn
Figure 11-9
-+ t
--+ t
Figure 11-11
,CAT
Chapter
II
up and down
Figure 11-12
wave
longitudinaVtransverse
medium
water wave
both
water
T
string
wave on plucked string
sound
L
air
earthquake
both
earth
light
T
electric, magnetic fields
F.
Interference
When two particles come together, like two balls or two cars, they generally
collide in some manner. When two waves come together, they do not collide but jumble
together in a process called interference. It is not as complicated as it sounds. For
example, consider a wave on a lake (amplitude 7 cm, wavelength 4 m) aniving from the
north, such that at point A at time 12:30 (exactly) its height would be +5 crn if it were
the only wave around (Figure 11-13).
Now another wave (amplitude 10 cm,
wavelength 6 m) arrives from the east,
1such that at time 12:30 its height would
be -2 cm if it were the only wave around.
The resulting height of the water at 12:30
at point A is (+5 + -2) cm = +3 cm.
(These heights are measured relative to
A
the equilibrium height of the water.)
2
Three seconds later, let's say, wave 1
When two waves come
would have height +7 cm and wave 2,
together; they interfere.
-1 cm. Then the new height, with both
waves, would be (+2 + -1) cm = +1 cm.
Figure 11-13
!-I
II
Principle of Superposition
When two waves come together, the resulting displacement of
the medium is the sum of the individual displacements.
Figure 11-14 shows another example of this, in which two wave pulses come
together, one from the right and one from the left.
Figure 11-15 shows a third example
of interference on water. In this example a
wave train from the left encounters a
wave train from the right with the same
wavelength. Water skaters are sitting on
A
the water at points A, B, and C.
---
Figure 11-15
Figure 11-14
6h
192
Chapter
11
. . . . . . Periodic
M o t i o n and Waves
Sound from
speaker 1
A f
V r
Sound from
speaker 2
Alice hears sound from
speakers 1 and 2.
Figure 11-18
Bob
Figure 11-17
were making a sound, the wave arriving at
her ear would look like that shown in the
same figure. Note that these figures show
displacement of air particles versus time.
Figure 11- 19 shows what Alice hears with
Sound wave
heard by
Alice
Alice
Bob
Figure 11-20
Figure 11-21
the two speakers. Figure 11-22 shows
what Bob hears, nothing.
Sound wave
heard by
Bob.
For Bob the sound waves
add destructively.
Figure 11-22
I.
be
A mode of motion in which every part of the medium moves back and forth at the
same frequency is called a normal mode. The example of the guitar string will help
make this clear.
Example 1: A guitar string has a wave velocity 285 mls, and it is 0.65 m long.
What is the lowest frequency which can be played on it? (The lowest frequency
corresponds to the normal mode with no nodes except at the ends.)
Chapter
Solution: Figure 11-23 shows the normal mode for the lowest frequency. A full
wavelength looks like,-
write
Example 2: What is the second lowest frequency that can be excited on the same
guitar string?
Solution: Figure 11-24 shows the
normal mode with one node between the
ends. That is, the midpoint of this string
-------- ------- experiences destructive interference and
thus no motion at all. For this mode the
wavelength is A = 0.65 m. We write
Next lowest frequency mode
v
for a string held a t both ends.
f=-
= 440Hz.
rn
Figure 11-24
This frequency is called the second hannonic (the first harmonic being the fundarnental, but that is always called thefundamenraf).
Example 3: What is the third lowest frequency that can be excited on the guitar
string?
Solution: Figure 11-25 shows the normal mode with two nodes between the
ends. The wavelength is given by
F i r e 11-25
= 660Hz.
The
Chapter
Chapter 1 1 Problems
Section
. ... .
3.
4.
5.
11
bob?
A.
B-
7.
t
1
c.
D.
A.
B.
C.
D.
1
\
The
Section
--F
1
constant by f = 2n
m .)
198
Chdpter
11
Immediately after the hit, the two blocks are going 0.2 mls.
They then oscillate. Consider the following three statements in questions 19 and 20.
I
15. What is the amplitude of the oscillation?
A. 0.15m
B. 0.3 m
C. 0.6 m
D. 3 m
I. Momentum is conserved.
11. Kinetic energy is conserved.
In. The sum of kinetic and spring potential energy is
conserved.
D.
5.0n-h
22. How far does the spring get compressed from its
resting position?
A. 0.02 m
B. 0.04 m
C. 0.08 m
D. 0.16m
'
199
Book
Section D
25. A research and development lab has just built a prototype for a potato peeler. It has many moving parts, but
one particular part, the blade patroler, is not supposed to
move. When the researchers turn on the machine, they .
notice that this piece vibrates a little at first, then more,
until it is flopping around uselessly. If resonance is
responsible for this phenomenon, what can you
conclude? Consider the following statements.
I. There is a weak connection between this vibration
and another vibration.
11. There is a strong connection between this vibration
and another vibration.
111. The two vibrations have similar or equal frequencies.
A.
B.
C.
D.
I1 only.
I and 111.
IIand III.
I or I1 is true, and 111 is true.
Section E
A.
B.
C.
D.
2ds
3.14m/s
4ds
6.28 m/s
2000 N
2000 N
--
200
Chapter
1 1 . . .. .
speaker 2
Section F
Lo
detector
36. What kind of point exists exactly midway between the
two speakers?
A. An antinode.
B. A node.
C. Neither an antinode nor a node.
D. Both an antinode and a node.
eoi
GO ON TO THE NEXTPAGE
38. What kind of point exists exactly 0.4 m to the left of the
right-hand speaker?
A. An antinode.
B. A node.
C. Neither an antinode nor a node.
D. Both an antinode and a node.
D.
B.
C.
D.
2m
Passage 1
I
2
4
I
GO ON TO THE NEXT PAGE
Chdpter
m,
where T is
The wave velocity is given by v =
the tension in the string, and p is the linear mass density,
which is the product of material density and cross-sectional
area.
For the following questions, consider an E string
(frequency 660 Hz) which is made of steel. It has a mass of
0.66grams for each meter of wire and has a circular cross
section of diameter 0.33 mm. The string length when strung
on a guitar is 0.65 m.
Also note that the D string has a wave velocity of
382 m/s.
5.
11
.. . . .
6. If we want to increase the frequency of the fundamental of a string by 3%. by how much do we want to
change the tension in the string?
A.
increase it by 1.5%
B.
increase it by 3%
C. increase it by 4.5%
D. increase it by 6%
-."
1.
2.
4.
Passage 2
Book
3.
In addition to being reflected and transmitted, sometimes wave energy is absorbed. Certain media convert wave
energy into heat energy. This can happen, for instance, for a
wave traveling along a rope. The rope fibers rub against
each other and the energy dissipates as heat (see figure).
A.
B.
C.
D.
4.
typical wavelength
10-100 m
0.1 m
1m
lo-' m
204
Chapter I 1
Passage
. . . ..
4.
5.
STOP
Chapter IP
Sound
A. Introduction
Sound is a longitudinal wave in some material medium, usually in air. Alternatively, we can say that sound is a wave of pressure variation, as Figure 12-1 shows.
air in
.-.*..**.
*
..
.ye.
2: .
;
,. *..:.,..
4-
airwitha
:**sound wave
:*,.t*:*:.
4-
ZWS..
- 0
**.:*.
* * .
+ +-
*..-#ye
*.*s*<S,*
.. .
.-.,ye ...
**:
.=
*.*.
**.
+
*.
*.?
-*-s:
.***'*
F i r e 121
Notice several things about the above graphs. The variations of pressure are much
smaller than the barometric pressure itself, that is, the equilibrium pressure P,. In fact,
the pressure variations are much exaggerated in the figure. The quantity Ax gives the
displacement of an air particle from its equilibrium position to its position with the
sound wave, so that a positive Ax corresponds to displacement to the right. Where Ax =
0, the pressure is a maximum or minimum, and where P = P,, we have Ax a maximum
or minimum. We denote the variation of the pressure from the equilibrium pressure as
AP=P-P,.
There are many ways in which energy can be converted into the energy of sound;
for example, an underground nuclear explosion creates pressure waves in the solid
Earth.The vibrating column of air in the plaintive oboe creates pressure waves in air.
The
MCAT Physics
Book
In general, sound waves travel faster in a stiff material than in a material which is
not as stiff. Thus waves travel a littIe faster in solids than in liquids, and a lot faster in
solids and liquids than in a gas. (See table.)
material
air
water
steel
speed of sound
340 d s
1600 d s
16000 mls
sound
waves
I = - AE
.
AAt
(1)
Chdpter 1 2 . . . . . .. . . . . . . . . . . . . . .. . . . . .
Description
I0
rush of air
wind
conversation
water fall
pain
1
1o - ~
lo4
lo-'
1
1 o3
lo6
1 0'
10'~
0
3
6
9
12
0
30
60
90
120
Example 1: A loud argument takes place in the next room, and you hear 70 dB.
How much energy lands on one ear in one second? (An ear is about 0.05 m by 0.03 m.)
Solution: The intensity is given by
log,,
I
= 7,
I0
1=
W
-
m2 '
.
I
'-B
Figure 12-3
Thus,
AE = IAdt
As you go further from a source of sound, the intensity of the sound decreases. In
order to figure out how much it decreases, think of an alarm clock in the center of two
concentric spheres (Figure 12-3). The alarm clock produces a certain amount of energy
each second. That same amount of energy flows out of sphere A each second, and the
same amount flows out of sphere B each second. Thus we have
[power going through surface A] = [power going through surface B],
because the surface area of the sphere is 41rr-i.Similarly, a man at radius r, experiences
intensity
...
Sound
"
1
In words, the intensity decreases as the square of the radius
(inverse square law).
Memorize the formula, but also understand the reasoning that led to the formula.
Example 2: Jack and Jill are in a field, and Jill is playing a violin. If the intensity
of the sound Jack hears is Po (in decibels) when he is 63.2 m away, how much louder
does she sound when he is 2 m away?
Solution: If Jack moves from 63.2 m to 2 m, then the radius decreases by a factor
of 63.212 = 31.6. Then equation (3) indicates that I increases by a factor of (31..6)' =
1000. Three factors of 10 is equivalent to adding 10 to P three times, so P = Po + 30.
The violin sounds 30 decibels louder.
The pitch you hear depends on the frequency of the sound wave; the lower the
frequency, the lower the note. For instance, the wave in Figure 12-4, with period T =
0.8 ms, corresponds to D#,. The wave in
12-5, with period double the first one,
corresponds to D#,, the same note one octave higher.
Figure 12-5
Figure 12-4
C. Resonating Cavities
In the last chapter we looked at the sound produced by a plucked guitar string or
a struck piano string. Now we will look at resonating pipes, like organ pipes. While the
resonating cavity of a soft drink bottle or of an oboe are more complicated than the
pipes in this section, the principle behind all these pipes is the same. Standing waves
are set up in the cavities, and these produce sound of a particular pitch and timbre.
A closed pipe is a pipe closed at one end and open at the other. If we excite the
air column, the air in the pipe vibrates longitudinally. The variable x gives the location
along the length of the pipe, and Ax gives the tiny displacement an air particle can have.
(See Figure 12-6.) The double arrow shows the air particle moving back and forth.
Since its equilibrium point is in the middle, the distance from one side to the other side
of the displacement is 2A.r.
210
Figure 12-6
Figure 12-7
The
Example 1: A boy blows across the top of a bullet casing (a cylinder closed at
one end, open at the other) which is 0.03 m long. What is the frequency of the note he
hears (the fundamental)? (The speed of sound is 343 mls.)
Sdution: The fundamental mode is shown in Figure 12-7, and the wavelength is
d = 4 (0.03 m) = 0.12 m. Thus f = vld = 2860 Hz. (At a different temperature the sound
speed will be different.)
Some hints for drawing these diagrams appear at the end of Section ll.G. For
closed pipes, each successive harmonic has one additional node.
Now try drawing the second harmonic without looking at Figure 12-8.
For the second harmonic we have drawn three fourths of a wave, so L = 314 A,
and d = 413 L. The next harmonic is shown in Figure 12-9, but try to draw it also
without looking. What is A? (Did you get 415 L?)
Let's go back and look at the fundamental. Note this peculiar fact: If we are
thinking in terms of displacement of air particles, then the node is at the closed end and
the antinode is at the open end. If we are thinking in terms of pressure variation (see the
beginning of the chapter), then the closed end is the antinode and the open end is the
node. Figure 12-10 shows the fundamental in both cases. The frequency we calculate
comes to the same, of course. You should check this point.
Figure 12-8
Figure 12-9
Figure 12-11
AP
Figure 12-10
fundamental
second harmonic
.third harmonic
Figure 12-12
If we consider pressure variation, then both ends are nodes, and we can draw a
mode with no nodes in the middle. Try drawing these graphs yourself without looking.
They represent the fundamental and the second and third harmonics. Draw also the
graphs for the fourth harmonic.
0. Beats
If you play the lowest two notes of a piano you may hear the notes separately, but
you may also hear a beating pattern, like aaaaaaaah-oooooooo-aaaad~ooooooooooaaaaaaaah about twice a second. Try this if a piano is available.
If you have a guitar, you can hear this effect by playing an A on the fifth fret of
the sixth string and an A on the fifth string open. If the two strings are slightly out of
tune, you will hear a single note that gets louder and quieter, louder and quieter. This is
called beats.
E. Doppler Shift
If you have ever been standing
around where a train or car goes by, you
are familiar with the eeeeeeeeeee
aaaaaaaaaah sound it makes as it passes.
Why does this happen?
Figure 12-14-Figure 12-16 show
this phenomenon. Figure 12-14 shows a
train whistle making sound waves when it
is still. The man h&s these pressure
waves, so that his ear records a certain
frequency of pressure-maxima arrival
Fire 1214
The
Figure 12-15
Figure 12-16
where f,is the detected frequency, vsis the speed of the wave in the medium, v, is the
speed of the detector,,v is the speed of the emitter, and f, is the emitted frequency.
The speed of sound is 343 m/s.
a. What frequency would the car detect if it could detect the sonar?
b. What frequency would the officer detect from the reflection?
Solution: a. The frequency that the car would pick up if it could intercept the
police sonar is
Chapter
f,, = (60kHz)
' s
m
343 -
= 66.7 Hz.
We choose the negative sign because, again, we know the result must be a higher
frequency. Thus
= 75 Hz.
In this chapter we looked at sound as an example of waves. We especially noted
resonating cavities of air which exhibit standing waves just like the waves on the guitar
string in the previous chapter. The key to doing problems involving these cavities is
drawing the pictures correctly.
The most important thing to remember about the Doppler shift is that the detected
frequency is greater than the emitted frequency if the emitter and the detector are
approaching each other, and less if they are receding.
Chapter I 2 Problems
5.
7.
Section C
Use the following informationfor questions 9-1 3:
4.
216
Chdpter
air
13. If the tube were filled with helium, which has a sound
speed of 965 mls, what would be the frequency of the
fundamental?
A.
160Hz
B. 320 Hz
C. 430Hz
D. 970 Hz
GO ON TO ME NEXT PAGE
C.
D.
18. The two lowest notes on the piano are A, (27.5 Hz) and
A#, (29.1 Hz). If you play the notes simultaneously, the
resulting sound seems to turn off and on and off and on.
How much time exists between the successive "on"s?
A.
0.6 s
B.
1.6 s
C. 28.3 s
D. 56.6 s
Use the following informationfor questions 19 and 20:
Sarah has correctly tuned the B string of a guitar. She
frets the string to play an E (660 Hz). The E string is not yet
in tune. When she plucks the true E (on the B string) and the
E string together, she hears a note that changes from loud to
soft to loud twice a second.
Section
resonance
dispersion
I
21 8
B.
C.
D.
367.5 Hz
480 Hz
490 Hz
's
fdet
Passage 1
Bats are mammals which have acquired the ability of
flight and of echolocation. Echolocation involves using
vibrating membranes to direct a high frequency sound, with
frequencies ranging from 12 kHz to 150 kHz. If the sound
encounters a flying insect or obstacle which is larger than the
wavelength of the sound, then a portion of the sound wave is
reflected, and the bat detects it.
Beyond this basic framework, different species of bats
use different strategies in echolocation. Some species emit a
series of pulses, determining the distance to an object by the
delay in return of the signal. Some emit a constant frequency, using the frequency of the returned sound to
detennine information about the velocity of the insect.
Others use a sweep of frequencies, presumably to detennine
size information or directional information. Some emit a
sound with a high harmonic content. Many use some
combination of these strategies.
Several adaptations provide for better processing of
the returned signal, including isolation of the detection
apparatus from the emitting apparatus and specializations in
the middle ear.
For the questions, use the following: The speed of
sound is 343 m/s, and the Doppler-shifted frequency for a
.detector and emitter moving relative to each other is
'det
=fa,
vs +- Ye,
1.
2.
Chapter
5.
A bat is traveling west at 15 m/s, emitting a constantfrequency sound of 50 kHz. If it encounters an obstacle,
such as a tree, what frequency sound does it detect?
A.
2.2 kHz
B. 48 kHz
C. . 50 kHz
D. 55 kHz
The
2.
3.
4.
Passage 2
It is possible to construct a device for determining the
speed of moving objects using sonar. The device consists of
a sound emitter and a detector. The emitter creates a sound
of a single frequency. The outgoing sound reflects from a
moving target and is Doppler shifted. When the incoming
signal arrives at the detector, the incoming signal and
outgoing signal are combined, so that the detector actually
detects the beat between them. The detector is a squareamplitude detector, which is thus able to pick up the beat
(see figures below).
Power
Passage 3
Chdpter
3. Which is true?
A.
B.
C.
D.
4.
Passage 4
12
Sound
31.79
D.
Power
speakers
d,
Alice Bob
A.
B.
Power
Power
fi -f1
I
222
Chdpter
6.
D.
4.
5.
STOP
A. Introduction
Light is pretty mysterious: sometimes it acts just like a wave, interfering with
itself and undergoing Doppler shifts and so on, and sometimes it acts like a particle,
such as when it interacts with an electron. In this chapter we will explore the wavelike
properties of light and leave the particle-like behavior to Chapter 16.
So light is a wave, like sound and water ripples. But sound involves the motion of
air, and water ripples involve the motion of water, but what motion happens in a light
wave? Well, it turns out to be a hard question. The answer is not "this molecule" or
"that substance" but rather a combination of electric and magnetic fields. When all the
material is removed from a piece of space, we call that piece of space a vacuum, but
electric and magnetic fields are still there, and light disturbs these fields. That is why
light can travel in a vacuum.
Here is another strange property of light, or perhaps it is a property of the space
and time in which light travels. Its speed is the same c = 3.00 x 10' d s to every
observer. Think of how strange this is. If you are driving 60 mph down the freeway, a
car may pass you going 75 mph, and a car on the other side of the yellow stripe may be
going 60 mph in the other direction. From the point of view of your car, the first car is
going 15 mph and the second is going -120 mph. This makes sense.
,
In the superfast space freeway, your spacecraft may be going 1.00 x 10' d s and
another spacecraft passes you going in the same direction going 2.00 x lo8 d s . A light
beam passes both of you in the same direction going 3.00 x 10' d s . From the point of
view of your spacecrafi, it is going (not 2.00 x 10' d s but) 3.00 x 10' mls, and from
the point of view of the other spacecraft it is going 3.00 x 10' mls. Go figure.
The speed of light is this mysterious c = 3.00 x 10' m/s only in a vacuum. In
other materials the speed is a little slower.
The
microwaves
lo8 TV, FM radio
106 AM radio
1010
lo4 radiowaves
102
Figure 13-1
fact, the light from the sky is polarized in
most places, as well as light which is
reflected from the hot layer of air on a
desert road. You can detect the polarization using polaroid glasses. Look through
the glasses at the shimmering surface of a
hot road and rotate the glasses. You can
see the light become dimmer and brighter.
To see what is happening, look at
Figure 13-2. Beth is waving her hands up
and down sending waves to Sam.The
waves easily pass through the picket fence
as it stands, but if the slats were horizontal, the waves would not be transmitted to
Sam.Polaroid glasses allow the vertically
polarized portion of light to go through
but absorb the horizontally polarized
portion.
C.
n,
waves
+-
Figure 13-2
substance
vacuum
air
water
glass
1
= 1.3
= 1.5
+q
4-
<
air
glass
Figure 13-5
air
The
Snell's Law
If a beam of light encounters a boundary, and if the beam is
transmitted, the transmitted beam is refracted, or bent,
according to
ni sin 8, = n, sin Or,
(2)
You will want to memorize it since it is in the MCAT study guide, but it is more
important that you know how to use it.
Example 2: A beam of light in air strikes the surface of pure hydrogen peroxide
(n = 1.414) making an angle 30" with the normal to the surface.
a. What angle does the reflected beam make with the normal?
b. What angle does the transmitted beam make with the normal?
Solution: a. The diagram for this problem is similar to Figure 13-4, with glass
replaced by hydrogen peroxide. The reflected angle is the same as the incident angle,
30".
I
I
I
sin 8, = 0.354,
8, = sin-' 0.354
= 20.7",
where that last equation must be solved on a calculator.
Example 3: A beam of light in a
piece of diamond encounters an interface
with air. The beam makes a 30' angle
with the normal. What is the angle of
refraction? (The index of refraction for
diamond is 2.42.)
Solution: Figure 13-7 shows the
diagram for this problem. Snell's law
becomes
Figure 13-7
sine, = 1.21.
'
Now we need to find an angle whose sine is 1.21. But wait a minute! There is no such
Figure 13-8
Figure 13-9
Figure 13-11
Figure 13-10
0.Optics
Using Lenses
&:-------
--------
Figure 13-14
Figure 13-13
For lens problems, there are two formulas and a ray-tracing method. It is probably worth your while to learn both. The formulas are better for calculating numbers,
but ray tracing is better at answering qualitative questions. The method of ray tracing
will sound confusing at first, but it will become more clear as you work through the
examples.
di
do
(3)
and
dm = -2
do
(4)
Figure 13-15
In this example, we had to extend the rays backwards in step 4 in order to find the
image. For this reason the image is virtual,meaning light rays are not actually coming from the position from which they seem to come.
If the light rays pass through the point from which they seem to come, then the
image is said to be real. We can get the magnification by first calculating the exact
position of the image:
Then
The negative sign for dimems the image is on the same side as the object. The positive
sign for rn means the image is upright (not inverted).
Example 2: The eye contains a lens whose focal length can be adjusted. A candle
(2 cm long) sits 0.1 m from the lens of the eye, and the image is focused on the retina.
Assume the length of the eye from front to back is 2.5 cm.
a. What is the focal length of the lens?
b. Is the image upright or inverted?
Solution: We calculate the focus as follows:
1 1 1
-=+-r
f di do
object
Figure 13-16
We draw the horizontal line on the left of the lens in step 2. The other line, you
will notice, takes the near focus into account. The image is where the lines meet,
.inverted and very small.
Because the image is located where light rays actually converge, the image is
real. Figure 13-16 is a physics diagram. In an actual eye, there are many rays which
converge to a point, and none of them need to be the two which we have drawn here.
See Figure 13-17 for a more realistic diagram.
object
image
Figure 13-17
Figure 13-18
We had to extend a ray backwards in step 2. That means the image is virtual. Also
the image is upright. The location of the image can be gotten from the equation:
where the negative result indicates the image is behind the lens, which we knew from
the diagram. The magnification is given by
where the positive result indicates the image is upright. That gives the magnification.
The size is then (0.333) (9.0 cm) = 3.0 cm.
It 'is a general rule that if only one lens or mirror is involved in a problem, then
the image is either both real and inverted or both virtual and upright. It will never be
real and upright, for exarnpIe.
F i r e 13-19
------*
C-
object
image
Figure 13-20
Note.that the second ray uses the same focus as the first ray. The image of the car is
behind the mirror and is virtual and tiny. The exact location is given by
1
1
-=-
di
+-*1
do
where the negative sign indicates the image is behind the mirror. The magnification is
given by
where the positive sign indicates the image is upright. You do not need to pay attention
to the sign conventions if you get the diagram right.
So why is there a warning "Objects in mirror are closer than they appear"? There
are two things going on. The first is that the image is much closer to Larry than the
object itself, and the second is that the image is smaller than the image Larry would see
if he turned around and looked. Larry's brain does not care about where the image is
and does not notice from which point the light rays appear to be diverging. Larry's brain
compares the size of the image to what it knows is the size of a car in order to obtain a
distance to the car.The distance thus calculated is about a factor of two too far away.
f di
1
+-9
do
I
l
l
-=+-,
w
di 4m
o=-+-,
d, 4m
di = - 4 m .
Also
Figure 13-21
Example 3: The image of a candle lies 10.0 meters behind a converging mirror
(focal length 5.0 m). Where is the object?
SoIution: Figure 13-22shows the ray diagram. Treat the image as the object and
thus the origin of light rays. The hard part of this problem (if you have to do the
calculation) is remembering the sign di= -1 0 m. Then
Figure 13-22
The
F.
Dispersion
In Section C we discussed the idea that the speed of light in each substance is
related to its index of refraction n, and the index of refraction governs the bending of
light as it crosses a boundary. Now, although this is true, it is not the whole truth.
A more complete version of the
truth is that the index of refraction
f
color
n
depends slightly on the frequency of the
red
1.4566
4.6 X 1 0 ' ~Hz
light. (Sometimes not so slightly, but that
orange
4.9 X 1014Hz
1.4578
is another story.) The chart shows the
yellow
5.1 X lot4Hz
1.4584
index of refraction for glass. You can see
5 . 8 X 1 0 ' ~ H z green
1.4614
blue
1.4649
6.5 X loL4HZ
7.5 X 1014Hz
indigo
1.4702
--Ad
blue
Figure 13-23
G. Combination of Lenses
When we view an object through several lenses which are near each other, then it
is possible to treat the combination of lenses as one lens.
Combination of Lenses
When several lenses with focal lengthsf,, f,, ..., are near each
other, then the combination has a focal length&,, given by
The quantity l/Jfor a lens is called the power of the lens, measure in [m-' =
diopters = Dl. This word power has nothing to do with the other definition of power,
that is, energy per time. The point here is that the power of a combination of lenses is
the sum of the power of the lenses.
Example: Dieter has an eye which, when the eye is at rest, focuses light to a
point 0.024 m behind the lens, which is 0.001 m in front of the retina. What is the
power of the corrective lens he must wear?
Solution: The power of Dieter's eye is P,, = 110.024m = 41.67 D. The combination of lenses should have a focal length 0.024 m + 0.001 m = 0.025 m, so the power of
the combination of the two lenses needs to be Pa,,= 110.025 m = 40 D. Since PC,,=
Peye+ Pconscl,we have P m I = -1.67 D.
The
Chapter 13 Problems
3.
-nu=
Section B
glass
substance
air
water
hydrogen peroxide
1.414
glass
ammonium bromide
water
1.7
water
lass
air
glass
lass
air
glass
A.
5.
glass
air
glass
glass
air
glass
C.
2.
glass
JZ).
hydrogen
peroxide
9.
7.
B.
sin-'
-1
C.
sin-'
2
-
D.
3
sin-' -
D.
30"
60"
C.
2
sin-' 3
D.
3
sin-' -
ammonium
bromide
-1
A.
sin-'
B.
2
sin-' -
C.
sin-'
D.
11. What is the smallest angle the refracted ray makes with
the surface?
A.
17"
-3
4
None of the above.
12. What is the wavelength of the refracted ray?
A. 300 nm
B. 510nm
C.. 867 nm
D. 1020 nm
241
GO ON TO THE N M PAGE
A.
B.
C.
D.
A.
B.
C.
D.
A.
15. What is the magnification of the image?
A.
B.
C.
D.
0.5
0.6667
1.5
2
B.
C.
D.
7 cm
9 cm
49 cm
63 cm
A.
B.
-4m
-4/3 m
C.
D.
413 m
4m
A.
B.
C.
D.
Section D
{I,
- --------
-------*----------------
6 6 m , 12m
24 m
4
1
B.
C.
D.
1/10 D
5112D
1215 D
IOD
Passage 1
Section G
34. Two thin converging lenses are near each other, so that
the lens on the left has focal length 2 m and the one on
the right has focal length 4 m. What is the focal length
of the combination?
A.
116 m
B.
314 m
C. 413 m
D. 6 m
vertical
polarizer
unpolarized
light
observer
vertically
polarized
light
optically
active
substance
D.
C.
D.
2.
4.
5.
C.
D.
A.
B.
C.
1
less than -I, but greater than zero intensity
2
0
D.
945
The
Passage
3.
4.
5.
B.
C.
D.
On the retina.
Behind the retina.
Both in front of and behind the retina.
6.
incoming
light
A.
B.
C.
D.
2 x lo-' radians
0.045 radians
0.09 radians
0.18 radians
STOP
1 Chapter 14
A long time ago, someone noticed that when amber is rubbed with a cloth it
attracts small seeds or pieces of straw. No one knew why. (Amber is a soft ochre "stone"
of hardened tree sap.) Many years and many experiments later, the following story has
emerged as the best explanation:
Most material on Earth is composed of three particles, called protons, neutrons,
and electrons. The protons and neutrons hold together in tight lumps, called nuclei. The
much less massive electrons exist in a cloud around the nuclei, forming atoms. Sometimes the electrons hold the atoms together, forming molecules and so on. The electrons
are sometimes quite mobile, and this mobility results in most of the changes we observe.
But that's chemistry. That is not our story.
This is our story: Electrons have a
negative charge, and protons have a
positive charge. l b o charges of like sign
exert a repulsive force on each other, while
two charges of unlike sign exert an
attractive force on each other (Figure 141). Most objects are neutral, that is, they
have nearly the same number of protons as
electrons. When our unknown predecessor
Like charges repel and
rubbed the neutral amber it acquired a few
unlike charges attract.
extra electrons, giving it a net negative
charge.
Fire 14-1
Conservation of Charge
If a system is closed (no matter goes in or out), then the total
charge of that system is conserved (stays constant as time
passes).
i
C.
0
+ +
...
A
(left)can induce
a charge in a conductor (right).
Figure 14-4
Generally the induced charge on nonconductors is smaller than that on conductors. Instead of drawing all the polarized
molecules, we generally summarize these
with a picture like Figure 14-5b.
Figure 14-5a
charge
F i r e 14-5b
The following example illustrates
these concepts. In the illustrations, a slight
excess in the number of electrons over the
number of protons is indicated by a - sign,
and a deficit of electrons is indicated by a
+ sign.
ground
Figure 14-6
Figure 14-7a
sphere
The
Figure 14-7c
Figure 14-7d
Figure 14-7e
D. Coulomb's Law
Knowing that like charges repel (and unlike attract) is only a part of the story. We
can calculate numbers as well. For this we must introduce the unit for charge
[Coulombs = C]. A Coulomb is a large amount of charge, and most laboratory situations involve the accumulation of at most lo4 C.
Coulomb's Law
If two simple charges q, and q, are a distance d apart, then
they exert a force on each other, of magnitude
F,,
=-kq,q,
dZ '
(1)
~m~
where k = 9 X 10' 7
is Coulomb's constant. This force is
C
attractive if the charges have unlike sign and repulsive if they
have like sign.
Note that the distance appears in the denominator as a square, so it is called an inversesquare law, just like the law of gravitation.
0
--
E. Uedric Field
Coulomb's law is simple, but it is not the only way we can explain the forces
which charges experience. Often in physics, there are two viewpoints which explain the
same phenomena. Usually, only one eventually prevails. Sometimes it prevails because
it is easier to use, and sometimes it prevails because it provides a deeper insight into the
working of the universe. The electric field is an example of a deeper insight
Again, we consider two charges, now call them Q and q, and assume they are
positive. Fix Q at some point. If we place charge q nearby, it experiences a force away
from Q. Now, using Coulomb's law, we can explain this phenomenon-as a repulsive
force that Q exerts on q.
Or we can explain this as follows:
The charge Q creates a "field of arrows"
around itself, directed outward and
getting shorter with distance from the
charge Q. When we place q at a point,
charge q, being nearsighted, does not see
Q over there. But it does see the arrow,
which tells it what force to feel (Figure
14-9).
Lx
Stop for a minute and think about
Z
this. Both viewpoints explain the observations, but they are very different perspecf
tives. The second viewpoint introduces a
In another viewpoint, q does not feel the
forcefrom Q directly, but Q creates an
new animal: a field of arrows (or vector
electric field, and q feels thisfield
field). Charge Q creates an electric field,
and the electric field tells charge q what
F i r e 14-9
force to feel.
So why do we introduce a new viewpoint? Isn't Coulomb's law adequate for our
needs? Actually, there are good reasons to talk about an independent existence of an
electric field. One is that Coulomb's law is not really designed to deal with moving
charges. What happens if we abruptly move charge Q. If Q is moved to a different
place, then q must experience a different force. But does it feel it immediately, or is
there a delay, and if so, how much of a delay?
Figure 14-10c
The important concept is this: The
universe is filled with arrows, that is, an
electric field. Charges (some stationary and some moving) create the electric field. The
electric field tells charges what force to feel.
There are three rules for the electric field which you need to know for the MCAT.
Rule 1
A stationary charge Q creates at every point an electric field
of magnitude
E = -kQ,
d2
Rule 2
Assume there are several stationary charges Q,, Q,, and so on.
Charge Q, creates electric field E, at point P, charge Q, creates
electric field I?, at point P, and so on. The electric field at P is .
the vector sum
E = E, + E , + ... .
2
Rule 3
A charge q placed at point P will experience a force given by
F = qE.
Figure 14-9 shows the electric field due to a single positive charge. A sketch of
the electric field due to a single negative charge would look the same except the arrows
would point toward the charge.
Example 1: A dipole is a positive charge and an equal-magnitude negative charge
separated by a distance. Sketch the electric field around a dipole.
Solution: Figure 14-1l a shows such a sketch. Each electric field vector shown is
actually the sum of two vectors. Figure 14-1l b shows this explicitly for point P.
A positive charge q placed at point P would experience a force to the right, while a
negative charge would experience a force to the left.
Figure 14-lla
I
I
Figure 14-llb
Example 2: Square ABCD has sides of length
meters. Charges of Q =
1.1 X lo-'' C are placed at corners B and D. (The charge of a proton is 1.6 x
C.)
a. What is the electric field at point A?
b. What force would an electron
placed at point A experience?
fE"
c. If a proton were placed at point
A, what would be the ratio of
the magnitude of the force it
would experience to that which
the electron in b experiences?
Solution: a. First, we DRAW A
DIAGRAM showing the electric fields
caused by the two charges (Figure 14-12).
D
C
The magnitude of the electric field at A
F i r e 14-12
Figure 14-13
This is the magnitude of the electric field due to the charge at D as well. The two
electric fields add like vectors. Figure 14-13 shows this sum, in which we may use the
Pythagorean theorem, so we write
F =qE
The force the electron at point A experiences is directed toward C, since the charge of
the electron is negative.
c. The force on the proton at point A is obtained in the same way, except the
direction of the force is away from C, since the sign of the proton is positive. The ratio
of the magnitudes of the forces on the proton and the electron is 1.
The following example shows how important it is to have a mental picture of the
charges, electric field, and forces, much more important than memorizing several
equations.
0
Simp1e
'fa
water molecule.
Figure 14-14
F =q,E,,.
F-
Figure 14-17
The
Figure 14-18
Figure 14-19
One last note here: To draw electric
field lines, simply connect the arrows of
the electric field, each arrow directing your pen to the next arrow. Figure 14-!8 shows
an example for a dipole, and Figure 14-19 exemplifies two positive charges. This is just
another way of graphically representing the same information.
II
F.
Electric Potential
We begin this section with an analogy.
b. Another bee carries the same amount of charge along Path 2 fiom A to B.
c. A third bee carries a negative charge q, along Path 2.
Honey bees a and b perform the same amount of work. Although honey bee c
performs a different amount of work, it is different only because of the different charge.
The work per charge is the same in all cases. This work per charge is called electric
potential. We could drive a stake into space at point B with the title "7 JIC to get to this
point from A" or something to that effect. This is analogous to gravitational potential
energy per mass or roughly analogous to height.
Thus, to every point in space we assign a number, the electric potential. In
physics, point A is often set at infinity as a standard. In that case, we refer to the
w=qv,.
The units are [JIC = volts = V].
Now the analogy to gravitational potential energy works only so far. For one
thing, mass is always positive, while charge may be positive or negative. So Sisyphus
performs a large amount of work with his large rock, and his son performs a smaller
amount of work. Honey bee a performs a large amount of work, but honey bee c
performs a negative amount of work, so she is able to derive work from the system.
Another difference is that the
Earth's surface is two dimensional, and
the third dimension, height, is equivalent
to the worwmass ratio to get to that point.
Each point on the Earth's surface may be
labeled by a height or the worWmass
ratio. On the other hand, the honey bees
can fly in three dimensions, and the
electric potential is in a fourth dimension
that we can only imagine. Figure 14-22 is
A honeybee with charge I @ C
an attempt to illustrate this. The bee
approaches aflower at 600 volts.
carries a charge of lo4 C, and the flower
Because of the amount of energy this
has a electric potential of 600 volts.
entails, the bee perceives the task as
Figure 14-22a shows how it looks to us.
we would view climbing a mountain
(as in Figure 14-226).
But since it takes a lot of energy to get to
the flower, the situation appears like
Figure 14-22a
Figure 14-22b to the bee.
If we want to know how much
energy is required to move a rock from A
to B, we don't need the exact heights of A
and B above sea level. We need only the
difference in height, the mass of the rock,
and the acceleration due to gravity.
Similarly, in most electrostatic problems,
we do not care about the absolute
potential; we need only the potential
dgerence.
Fire 14-22b
W = qAV,
=
- v*),
dv.
We need to check the sign. Does such an action require work? Or can we derive work
from it? Removing an electron from a positive terminal requires work, and placing it on
a negative terminal requires work as well, so the positive sign in our answer is correct.
It is a good thing to know the amount of work to move a charge from one place to
another, and we know how to do that now, if we know the electric potentials at the
various points. But how do we calculate the potentials? In many situations, this is easier
than it sounds.
V = k -Q
,
d
where d is the distance from P to the center of the charge.
I.
,=
A
Qt.
Qz
Figure 14-24
V - ~ Q Ike,
A
--
dl,
dZA
Now let's check the sign. Since the test charge is positive, we imagine the two charges
Q are mountains next to each other, and A is a mountain pass. Point B is further down,
so the energy change is negative. No energy is required, but instead energy can be
derived. Therefore the negative answer is justified.
The
The other way to do this problem is to regard Q,as fixed and move Q, toward it,
but of course we would get the same answer. Or we could move them both from
infinity. This would be harder to calculate, but since the energy is independent of path,
the answer must be the same.
b. After the particles are released, the potential energy is converted to kinetic
energy. Since there are no external forces and no heat generation, energy is conserved,
J.
and the final kinetic energy is 1.1 x
@&
300 v
200 V
Figure 14-26
G. Magnetic
Fields
-0
-
H. Electromagnetic Radiation
The electric potential is related to energy and charge. When a question mentions
energy and charge. you should immediately think of using electric potentials. The work
to move a charge from point A to point B is W = q(V, - V,). We can find the potential at
point A by simply adding the potentials (V = kQlr) from the charges in the problem.
An electric field is generated by both stationary and moving charges, although we
have calculated only the former. A magnetic field is generated only by moving charges
and affects only moving charges. Qualitative information can be obtained by the hand
rules. The MCAT will not ask for any more detailed information.
The
Chapter 1 4 Problems
B.
C.
D.
4.
5.
Sections A-D
~rn'
k=9x109q-,
cZ.
=- 1 . 6 ~ 1 0 - ' ~ ~ .
IorII.
B.
C.
D.
I or II or rn.
A.
111.
111or n!
266
B.
C.
D.
Na'
A.
B.
C.
D.
Section
H\
0
'H
C is located on the
14. A positive charge Q = 1.1 x
m, and a negative charge of the same
x-axis at x =
magnitude is located at the origin. What is the magnitude and direction of the electric field at the point on the
m? (Right means the positive
x-axis at x =
x-direction.)
A.
7.5 x lo6NIC, to the left.
B.
lo7 NIC, to the right.
C. 1.25 x lo7 NIC, to the right.
D. 2 x lo7 NIC, to the right.
C.
D.
4 x lo3NIC
8x103N1C -
A.
B.
f
\
c.
D.
GO ON TO ME N M PAGE
ttttt
27. If a water molecule is placed between the two plates,
A.
B.
C.
D.
-100C
-2.25 x lo4
2.25 x 104C
l00C
A.
No force.
B.
C.
D.
t
1
+
The
Section
F
A
electrons
.-
transfer
device
lo-" J
2x10-"J
B.
C.
D.
-0.016 J
0.016J
0.08 J
D.
+1.6x10-'~~
+3.2 x 10-l6J
I
Use the following information in questions 39 and 40:
I
37. What is the potential difference between A and B?
A. 0 volts
B.
lo6volts
C. 2 x lo6 volts
D. 7.5 x lo6 volts
turning
radius
k-4
Q
proton
distance away?
A.
B.
C.
D.
2xldvolts
4x ~ O ~ V
O ~ ~ S
5 x 10' volts
7.5 x l d volts
B.
C.
D.
42. Two protons are fired at the nucleus, the second with
four times the velocity of the first. How would the
electrostatic energy of the second proton at its turning
radius compare with the electrostatic energy of the first
proton at its turning radius?
A. It would be greater by a factor of 16.
B. It would be greater by a factor of 4.
C. It would be greater by a factor of 2.
D. It would be the same.
Section
magnetic
lPll
C.
D.
272
proton
u
In which direction must the electric field point?
A.
Up the page.
Down-the page.
Into the page.
Out of the page.
B.
C.
D.
electron beam
B.
C.
D.
52. An electron beam is traveling to the right, and encounters a region R with a magnetic field pointing down. as
shown. ?his region also has an electric field. We
observe that the effects of the magnetic force and of the
Passage 1
'
4.
Upldown.
B.
C.
D.
2.
30m
60 m
120111
C.
D.
Northlsouth or eastlwest.
Upldown.
A.
B.
C.
Northlsouth.
Eastlwest.
Northlsouth or eastlwest.
B.
C.
D.
Passage 2
3.
where
wire
= -1.6 x 10-19c.
2.
4.
wire
wire
1.
2.
5.
STOP
Chapter 15
Electric Circuits
A. Introduction
In the last chapter we developed some intuition about ele-ctric fields and electric
potentials, so in this chapter we will apply this intuition to electric circuits. A simple
electric circuit consists of a voltage source, wires, and resistors, so that charge flows in
a closed path or circuit. This flow of charge is called a current. In this chapter the
concepts to watch are current and electric potential, two reIated but distinct concepts.
Whenever you see a circuit diagram, you should imagine seeing currents and potentials.
By doing this as you read this chapter, you will find that this subject becomes fairly
straightforward.
A voltage source (often a DC cell or battery) is a potential difference enforcer, its
one job being to ensure that the potential difference between the two terminals remains
constant. It does this by chemically transporting electrons from the positive terminal to
the negative terminal. When the potential
I
I
between the terminals becomes the rated
voltage (6 volts, or whatever), the
-r
I
chemical reation in the cell reaches
DC cells o r batteries
equilibrium and the electron transport
stops. The symbols for a cell are shown in
Figure 15-1
Figure 15-1, where the long bar is the
positive end, the end with higher potential.
A wire is simply a long, long cylinder of metal. Because it is a piece of metal,
however, the potential difference between any two points in it is zero. That is to say, all
along its length, a wire has one potential. In our analogy with Sisyphus and the mountain (Chapter 14), a wire is like a plateau, all of it at one height. The reason for this is
simple: If it were not so, that is, if one end of the wire were at a higher potential, then
electrons, being free to move, would rush toward the higher potential and lower it.
Perhaps a better analogy is a mountain lake which is, of course, flat. If one end were
higher than the other, the water, free to move, would flow away from that end into the
lower one.
For this reason also the electric field inside a piece of metal is always zero. If it
were not so, then the electric field would push electrons to one side, and the shifting
electrons would cancel the electric field.
w'
Figure 15-3
Figure 15-4
Ohm's Law
If I is the current through a given resistor and AV is the
potential across the resistor, then
AV = IR.
(1)
Figure 15-7
We label the lower wire 0 volts.
There is 9-volt jump across the voltage
source, so the upper wire is labeled
9 volts. We label the wire between the two
resistors with the potential V,. The
current is the same through both resistors
and the source.
The equivalent water course is
shown in Figure 15-8 in which the current
in the top wugh is the same as the current
through both waterfalls and through the
bottom trough.
Water-courseanalogy for
Figure 15-7.
Figure 15-8
DO
981
Example 2: In the circuit above, what is the potential drop across resistor I?
Solution: If we apply Ohm's law to resistor 1, then we have
AV, = I, R,
= (0.3A)(lOQ)
= 3v.
Generally, we do not have to go through as much trouble as we did in Example 1
because there are two rules for combining resistors:
1
f
If several resistors (R,, R,, and so on) are in series, then we can
replace them with one resistor whose resistance is the sum
RT = R, + R,
+.-..
(2)
becomes
R,
R,
R3
RT
.
If several resistors (R,, R,, and so on) are in parallel then we
can replace them with one resistor with resistance RT, where
1
1
-=-+-+
RT
R2
R,
....
(3)
RT
becomes
Chdpter
1 5 . . . . . . . . . ... . . . . . . . . .
Electric Circuits
Figure 15-9
Figure 15-10
AV, = 12V,
Figure 15-11
RT = i o n ,
Water-course analogy for
the circuit in Figure 15-10.
Figure 15-12 shows the analogous water
course.
Figure 15-12
Figure 15-13
because we do not know a potential
difference across resistor 1 to apply
Ohm's law. (Many students make. the
mistake here of using 6 volts and 1 Q in
Ohm's law, but the potential difference
across resistor 1 is not 6 volts.)
Figure 15-15 shows the result of
Figure 15-14
combining the two parallel resistors.
Figure 15-16 shows the final equivalent
circuit, by combining the two resistors in
Figure 15-15. The total current is (by
Figure 15-15
Ohm's law. finally) 2 A. The total current
is the same as the current through resistor 1
(see Figure 15-14), so the answer to part a
is 2 A.
F i r e 15-16
F i m 15-17
Chapter
15
In addition to knowing this equation, you should understand the discussion which leads
to it.
= (0.57~)(10R)
= 5.7v.
?he potential on the other side of
the external resistor is 0.3 volts. The
potential difference across the internal
resistor is 0.3 volts, and Ohm's law gives
us the resistance
The
1
R=p-,
(5)
A
where p has the units [Ohm meters], 1 is the length of the wire
(in [m]) and A is its cross-sectional area (in [m2]).
resistivity(i2m)
1.5 X lo-8
1.7 X lo4
2.4X lod
You can assume wires have zero resistance unless the passage tells you otherwise.
D. Power
Recall that power is a measure of how quickly energy is transformed, measured
in [Jls = Watts = W]. The power dissipated by a resistor is
P, = IAV,
where I is the current through the resistor and A V is the
potential difference across it.
(It is better to remember how to derive this equation than to memorize it.) The power
provided by a DC cell to a circuit is
Pd, = IAV,
where I is the current through the cell and A V is the potential
Chapter
1 5 . .. . . .. .. . . . . . . . . . . . . Electric Circuits
Example: Light bulbs that you use around the house are designed to have 120 V
across their terminals. For instance, a
120-W bulb uses 120 Watts of power
*120-W" "30-w'
when placed in a socket with a 120-V
potential difference. (Actually it is a little
more complicated. See Section F.) With
that in mind,
a.
what is the resistance of a
" 120-Watt" bulb?
What is the resistance of a
b.
Which b u m brighter when a
"30-Watt" bulb?
30-W bulb and a 120-W bulb
c.
If these two bulbs are connected in
are connected in series?
series, anaplugged into a wall outlet,
Figure 15-20
which bulb would be brighter (Figure 1520)?
Solution: a. Figure 15-21 shows the circuit diagram for a single 120-W bulb
plugged into a potential source. The equation for power is
AV=IR,
1 2 0=
~( ~ A ) R ,
R = 120 St.
120 v
Figure 15-21
Figure 15-22
E. Capacitance
slight positive charge
A DC cell creates a potential
difference between its terminals by
transferring electrons from the positive to
the negative terminal. As it transfers more
and more electrons, the negative terminal
gains a greater charge, and the terminal
exerts a greater force on every additional
electron the cell transfers. Thus it takes
more energy to transfer each additional
electron onto it. Finally, the energy cost of
adding an electron is too much and the
cell stops, but by that time the terminal
potential has been reached (Figure 15-23).
Figure 15-25
.-
Chdpter
15
. . . . . . . . . . . . . . . . . .. .. Electric Circuits
c=-Q
(8)
AV '
where Q is the charge on one plate, and AV is the potential
difference across the plates. The units of capacitance are
[Coulombs/volt = Farads = F].
Example: We place two metal plates parallel to each other and 0.01 meters apart.
One sheet we connect by a wire to the positive end of a 6-V DC cell; and the other, to
the negative end. The two plates now have a 6-V potential difference between them.
The electric field between the plates is uniform (though we will not prove it), that is,
the same direction and magnitude everywhere. A dust mite has a 2 x 10-l4C charge tied
around his ankle.
a. How much work is required for him to cross from the negative plate to the
positive plate?
b. What force does he experience as he crosses?
c. What is the magnitude of the electric field between the plates?
Solution: a. First let's DRAW A
DIAGRAM showing the electric field
(Figure 15-26); second, a diagram
showing the forces on the mite (Figure 1527).
+
-Moving from a negative to a positive
+
-plate is an uphill battle for the mite, so we
+
expect the work to be positive. We write
B
riq
Figure 15-26
=1 . 2 ~ 1 0 - ' ~ ~ .
b. If the electric field is uniform, then the force (F,, = qE) that the mite experiences must be constant. If he goes straight across, then we have the old work equation
W = Fm,Ax cos 4.
Now the mite moves at a constant
velocity, so the forces are balanced and
we can replace Fmi, with F * . Also, we
know cos$ is 1. Thus we write
w = FClrcAx,
and
Fekf =-W
Ax
Fmia
Fe,
Figure 15-27
c. We obtain the electric field from F,, = qE, so that E = 600 NJC.
It is more important to understand the pictures and the ideas than to apply the formulas
in this example.
The equation we derived is worth remembering in its own right:
F. Alternating current
When a DC cell is connected in a circuit, the potential difference between the
terminals stays constant. This is called direct current (hence DC). In a wall outlet,
things are more complicated.
In the United States, the long slit is the ground, connected to the Earth itself,
which is a large supply of charge. This ensures that this terminal stays at a constant
potential which we call 0 volts. The short slit is "hot". Its potential, relative to ground,
Chdpter
15
. . . . . ....
-120
-170
Figure 15-29
Example: Consider your toaster
plugged intithe wall. Thebower company pulls electrons out of the Earth and
pushes them onto a wire. The resulting
electric field pushes electrons on down the
wire, until electrons are pushed into your
Figure 15-30
toaster. Electrons are pushed out of the
toaster into the wire, out of the wire into
the Earth. Any one electron does not go very far, but the signal goes from the power
company to your toaster. (See Figure 15-30.)
Then the power company pulls electrons out of the wire and pushes them into the
Earth. The resulting electric field pulls electrons along the wire, out of of your toaster.
Electrons in the other wire go into the toaster and are replaced with eIectrons puIled
from the Earth.
Sixty times a second. Until your bread is toasted.
Note that only one wire needs to go from the power company to your house,
since the Earth itself completes the circuit.
Generally we do not talk of the line current as being 170 volts (the maximum).
Instead we talk of a kind of average (root mean square) which is V, = 120 volts for the
line current. The following equations hold for alternating current:
Chapter
4.
15 Problems
Section A
Section
R,=MR.
B.
C.
D.
2.
0.4A
0.6 A
1.2A
7.
B.
C.
D.
V,= 6 volts
v, = 9 volts
R=60R
What is the current through the resistor?
A. 0.083 A
B. 0.125 A
C.
D.
0.020A
0.250A
The
MCAT Physics B ~ o k
18. How does the voltage drop across resistor 2 (that is, V,)
compare with that across resistor 3 (that is, V,)?
A.
V, = 2 V3
B.
c.
D.
V2 = V3
v3= 2 v2
V2+V3=36V
B.
C.
D.
1 ampere
2 amperes
4 amperes
8 amperes
Chapter
Sections
C and D
24. What would happen if the points A and B were connected with a wire?
A.
Light 1 would extinguish, otherwise nothing.
B. Lights 1 and 2 would extinguish, light 3 would
remain the same.
C. Lights 1 and 2 would extinguish, and light 3
would be brighter.
D. All lights would extinguish, and the battery would
be shorted.
995
The
A.
B.
C.
D.
Light bulb 1.
They dissipate the same.
Light bulb 3, by a factor 2.
Light bulb 3, by more than a factor of 2.
wire?
A. Light 1 extinguishes.
B.
Light 1 extinguishes, and light 3 is brighter.
C. Lights 1 and 2 extinguish, and light 3 is brighter.
D. All lights extinguish, and the voltage source is
shorted.
34. What happens to the voltage across light 3 if A and B
are connected with a wire?
A. It becomes zero.
B. It stays the same.
C. It increases but not to 12 volts.
D.
It becomes 12 volts.
A.
B.
C.
D.
lamp
2 amps
4 amps
None of the above
D.
300Ohms
Chdpter
1 5 .. . .. .. . . .. . . .. .. . . . . Electric Circuits
variable
-.
'-- - - - - - - - - - - ,
battery
42. The emf of the potential source is 6.2 volts and the
internal resistance is 0.1 Q. If the variable resistor is set
.for 0.5 Q, what is the current through it?
A.
10.3A
B.
15.5 A
C. 30.6A
D. 74.4 A
297
1
2.
B.
C.
D.
Q,
3Ql
9Q1
3.
4.
5.
where
Chapter
4.
Passage 2
B-
Q1
C.
9Q,
81Q,
Passage 3
D.
5.
15
ionosphere,
,stratosphere
C.
281
Thus, a very much simplified model of the Earth and
its atmosphere consists of two conductors separated by an
insulator. Furthermore, there is a net charge of -lo6 C on the
surface of the Earth and a corresponding positive charge on
the ionosphere. The potential between the Earth's surface
and the ionosphere is about 9 x 10' volts. Thus the model
roughly resembles a parallel-plate capacitor.
For these questions, you may consider the charge on
the electron to be -1.6 x
C.
999
GO ON TO ME NEXTPAGE
The
clean gas
B.
C.
D.
4.
-1.8 x
Joules
1.8 x
Joules
1.4 x 10-l3Joules
polluted gas
.-.
A. . right
B.
left
C. up
D. down
4.
C.
D.
5. If the flow rate of gas through an electrostatic precipitator is 100 m3/s, what would be the electrical current
through the device?
A.
0 amps
B.
low2amps
C. 0.8 amps
D.
3 x lo4amps
6.
The
I.
2.
A.
B.
5 m n
2x104sz
3.
4.
D.
posterior
anterior
0.5 seconds
Equilibrium State
C.
Chapter
1 5 . .. . . .. .. .. . . .. . . .
. ..
Electric Circuits
4.
5.
6.
Activated State
(arrows show currentflow)
/
2.
B.
C.
D.
3.
75 amps
150 amps
300 amps
A.
B.
1.6~10-19c
3 x 1 0 ~ ~
STOP
Chapter
16
A.
B.
Introduction
In this chapter we break out of the tradition and viewpoint of classical physics
and discuss results from quantum theory. In the 1890s, physicists began to realize that
the very language of particles and positions, forces and fields was insufficient to discuss
the world of the very small, of atoms and molecules. They needed a new language and
a new way of thinking, and quantum physics was born.
In classical physics, which we have been studying thus far, if we wanted to talk
about a particular electron in an atom, we would specify its position and velocity.
Physics would tell us how that position changed in time.
In quantum physics, we do not even talk about position and velocity in this way,
because it turns out these terms are impossible to define. Particles simply do not have a
definite position nor velocity. Instead we talk about a particular electron in an atom by
specifying the orbital it is in. Orbital it is in? An orbital is a state of being for an
electron. Knowing an electron's orbital means knowing its energy (generally) and
knowing something about its location (often), but not its exact position.
One way to think about it is to think that electrons are tiny particles which move
so fast that we do not know where they are, and the area of space they move in is an
orbital. That is one way to think about it, but it's the wrong way.
It is not the case that electrons have position and velocity of which we are simply
ignorant. An electron truly has no exact position, existing all around the nucleus at
once, although it does exist more strongly in some places than in others.
For instance, an electron in the lowest-energy orbital of a hydrogen atom exists
throughout the area near the nucleus, but it exists 90% within a radius of 1.4 x lo-'' m
and 10% outside of that radius. (That does not mean that it spends 10% of its time
outside of 1.4 x lo-" m. That is thinking classically again.)
As was mentioned in Chapter 14, an atom consists of a tiny positive charge center
m) called the nucleus and a surrounding cloud of electrons (lo-" m). Atoms
connect together by interactions of their electrons to form molecules and ionic solidr,
which comprise almost all of the matter around us.
The nucleus of the atom contains protons and neutrom. Each proton has a charge
+1.6 x 10-l9C (chemists call this +I), each neutron has zero net charge, and each
electron has charge -1.6 x
C (chemists call this -1).
The mass of the proton and the mass of the neutron are about the same
(1.7 x lo-'' kg), and electrons are about 2000 times lighter. As you can see, kilograms
are really too large a unit to use to talk of these masses, so we often use another mass
unit, called the atomic mass unit, or [amu] (or sometimes just u), so that
1 amu = 1.6606 x
kg .
The atomic mass (sometimes erroneously called the atomic weight) is the mass of
an atom in amu. The mass number is the total number of protons and neutrons in the
nucleus. The mass number is approximately the atomic mass of an atom in amu, since
most of the mass comes from the protons and neutrons.
Thus we can describe a nucleus by specifying the number of protons in it, or
atomic number, and the total number of protons and neutrons, or mass number. The
atomic number also determines which element the atom makes up. The notation we use
for a single atom contains all this information. For instance, we denote common helium
by one of the two symbols:
(pronounced "helium four"), where the mass number is 4 and the atomic number is 2.
We really do not need to specify the atomic number, since we can obtain that from the
periodic table. The mass of such a nucleus is about 4 amu (actually 4.00260 amu).
Several atoms which differ in the number of neutrons but have the same number of protons are called isotopes of that element, for example, 3 5 ~and
1 3 7 ~ 1The
. chemical
l 37C1are nearly identical, since the neutrons of the nucleus have
properties of 3 S ~and
hardly any effect on the surrounding electron cloud.
The symbol :He may refer either to a nucleus having two protons and two
neutrons or to an atom having such a nucleus. We write nuclear reactions using similar
notation as that for chemical reactions, except now we are concerned with changes in
the protons and neutrons in the nuclei.
where we have used the symbol i n for the neutron (do you see why?). The number of
protons mentioned among the reactants is 0 + 8 = 8, of which 2 went into the :He. The
number of protons and neutrons all together among the reactants is 1 + 17 = 18, of
which 4 are in He. Tbus the final nucleus has a 6 in the lower left position and a 14 in
the upper left, corresponding in the periodic table to I4cor carbon-14, so we write
When writing reactions, the sum of left superscripts for the reactants must equal
the sum of left superscripts for the products. This assures that the number of heavy
particles stays constant. Also, the sum of left subscripts for the reactants must equal the
sum of left subscripts for the products. This assures that charge is conserved in the
reaction.
There are always at least two product particles in any nuclear reaction. (Exceptions are vanishingly rare.) One of these particles is often a particle of light, called a
photon and symbolized by 7
:H+
':c+ ':N+
'H+
12c+1 3 ~ y+
or
where we have used H for the proton (again, do you see why?).
C.
For an isolated atom, the electrons are in various orbitals. The atom as a whole
has a certain energy, depending on what orbitals are occupied by electrons. Quantum
theory predicts that only certain energies are allowed for a given atom.
For instance, a hydrogen atom can have only energies corresponding to the
equation
The energy of the photons in the laser are (-0.24 x lo-'' J) - (-2.18 x lo-'' J) =
1.94 x lo-'' J. Thus the frequency of the photons is
Figure 16-4
Figure 16-2
Step 1
-./u
Eo
Step 2
El
Step 3
F i r e 16-3
state. This is shown in Figure 16-4. The
upward arrow represents the absorption of
the original photon. The downward
arrows represent emitted photons, so the
atom emits two photons for each one it
absorbs. This phenomenon, in which a
substance absorbs one frequency of light
and emits light of different frequencies, is
called$uorescence.
Do not get thrown off by notation. Sometimes the ground state energy is labeled
E, and sometimes E,. Also, sometimes we say the ground state has zero energy, but
sometimes zero energy corresponds to complete ionization and the ground state energy
is negative. It is a matter of convention which the problem will specify. The ground
state is always the lowest-energy state.
The nucleus is often content to spend many years undergoing no major changes.
The protons and neutrons hold together, while the electrons in the electron orbitals are
doing all sorts of things. Sometimes, however, the nucleus undergoes a change. If this
happens spontaneously, it is called radioactive decay.
There are three main types of radioactive decay. Nuclei which have an especially
large number of protons and neutrons will sometimes throw off a packet of two protons
and two neutrons, called an alpha particle (a).Note that an alpha particle is the same
as the nucleus of the common helium nucleus. This is called alpha decay.
Step 1
step 2
132Th
'r.
Figure 16-5
Generally alpha emitters are not dangerous to biological tissue (provided you do
not eat them), since the alpha particles lose energy very quickly and do not penetrate
very far even in air (several centimeters). That means they generally do not get inside of
you.
The second type of radioactive decay is beta decay. Nuclei with many neutrons,
compared with protons, undergo normal beta decay (p-). In this process, a neutron
decays into a proton, an electron, and an antineutrino. The proton stays in the nucleus,
and the electron shoots away from the nucleus. The speeding electron, often called a
beta pahcle, can be highly injurious to biological tissue, since it is able to speed
through the air and penetmte into the body. Once in the body, it slows down by ionizing
molecules that it passes by, which can be very dangerous if one such molecule is DNA.
The antineutrino is so penetrating that it generally passes through the body and the
planet without depositing any energy, so it is mostly harmless.
Example 2: What is the reaction for the (normal) beta decay of lithium-9?
Solution: On the left side of the reaction we have ; ~ i On
. the right side we place
an electron. For accounting purposes, the symbol of the electron is -ye. Thus we have
the incomplete equation
?L,i+?+ -qe.
In order to complete it, we need to make sure the upper and lower left numbers add up
correctly. Also we add an antineutrino, so we write
before
after
o0o
e0 0
oO.
.o
+coo
00.
2"
8 = proton
o = neutron
Nuclei with many protons undergo
= electron
positron decay (P'). In this process, a
proton decays into a neutron, a positron,
Figure 16-6
and a neutrino. The neutron stays in the
nucleus. The positron, which is a particle
just like an electron (in mass and so on) with a positive charge, shoots away from the
nucleus. The positron is dangerous to biological tissue as well, and the neutrino is
innocuous.
(A word about words: Radioactivity is generally divided into alpha, beta, and
gamma decays (although there are some other sorts of decays as well). Beta decays are
divided into beta decays and positron decays, so the wording can be a bit tricky.
Sometimes writers will be careful to say "normal beta decay" and sometimes they will
not. If it matters which sort of beta decay is required, assume the normal kind unless
otherwise specified.)
where the asterisk indicates an excited state. In this case @ c o gdecays into one excited
state of @Ni,which decays into a second excited state of 6 0 ~ which
i,
decays to the
ground state.
We can measure the time it takes a nucleus to decay in terms of a halfZi$e. This is
the time it takes half the atoms in a sample to decay. The lifetime of an atom is not the
same as the lifetime of, say, humans.
If we imagine a population sample of 1000 humans all born in the same year,
Chdpter
then after 75 years we would expect about half of them (or 500) to still be alive. After
another eight years, only half of those would be surviving. After another five years, it
would be half again. We would expect, 150 years after the birth date, that there would
probably be no survivors.
Consider now a sample of 1000 gadolinium-148 atoms, all generated at the same
moment. This isotope decays by alpha emission to samarium-144 with a halflife of
75 years. After 75 years we expect about half of the original Gd-148 to remain. After
another 75 years, about half of those have decayed, leaving about 250 Gd-148 atoms.
After yet another 75 years, there are around 125 left. Radioactive atoms do not age and
die, but at any moment they have some constant risk of decaying.
Example 5: The nucleus Ru-103 decays to the stable isotope Rh-103. We obtain
a pure sample and measure that its radioactivity to be 16-16millicuries. After 156 days
the radioactivity is down to 101 millicuries. What is the halflife of this isotope?
Solution: The activity of the sample is down by a factor of 16161101 = 16, that is,
down by four factors of 2. That means that four halflives must have transpired. One
halflife is (156 days)l4 = 39 days.
In Example 1, the mass of a Th-232 is 232.0381 amu. The mass of Ra-228 is
228.03 11 amu, and that of He-4 is 4.0026 amu. Notice that the sum of the Ra-228 and
He-4 masses is less than that of Th-232. Where did the missing mass go? This mass has
been converted into energy.
There is a deep connection between mass and energy which we will mention
only briefly here. In this reaction, mass is converted into energy, and the amount of
energy can be determined if we know the masses sufficiently well. The mass deficit m,,
is the difference of the mass of products and the mass of reactants, and the energy of
reaction is given by
where c = 3.0 x 10' mls is the speed of light. This is the famous E = mc2 equation. In
order to apply this equation we must be careful that the units agree.
Example 6: In the decay of Th-232, most of the energy of the reaction ends up in
the kinetic energy of the alpha particle. After a decay,
a. how much energy (in J) does the alpha particle have?
b. How fast (approximately) is the alpha particle going?
Sdution: a. The mass deficit in this reaction is m,,, = (232.0381 - 228.031 1 4.0026) amu = 0.0044 amu. We convert this to kg and multiply by c2 to obtain energy
1-66x
kg)kvo
1amu
108
J;
This energy is mainly in the form of kinetic energy of the alpha particle.
1.66x lo-'' kg
= 6.6 x
1amu
kg.
Then we have
This is quite fast, about 5% of the speed of light, but not so fast that we have to resort to
the full mechanics of special relativity.
In summary for this section, here is a chart of special particles and their symbols
for nuclear reactions:
Particle
Other names
Symbol
proton
:H
neutron
0"
helium-4
a,alpha
:He
electron
positron
photon
y, gamma
e
Y
antineutrino
neutrino
0
-1
0
+l
In this chapter we discussed the physics of the atom and nucleus: atomic structure, electronic energy levels, radioactivity, and nuclear reactions. The key to understanding many physical situations lies in understanding the energy level diagram. An
atom or nucleus can exist only in certain discrete states of precise energy. When the
atom or nucleus makes a transition from one state to another, a photon is absorbed or
released with energy equal to the difference of the energies of the two states.
The key to understanding problems involving nuclear reactions, including
radioactive decay, lies in visualizing the decay and in writing the nuclear reaction
correctly. There are three types of radioactive decay, called alpha, beta, and gamma
decay. These decays vary in the type of particle expelled from the nucleus and the
effect that this expulsion has on the nucleus.
Chdpter
Chapter 1 6 Problems
Sections A and
4.
Physics
'
B.
C.
D.
A neutron.
A proton and a neutron.
Two neutrons.
9.
Section C
A.
B.
-3.0x 1 0 4 ~ ~
-1.3 x lo-'* J
11. If an atom has only the three energy levels shown in the
figure below, what possible frequencies might be in the
spectrum of this atom? (Use h = 4.14 x lo-'' ev s and
c = 3.0 x 10' mls.)
A.
B.
C.
D.
7.2 x
2.4 x
2.4 x
2.4 x
1014HZ
1 0 ' ~Hz and 7.2 x 1014Hz
10" Hz and 9.7 x 1014Hz
10" Hz, 7.2 x 1014Hz, and 9.7 x loi4Hz
314
Chapter 16
Use the following information for questions 15-1 7:
Atoms of lithium are able to absorb photons of
frequency 2.01 x 10" Hz. This corresponds to a transition
from the ground state (at -5.37 ev) to an excited state.
Zero energy correspoads to the state in which one electron
is completely removed from the atom. (Use h =
4.14 x lo-'' ev s and c = 3.0 x lo8mls.)
. . . . . . Atomic
alpha decay?
A. -=8Th
B. 2 2 8 ~ n
C. 230Th
D.
"'~n
A;
'H+ 'H+ l H + y
A.
'H+ 'H+ 3 ~ e y+
B.
'H+'H+~+~H~
C.
' H + 'H+ 4 ~ e + y
D.
'H+ 'H+
3 ~ i + y-
B.
C.
D.
'=~r
"~AC
2 2 6 ~
The
C.
D.
on
1.00866 amu
tH
1.00783 amu
: ~ e
2.01410 amu
4.00260 amu
,Li
7.01601 amu
:-I
7
II
Passage 1
A hydrogen atom has energy levels given by
A.
B.
I
316
GO ON TO THE NE)CTPAGE
Chdpter
3.
4.
Physics
Certain toys found in cereal boxes display the phenomenon of phosphorescence. If a child exposes the toy to a
bright light (for example, the Sun), then the toy will glow
with a characteristic color when the child takes it into a dark
closet.
The phenomenon of phosphorescence requires the
interaction of light with three energy levels in an atom (or
molecule). The figure below (not to scale) shows a hypothetical energy diagram in which the lowest state shown is
the ground state (E = 0). When the toy is taken into the Sun,
a photon causes a transition from the ground state to state 2.
Almost immediately, .the atom emits a photon, making a fast
transition to state 1.
IE=O
ground
state
A.
L b s Cf,,
B.
f*, = f p h
frbr > f*
c.
D.
2.
f, cf*
B. fn = f p h
c* ffl> fph
A.
D.
317
4.
SampIe 2
After the child brings the object into the closet, which
atomic state is the most populated? That is, a number of
atoms have electrons in the ground state, a number in
state 1, and a number in state 2. Which number is
greatest?
A. The ground state is the most populated.
B.
State 1 is most populated.
C . State 2 is most populated.
D. Either the ground state or state 1 is the most
populated.
A.
B.
C.
D.
2.
5. Which is correct?
A. The Sun emits photons of many frequencies, but
only photons corresponding to the absorption
frequency will be absorbed by the object.
B. The Sun emits photons of many frequencies, but
only photons corresponding to the absorption
frequency make it through Earth's atmosphere.
C . The Sun emits photons of one frequency, and the
toy is constructed to absorb that frequency.
D. The Sun emits photons of one frequency, and the
toy converts the photons to the desired frequency.
Sample 1
We observe radioactivity from a sample of 209Po
(Sample I), but the radioactivity is effectively blocked by a
piece of thin (0.05 rnm) gold foil. The radioactive efflux is
made into a beam by placing the sample in a block of lead
(Pb) with a hole which allows the particles to escape as
shown in the figure below). In one experiment the beam is
subjected to an electric field which points up. In another
experiment the beam is subjected to a magnetic field which
points down.
C.
D.
3.
4.
A.
B.
p+
c. pD.
5.
A.
B.
C.
__+
'OSpb
209~t
209~i
209~o
D.
9.5 hours
12.6 hours
15 hours
None of the above is correct.
beam
Chdpter
6.
3.
Which of the following, if true, would explain why Lcapture, the interaction of a nucleus with a second-shell
electron, is extremely rare?
A.
The second shell has greater energy than the first
shell.
B. The second shell has a vanishing amplitude in the
nucleus.
C. The second-shell electrons are easily removed
from the atom.
The
second-shell electrons cannot be converted to
D.
positrons.
4.
ez,
where N is the number of neutrons and Z is the atomic
number, and the symbol means that N is greater than or
approximately equal to Z.
When the difference N - Z is strongly positive, it is
likely that a nucleus will decay by 0- decay, in which a
neutron converts into a proton, an electron, and an antineutrino (which rarely interacts with matter).
When N - Z is only slightly positive or even negative,
then there are two likely modes of decay: 0' decay and
K-capture. In P' decay (also called positron decay), a proton
is converted into a neutron, a positron, and a neutrino.
In K-capture, an electron from an orbital of the first
shell (or K-shell, that is, the 1s orbital) combines with a
nuclear proton to make a neutron and a neutrino. This often
happens in nuclei in which positron decay is forbidden, for
instance, because it is energetically unfavorable. K-capture
is possible because there is some overlap of the first-shell
orbital and the volume taken up by the nucleus, that is, the
first-shell orbital has a nonzero amplitude at the center of
the nucleus.
2.
Passage 4
16
A.
B.
C.
D.
5.
C.
D.
increases by 1
increases by 2
S 6 ~ i -%
+co+e'+v
B.
S6~i-+S6~~+e-+V
C.
S6~i-+56~0+e-+~
D.
S6~i+e--+m~o+v
'
2.
A.
B.
C.
D.
252 Cf
+2S0+
~ fIn + 'n
B.
C.
D.
Chapter
Passage 6
D.
16
Physics
2.
3.
4.
D.
An alpha particle.
nuclear energy.
STOP
Solutions
Solutions
Chapter 1 Solutions
1. A.
We start with the information 1.1 x 10-I2g. Unit
analysis takes us to the answer:
i 1=-
1 . 110-l2
~
1microbe
6x10-16g
1.1 10-12-(-L6)
microbes
6
= 0.2 x lo4microbes
-= 2000 microbes.
2. B.
The question again is, "How much?We have
information in mL,and we want it in mg. Thus we write
5.
=-3000
1000 mg
K rnol
0.082 1L atm
= 3 mg.
3. C.
lmolec(6.02 wlmol
10"molec
Kmol
-2--L--lOatm 40g 1
0.0821 Latrn 1
1 rnol 16g Ar
x ~ ) , ~ x 160 - z 3 g
1mol
B.
I
323
- 2.10-uK
0.08 -16
In the second step above, we multiplied numerator and denominator by 100. In the third step we
calculated 2 40/8 = 10. Most calculations go pretty
quickly if you look for these shortcuts. Our answer is
close enough for us to realize the correct answer is B.
13. B.
14. B.
8. D.
This time there is an r in the formula, so the
volume increases by a factor of 4) = 64.
9.
B.
15. A.
16. C.
Since we have (1 + 501100) = 1.5, the period is
multiplied by 1.5. Thus the frequency is multiplied by
(IS)-' = 0.67. We rewrite 0.67 = (1 - 331100), so the
frequency decreases by 33%.
10. C.
If the radius increases by 30%, that is the same as
increasing by a factor of 1.3. since (1 + 301100) = 1.3. If
the radius increases by a factor of 1.3, then the area
increases by a factor of 1 .32= 1.69. An increase by such
a factor is an increase by 69% so the answer is C. Keep
in mind that you need to know how to manipulate
numbers like this quickly.
17. D.
If s increases by a factor of 9, then V increases by
a factor of 81. Do not let the 113 in the formula throw
you off. If it did, try the problem with numbers to see
why the 113 does not matter.
11. B.
18. B.
This is a simple proportionality.
of f i = 2.
19. C .
12. B
Passage 1
1. C.
I
2.
B.
5. C.
- We can eliminate A and B immediately, since F
decreases as r increases. We can eliminate D, since F
does not have a linear relationship with r (that would
look like F = kr + c). As r approaches 0, the force
becomes infinite, so C is a good choice. Also, as r
becomes large, F approaches 0 but never reaches it.
B.
6. B.
A.
B.
The voltage V and electric field E are proportional, so A is correct. For C to be correct, the equation
would need to be V = Ed + V,.
3. C.
Concerning choice A, a factor of 4 in both q, and
q, will result in a factor of 16 in F, so this choice is
incorrect. If F is to stay the same, and q, increases, then
the distance r must increase, so choice B is incorrect. A
factor of 4 in q, is equivalent to a factor of 2 in r, since
2, = 4. Another way to see this is to solve for r, which
you should do if this discussion was unclear.
4.
D.
If the charge on one ball increases by a factor of 4
(from 2 C to 8 C), then the force must increase by a
factor of 4.
D.
In the previous solution, we realized that experiments 4 and 6 have the property that all the input
variables stay the same except for velocity v, which
increases by a factor of 4. The force increases by a
factor of 16. which means that p must be 2. That is, if p
is 2, then an increase by a factor of 4 in v results in an
increase by a factor of 4, in the force.
3. B.
In choice A, experiments 1 and 2 both use a cork
ball. For choice C, many input variables are altered
between experiments 1 and 6, so it is impossible to
isolate the effect of object density. As for choice D,
experiments 4 and 6 both use a steel ball. Choice B
involves two experiments in which only the density of
the object changes.
A.
Passage 4
1. B.
If v increases by a factor of 2, then the required
energy increases by a factor of 2' = 4.
'
3. B.
Comparing Scott's car to Laura's, all the linear
dimensions are increased by a factor of 2 (see figure).
The cross-sectional area A is width times height (A =
hw), so if both h and w increase by a factor of 2, then A
increases by a factor of 4. Thus the required energy
increases by a factor of 4. The increase in length does
not matter.
4. C.
Julie increases her speed by a factor of 55/50 =
1.1, so the energy increases by a factor of 1.1' = 1.21.
This is an increase of 21%.
5. C
The easiest way to do this is to solve for D, giving
Chapter 2 Solutions
5.
C.
Again we s&quentiallyplace the tail of one vector
on the tip of the previous one. The resulting sum is zero
(see diagram). We could see this in the diagram anyway,
since the arrows seem to cancel out each other.
6.
A.
They exert the largest net force when they pull the
same direction (see figure), giving the total 7000 N.
They exert the smallest net force when they are directly
opposed to each other, giving 1000 N. Therefore it is
not possible for the net force to be 500 N. If this is
unclear, try drawing a few vector diagrams to get a total
of 500 N.
3. B.
From the diagram of the previous solution, we
can see that the answer must be slightly larger than
2 m/s and certainly not so large as 2.3 d s , so B is
correct. To do the numbers, we use the Pythagorean
theorem:
The
7.
(s)'
= (3000N)'
14. C .
+ (4000N)' ,
F,, = 5000N.
8. A.
Since she starts from rest, her initial velocity is
zero.
9.
A.
10. B.
The net displacement is As = 27 krn = 27,000 m.
The total time is At = 75 min = 4500 s. Thus the
average velocity is v, = hldt = 6 mls.
11. D.
During these 9 s, she was accelerating uniformly,
so we can write the equation:
v2 -vl =aAt,
v, = v, + a h
m
~0.9-.
S
13. C .
Since the car is accelerating uniformly, we can
write v,= lf2 (v, + v,) = 17.5 d s .
We know Ay = -10 m, a = -10 m/s2 (approximately), v, = 0 mls (because it is dropped). We want At.
Thus we have
18. B.
22. A.
vi = v:
+2a&,
19. B.
20. C.
23. C.
v2 -vl =aAt,
21. B.
26. A.
= 15 km.
Thus x2 = 3015 km.Remember to be careful with the
units.
The
28. B.
LA&
A B C
D E t
32. A.
The net displacement is Ax = x, - x,, the difference between final and initial position. But in this case
x , and x, are the same, so the net displacement is zero.
33. C.
E
I
p9'
34. B.
4.
D.
We apply the definition of acceleration:
35. B.
36. A.
The car is backing up at the beginning of the
problem, so the velocity is negative. The only choice to
show this is A. The flat portion in the center of choice A
is when the car is stopped. The later flat portion shows
the constant velocity while going forward.
37. C.
A.
6. B.
For any 0.5s interval in the chart, the acceleration
(Avldt) is a constant 1.2 m/s2, even for the intervals near
t = 1.5 s, where the velocity is zero. Thus B is the best
answer.
1. B.
We apply the formula Ax = v,Af + 112a(dt)~=
(0 mls)(4 s) + 112 (10 mls2)(4 = 80 m.
t.
Passage
5.
1. A.
The initial velocity is just that at the beginning of
the experiment.
.
The
Av=gAt,
AV
We have
= g(t, - t , ) .
3. C.
V,
7.
Choice A is nonsense. Choice B is a true statement, but it cannot be an adequate explanation for the
fact, since the lead and iron balls fall at the same rate,
and the force of gravity is presumably different on those
two balls as well. Choice C is a good candidate,.since
the passage mentions air resistance as a caveat. Choice
D is irrelevant.
B.
C.
Chapter 3 Solutions
Case 1
1.
Case 2
Case 3
D.
From the first law of motion, a force balance on
an object implies it has constant velocity. From this we
conclude that the force of gravity and the drag force due
to the air are exactly balanced.
6. A.
(This was a review problem.) We calculate a =
(3.5 mls - 1.5 m/~)l3s = 0.67 d s 2 .
7. A.
2.
3.
D.
8. C.
C.
9.
--
10. C.
The magnitude of the net force is given by Fm=
ma = 167 N.
11. C.
The
13. C .
As in question 12, we write a = Flm. If F increases by a factor of 3 and m increases by a factor of 3.
then a remains the same.
14. A.
17. B.
15. B.
18. B.
We draw a diagram showing the forces on the
woman. Since she is traveling at constant velocity, the
net force on her is zero, and the forces are balanced.
Thus the force of the floor against the woman's feet has
the same magnitude as the force of gravity on her. This
is just the first law of motion. If you chose C, then you
need to study the first law of motion again.
19. B.
We can calculate an estimate of acceleration a =
AvlAf = (1 m/s)/(7 s) = 1/7 m/s2.Thus the total mass is
given by m = FJa = 9001(1/7) kg = 6300 kg. The mass
of the rocket case is then (6300 - 3300) kg = 3000 kg.
This is close to B.
25. A.
20. C.
= 2.
21. A.
28. B.
Gravity acts, of course. What else is touching the
arrow? Nothing, so B is correct. (See figure.)
23. B.
The gravitational force acts on the stove, pulling
it toward the Earth. The answer is B.
24. D.
The
Passage 1
C.
Only the table and the string are touching mass
1. D.
Choices A and B are irrelevant, and comparisons,
such as those in choice C, are meaningless if the units
do not match. The correct choice is D. From the table
we can see that equal jumps of time (for example, 0.2 s
to 0.4 s) result in equal jumps of velocity (0.1 m/s to
0.2 m/s). This is consistent with the statement that
AvlAt is a constant, or that acceleration is constant.
2.
Passage 2
1. A.
We have encountered two ways of calculating
acceleration: We can get the acceleration over an
interval by calculating a = AvlAt. Or we can obtain
acceleration Trom a = FJM. Since we do not want
acceleration over an interval (like 0 to 90 s) but at an
instant (t = 0 s), we will use the second approach, if
possible. We draw a force diagram (see figure). The net
Mg = 2.86 X lo7N force is given by Fnp=F-(2.0 X 106 kg) (10 m/s2) = 8.6 X lo6N. Thus we obtain
acceleration a = FnJM= 4.3 m/s2.
B.
It seems as if acceleration would be a useful
quantity to calculate, so let's choose the interval from
t = 0.0 to 0.2 s, obtaining a = AvlAt =
(0.1 m/s - 0.0 m/s) I (0.2 s - 0.0 s) = 0.5 m/s2. Now we
can look at the interval from t = 0.0 s to 0.9 s. We want
to find Ax, and we have At = 0.9 s, v, = 0.0 m/s, and a =
=
0.5 m/s2. From this we calculate Ax = 112
0.20 m.
3. B.
Gravity certainly acts on m. The only thing
touching mass m is the string, so B is correct. (See
figure.)
2.
A.
4. B.
The things touching mass M are the table and the
string. It is true that the tension in the string is, in some
sense, caused by mass m, but M does not know or care
what the other end of the string is connected to. It only
cares that there is a force due to a string which is
directed to the right.
5. C.
By definition, we have vavg= Addt =
(0.09 m 0.0 m)1(0.06 s - 0.0 s) = 0.15 mls.
3. C.
Choice A is not even a true statement. Choice B is
a true statement, but v increasing with time indicates
only that the ship is speeding up, hence having positive
acceleration. The ratio of Av to At is the acceleration, so
if this ratio is increasing, the acceleration is increasing.
Choice C is correct. The ratio Ax to Av does not have
any obvious meaning.
B.
In order for the shuttle to accelerate, there must
be a force on it by some agent. The third-law-pair force
must be a force of the shuttle on that agent. This
narrows the choices to A and C. Since there is little air
in space, A is ruled out. The shuttle pushes off from the
exhaust, and that is what creates the force which
accelerates it forward, according to the third law of
motion. (See figure.)
Chapter 4 Solutions
6. C.
The weight of the block on the Moon is an exact
= 20 N.
analogy to its weight on Earth: F,,, = mg,
B.
The fact that the stars revolve about their combined center of mass is irrelevant for calculating the
force of gravity between them. That depends only on
their masses and the distance between their centers.
Since we have F,,= ~ m , m , l dif, one of the masses
decreases by a factor of 2, then F,,, decreases by a
factor of 2.
2.
A.
3. B.
If m, decreases by a factor of 2, and m2 increases
by a factor of 5, then F,,, changes by a factor of 512.
4.
B.
8. A.
This time the relevant equation is
A.
D.
The equation we need for this problem and the
next is
In problem 7, we worked out the surface acceleration due to gravity of a planet. Since we are looking for
radius, we can solve for it, so we have
17. B.
Since we have a force diagram, we next need to
inventory the information relevant to the vertical
motion. We have a,= -10 mls2and At = 2 s. Because
the opener is traveling horizontally when it leaves
Barbara's hand, we have v,, = 0 m/s. Since we want to
know Ay, we use the equation Ay = v,Pt + 112 a, (At)2=
20 m.
18. A.
Since we are looking for Ax, we now inventory
the horizontal information. We have ax=0 mls2 and
v,, = 1.5 mls. Thus we have Ax = v,,At + 1/2a, (At)' =
3 m.
19. C.
&.
21. B.
22. A.
Since the initial velocity is horizontal, we have
V l Y = 0 Ids.
The
24. C .
30. B.
25. B.
Ay = -1.25m,
31. A;
Both coins have the same vertical velocity. In
addition, Barbara's coins retain their horizontal velocity.
We can calculate
First, let's draw a diagram showing all the forces
on the wagon body.
I
27. A.
28. A.
Since gravitational force is given by F,, = mg,
and since Barbara's coins have four times the mass of
Alice's coins, the force on them is four times as large.
29.
B.
The acceleration of Alice's coins is a constant
9.8 rnls2 down, as is the acceieration of Barbara's.
Solutions
40. C.
33. B.
We calculate T, as follows:
T, = T sin 30".
34. A.
41. B.
35. D.
36. A.
We don't have enough information to add up the
force vectors, but we don't need to. The problem states
the wagon is nonaccelerating (velocity is a constant
2 d s ) , so a = 0 m/s2 and Fn,,= 0 N.
42. D.
37. C.
Gravity is acting on the bale, of course. The rope
exerts a tension force. The force diagram is shown.
Note that the bale is not in free fall.
Passage
1. D.
For the student cunning along the roof, he starts
from rest (v, = 0 mls) and ends up running v2 = 5 m/s.
We have Ax = 5 m, and we want At. We use & =
1/2(v1+ v2)& to obtain At = 2 s.
2.
38. B.
If "up" is positive, then the net force is given by
F,= 4000 N - (500 kg) (10 mls2) = -1000 N. We have
used a negative sign for gravity since it points down.
B.
The acceleration on the roof is one problem; the
falling is another. For At, we need the vertical information:
Ay =-7.2m,
3. B.
We know gravity pulls down and the roof pushes
up, and these forces add to zero. In addition, there must
be a force accelerating the student forward (to the
right). Surprisingly, it is the roof which exerts the force
forward. His feet push backwards on the roof, and the
roof (by the third law of motion) pushes forward on
him.
C.
D.
where M is the mass of the body, and r is the radius.
The passage states that M for a neutron star is the gme
as M for the Sun, but r is 50,000 times smaller. That
means a,is greater by a factor of (50,000)~.
6. B.
All during the fall the student has the same
horizontal velocity 5 rnls.
3.
A.
The force due to gravity is
Fgm" =-GMm
d2 '
Passage 2
1. A.
We estimate density as follows:
Chapter 5 Solutions
C.
vertical
2.
C.
3. C.
At the top of the orange's path we have v,, =
0 d s . Since this is vertical information, let's see what
other vertical information we have. We have a, =
-10 d s 2 and v,, = 5 d s . We want At, so we write v,, =
v,, + a p t and obtain At = 0.5 s.
4.
B.
We want a,. The horizontal information we have
is v,, = 0 and F,= 6 N. We don't have enough information for the equations of Chapter 2, but we can use a, =
FJm = (6 N)/(3 kg) = 2 d s 2 .
5.
B.
B.
First we draw a force diagram. This problem has
those key words "at constant velocity", which means
there is a force balance on the shoe. The horizontal
forces are equal in magnitude so that their vector sum is
zero, so B is correct. If you chose C, then go back and
read the section on the first law of motion.
B.
We get this from the same diagrams shown in the
solution for 7.
..
14. B.
10. A.
15. D.
In order to obtain the normal force, we need to
consider all the vertical forces. We can obtain the
vertical component of the net force by looking at the
force diagram, so we write
II. B.
(FneJy= N - mg - F,.
Here we have used F,, = mg and have chosen "up" to
be positive. But we know that the sled is not moving up
or down, so the vertical acceleration a, is zero. And
from second law of motion, we know that (F,), = ma, =
0. The above equation becomes
12. B.
There are two things touching the sled: the
ground and the stick. So, in addition to gravity pointing
down, we draw the normal force pointing up and the
force due to the stick pointing downlright. There is no
friction (which would act to the left). So B is the correct
answer (see figure).
O=N-mg-F,,
16. B.
13. A.
We use the following diagram to obtain the
vertical component of the stick's force.
F, = sin 30"
F&k
19. B.
21. A.
30. B.
22. B.
23. B.
v2/r =
(3 m/~)~/(4
m) = 2.25 m/s2.
(F~,);= N - Mgcosa.
Once we know the acceleration of the car, we
necessarily know the net force on the car: F,,= ma =
(1200 kg) (2.25 m/s2)= 2700 N.
~ u t w know
e
that a, = 0,because the crate is not
moving up or down. This implies (F,,,), = 0, and we
have
O=
27. C.
N - Mgcosa,
N = Mgcosa.
(F,()~= T - Mgsina,
where we have taken the positive direction to be up the
incline. The acceleration is
=--T
39. B.
If there is no gravity, then there is no net force,
and Jupiter follows a straight line at constant speed,
maintaining the same velocity vector as it had when the
gravity was cut off.
33. A.
Passage
1. B.
34. C .
2.
35. B.
The frequency is the number of revolutions per
unit of time. Each revolution represents a trip of length
2w R. So the velocity is the total distance per time, that
is, v = 2 z Rf. Iff is doubled, then v is doubled.
36. C .
On the other hand the centripetal acceleration is
a , = v2/r, SO a,, is increased by a factor of 4.
3. D.
Since the man is traveling in a circle at constant
speed, his acceleration vector points toward the center
of rotation, and so does the net force vector. For this to
be so, the magnitude of the gravitational force must be
greater than the magnitude of the force of the ground.
37. B.
The person experiences a centripetal acceleration
because he is moving in a circle, and he experiences a
tangential acceleration because he is speeding up.
C.
4.
A.
The gravitational force for the two men is the
same. Since the scale reading on a rotating Earth is less
than the gravitational force, the correct answer is A.
5.
C.
Solutions ...............................
Chapter 6 Solutions
Chapter
= Tx - 4r*
1. B.
5. C.
We draw a diagram showing all the forces.
2.
D.
Since only the steel is touching the copper block, the
only forces besides gravity are the normal force and
friction. We choose axes which are tilted compared to
the level ground, and divide the gravitational force into
two components. In order to find the normal force, we
look at the "vertical" forces, so we write
But
,),'(
(F,,)~
= 0, and
= N - Gy.
N=Gy
= mg cos 30"
3. C.
Our first thought for this question is the definition, F,= &N, but none of the answers corresponds to
this, since N is not mg. Choices C and D mention the
tension T, so let's look at the force diagram again. The
problem states that the block's velocity is constant,
implying 4,= 0. Thus we must have ( F , ) ~ = 0.
12. A.
The friction in this problem is static friction. We
know this friction must be less than the quantity 4 N in
order for the friction to be sufficient to keep the
surfaces from slipping. Since we have j@ =
(0.2) (0.4 N) = 0.08 N, and F,is less than this, there is
no slipping.
13. D.
= mg sin 30"
9. B.
14. B.
The wall and the pencil touch the card. The problem
states that the pencil exerts a horizontal force (see
diagram), and the wall exerts a normal force and a
frictional force. The normal force is perpendicular to
the surface and the frictional force parallel to it, even if
the surface is vertical. The gravitational force is simply
FP,= mg = (0.004 kg) (10 m/s2) = 0.04 N.
15. C.
The man would have to exert a force in excess of
the maximum possible friction, that is,
I1
= 800 N .
Solutions . .. . . . . .. . . .. . . .. . . . . . . . . . . . . . . Chapter
16. B.
We have gravity, pointing down. The road exerts a
force normal to its surface. Also, there is a force of
friction. Since the car is going uphill with its brakes
applied and tires skidding, the kinetic friction force is
parallel to the surface and downhill. ?Ae force diagram
is shown.
(F,,)~= 0, SO we have
O=N-Gy,
N=Gy
= GCOS
= mgsin 8.
19. C .
20. C.
Did you choose D? If so, go back and read about
the first law of motion. Meditate on it for about fifteen
minutes.
=&N+mgsin8
17. C .
The components of gravity are shown in the
diagram below.
21. A.
Because the car is turning at constant speed, we
know the net force points toward the center of the turn.
There is no force in the direction the car is going
because the car is neither s p e d n g up nor slowing
down. The force diagram is shown, in which we view
the car from the rear and the turn is to the left.
G, = Gsin 8,
G, = mgsin 8.
18. B.
( F ~ )=, N- G,.
22. C.
Friction supplies the centripetal force. (The turn
would be impossible if the surface were frictionless.)
Since the tires are not slipping on the road, the appropriate friction is static.
23. C .
The acceleration is centripetal acceleration given
by a- = v2/r = (8 m/s)'/ 10 m = 6.4 m/s2.
The
25. A.
3.
A.
26. A.
Remember that the expression @ gives the
maximum force of static friction. If any more friction is
required by a situation, then surfaces begin to slip. In
this case, this would mean the car goes into a skid. But
@ = (0.9) (10,000 N) = 9000 N, which is large
enough. Thus the car will not go into a skid.
Passage 1
1.
A.
4.
C.
The rider traverses the circumference of the circle
(2n R) during each period of time T. Thus his speed is
2 n RIT.
5. A.
The upward force must balance the gravitational
force, so the magnitude of the upward force must be Mg
as well.
6. D.
The force of friction (which is the upward force
in the previous question) must be less than the maximum possible static friction (F,),, = @:
Mg < PY
But N is the centripetal force, so we substitute N =
MV*IRto obtain
'
Dividing both sides by M and multiplying by ~ l v gives
The only thing touching the rider once the floor drops is
the side of the drum, which exerts a normal force that
ends up being toward the axis of rotation. It also exerts
a frictional force, which is up (balancing gravity). For
uniform circular motion, we know the net force must be
toward the center of rotation, so we can see that this
force diagram is complete.
Chapter
4.
Passage 2
1.
D.
For turbulence we need the Reynolds number to
be greater than about 2 x loS,so we have
Re> 2 x 10'
B.
According to the passage, equation (1) is valid as
long as the Reynolds number is greater than about 100,
so, using equation (2). we have
Re> 100,
-pvl
>2x10S.
tl
5.
B.
We apply equation (1) again, so we have F,,,=
(0.2) (lo3 kg/m3) (0.01 m2) (2 d s 1 2= 8 N.
6. B.
2.
B.
The diagram shows the car, modeled by a block
shape.
The arrow shows the direction the car would go, so the
shaded face is the cross section we are interested in.
The area is A = (1-5 m) (2 m) = 3 m2. The length of the
car does not matter.
3.
A.
We apply equation (1) to obtain
Fdrag= CPAV'
The
Passage
1.
A.
A.
C.
Even if the density of air changed appreciably (it
does not), it would not help cats survive a greater fall.
This holds for choice B as well. Regarding choice C, if
cats stretch their legs, then this increases their crosssectional area, which would decrease their terminal
velocity, so is a viable possibility for an answer.
Regarding D, the statement is true, but the fact would
not help a cat to have a lesser terminal velocity from a
greater fall.
= Fgrav .
2.
+ 2ayAy ,
B.
3.
N.
C.
According to the passage, the air resistance must
be small compared to the other forces in the situation,
but this is just the force of gravity.
4.
A.
As the ball travels away from the center of the
Solutions . . . . . .. .. . . . . . . .. . .. . . . . . . . . . . . Chdpter
Chapter 7 Solutions
1.
4.
A.
A.
z,
5.
C.
There are two ways to calculate the torque of
force g about the pivot (see diagram).
= 5 Nm.
The torque is clockwise, so the sign is negative.
= 2.5 Nm.
3.
B.
e,
The
6.
B.
9.
A.
The force is perpendicular to the line connecting
= -60 Nm.
10. C.
The tension due to the person pulling the rope
acts at the point B, so the torque due to this tension is
zero (since r = 0 m).
11. D.
7.
A.
rnct = 0
= 4 1 . 5 m)(40 N)
+ r2,
5 = 60 Nm.
This makes sense, because the torque due to the weight
of the book about A should be balanced by the torque of
the tension T about A.
A more difficult way to do the problem is to
calculate T using force balance (F,), = 0 and obtaining
T = 80 N. Then apply the definition of torque.
The net torque on the meter stick about the fulcrum
must be zero. Since the torque due to the force of the
fulcrum is zero, we have
met = 0,
-F,r,
+ F2r, = 0,
m2 = 2.4
12. A.
We draw a diagram (see figure) showing all the
forces on the seesaw itself.
kg.
c.
We draw a diagram (see figure) showing all the
forces on the rope. Since the weight of the book acts at
point B, the torque is zero (because r = 0 m).
16. C .
13. A.
We draw a diagram showing all the forces on the
meter stick (see figure).
Z,
17. B.
= 4 Nm.
18. C .
Torque balance about the elbow yields the
following equation:
The torque due to the weight of mass m is
z = (L)(mg)l .
15. C.
We can take torques about the axis of pulley B. In
both cases we have sin@ = 1, and torque b,alance yields
-dF, - lmg = 0,
21. D.
The sum of the vertical forces must be zero, so
we have
Fh + FBr- mg = 0,
Fh
We do not know if the forces are to the left or right, but
the equation will tell us later on if our choices were
right. If we take torques about A, then torques due to
>
F,, FAY,
and FBy
are zero. Torque balance becomes
dFB,sin900 - lTsin90 = 0,
+ FBI= mg.
22. D.
dFB,- lmg = 0,
1
FBK
= ;T mg.
The positive sign tells us that our choice was correct,
that is, that
to the right.
2-0. A.
If we take torques about B, then torques due to
FAY,FBx,
and FBy
are zero. We can calculate the torque
due to the tension in the wire easily by sliding the force
down the string to the point shown in the figure.
m) Tsin8
Solutions . . . .. . . . . . . . . . . . . . . . . . . . . . . . . . . Chapter 7
23. C.
( l r n ) ~-20Nm-10Nm=0,
F, = 30 N.
26. B.
We draw a diagram with all the forces on the leg
(see figure).
24. C.
"1
sin 90' = 0,
27. B.
25.
D.
This time we eliminate F, and T from the torque
balance by taking torques about point C. The torque due
to Fxis (1 m) F,, since q3 = 90".We can get the torques
due to F,,,, and F2 by sliding the forces upwards,
that it, opposite the d~rectionthe vectors point (see
figure).
-0.3m1+0.6m2= 0,
The
28. C .
30. B.
Stress is defined as
stress =
F
A'
31. A.
The circumference is C = 2nr, so if the circumference increases by a factor of 4, then the radius increases
by a factor of 4. (See discussion in Chapter 1 if this is
unclear.) Then the cross-sectional area increases by a
factor of 42 = 16. Refemng to the equation in solution
29, if area increases by a factor of 16, then A1 decreases
by a factor of 16.
32. A.
Passage
1. B.
Don't let the information in the problem distract
you from the fact that there must be a force balance on
the muscle, since the muscle is not accelerating. Thus
and Fh,, are the same.
the magnitudes of F,,,
D.
Statue B has 8 times the volume of statue A.
Since m = pV, statue B has 8 times the mass. Since
weight w = mg, statue B has 8 times the weight as well.
5. C .
The passage indicates that the breaking point is
related to the threshold stress for the material in the
cable. Stress is force per area. Increasing the length
does not change the force of the load (the lantern
weight) or the cross-sectional area, so the breaking
weight would be the same.
The
Chapter
3.
8 Solutions
B.
The sticking together provides the clue that this is
a conservation-of-momentum problem. We draw a
diagram showing the system before and after the
collision (see figure).
before:
1. A.
after:
before:
I
The collision of A and B does not affect the
momentum of the system, so the answer is the same as
the answer for problem 3.
A.
= Puw '
=P h
-1.1- kg m =(6kg)v,,
S
6. C.
If the track were not level, gravity and the normal
force would not be balanced, and there would be a
component of gravity accelerating the carts. Thus the
momentum of this system would not be conserved. Of
course, if the system included the Earth, then the
momentum would be conserved, but then there would
not be enough information to do the problem.
7.
before:
1-
A.
before:
after:
mto,
ll.A.
The
12. A.
If we draw a diagram,
15. D.
after:
Vbullet
= Palter
13. C.
Since the problem mentions force and time, we
think immediately of momentum. The external force of
3 N imparts to cart A the momentum given by
Ap, = FAt
= 3~
16. A.
Since momentum is conserved during the
collision, the change in momentum is zero.
(s)2
= 6Ns
17. A.
before:
pA= 6 b + o - = 6kgm
-a
S
kgm
S
1.25 kg
19. D.
. An impulse is a change in momentum. If the ball
is initially going in the positive direction, after the
bounce it is going in a negative direction. We calculate
Ap = p,- p, = (0.3 kg) (7 m/s) - (0.3 kg) (-5 m/s) =
3.6 kg m/s.
21. A.
When the first ball hits the floor, it has a momentum vector pointing down. After the bounce, its
momentum vector points up. The impulse imparted by
the floor is shown in the first figure. (Recall that p,=
Pbelorc + 4 . )
18. B.
before:
after:
impulse
.A
before:
after:
ball 1
I AT
m
3- S
4 = 0- (mw)(v,,,)
The impulse is the same in either case, so choices A and
B are incorrect. Choice C mentions force and time, so
we think to write
&=FA.
Riding the punch has the effect of increasing the time of
contact with the fist and thus decreasing the magnitude
of force of the fist on the face, so choice C is a possibility. Choice D is not relevant. This question is not really
a very good one, but it is typical of some of the questions on the MCAT.
Passage
1. C.
Both the second and third laws of motion concern
unbalanced forces. But the third law of motion states
that, if the ship exerts a force on the gas, then the gas
exerts an equal and oppositely directed force on the
ship, so C is correct.
2. C.
Choice A is a true statement: Neon is not a
product of uranium fission, but neither is hydrogen. The
passage says that hydrogen is heated and then expelled.
This is different from conventional rockets in which the
products of the reaction themselves are expelled.
Choice B is a true statement but also irrelevant, because
the hydrogen does not react chemically in this process
either. Concerning choice C, let's think of exhaust
velocity. It is related to temperature and molecular mass
of the exhaust gas. Since neon is more massive, the
exhaust velocity will be less, and the thrust will be less.
Choice C is correct.
3. D.
According to the passage, the hydrogen can not
be heated fast enough. This would result in a low mass
expulsion rate.
4.
D.
It helps to visualize this problem if we draw a
diagram (see figure).
From the figure we can see that, reiative to the ship, the
gases are going 5000 m/s.
5. A.
Let's draw a diagram of the system.
before:
after:
=Par
6. B.
This problem asks about force, and since a time is
given in the problem, we immediately think to write
A
Ap = F,, At.
The
Chapter 9 Solutions
8. A.
We have p = mv = (4 kg) (3 d s ) = 12 kg m/s.
9.
C.
3. B.
The gravitational force and the direction.of travel
are perpendicular, so cos $ = 0.
4.
A.
11. B.
The gravitational force is perpendicular to the
direction of travel, so cos I$= 0.
12. D.
The friction does negative work on the sled. The
only other work is from the rope (question I), that is,
260 N. Since the total work is 0 J, the friction must do
-260 J.
6.
B.
The box is not sitting on a surface, so there is no
normal force. The man is certainly pushing up. Is he
pushing forward? There is no reason to think so, since
the box is moving at constant speed in a straight line.
Solutions . . . . . . . . . . . . . . . .. . . . . . . . . . . . . . . Chdpter
We set
13. B.
O+mgH=E,+O,
19. C .
As in problem 17, we obtain E,=
were doubled, then E, would double.
mgH. If H
20. A.
Change in energy is always final minus initial.
Thus we have E,- E,, = 0 J - 112 (1000 kg)(20 mls), =
-2 x 105~.
15. A.
16. D.
In this case the direction of travel is in the
opposite direction of the road's force, so cos $ = -1.
Thus W = FAx cos 4 yields 8000 N.
17. C.
This is one of the cases in which the simple
statement of the conservation of energy holds. The only
force acting on the cat is gravity. (See figure.)
22. A.
Using conservation of momentum gives
PI = P 2 3
31. D.
The first method of doing pulley problems
involves drawing a force diagram for the bottom pulley,
as shown. This gives the force equation T + T - mg = 0,
so that T = 112 mg.
D.
The amount of energy expended is AE = P A , by
definition. This is equal to the increase of the potential
energy of the box. The energy does not go anywhere
else: not into heat because the transfer is 100% efficient
and not into kinetic energy because the box moves
slowly and at constant speed. Setting P A equal to mgH
yields H = 40 m. Note the the 30' angle had nothing to
do with the answer.
28. A.
38. C .
33. C .
41. C .
35. B.
36. C .
Whenever we see force and distance, we think of
energy and the equation W = FAxcos $. In this case,
cos @ = 1, so that F = 130 N.The force is the force that
the track, through friction, exerts on his feet.
The
45. C.
A.
PI =p29
50. B.
This is the correct, although contrived, answer. It
is correct because the forces are, in fact, balanced and
the momentum stays constant throughout the problem.
It is contrived because this information does not allow
you to calculate anything of use.
46. B.
In this question we are talking of part 2 of the
event. The forces on the block (with bullet) are gravity
and the tension of the strings. But the tension is always
perpendicular to the direction of travel, so the tension
forces do no work, and the simple statement of the
conservation of energy applies:
51. A.
EKI+ E P 1 = E K Z + E P 2 '
- "after
h = - 2=
1
(5.):
z(10;)
= 0.11 m = 11 cm.
EK1+ E P 1 = E K 2 Em.
+
Passage 1
B.
It is true that there are more particles on the left
side of the reaction, but that would tend to make the
pressure go down, not up, so A is false. The pressure
goes up because the temperature goes up, so B seems a
good choice. Spontaneous reactions can have either an
increase or a decrease in pressure, so C is incorrect. D
is incorrect for the same reason.
2.
C.
D.
One of the reactants would be in excess. The heat
of reaction is unchanged, so A is false. The combustion
would still ignite, so B is out. The answer is C or D. If
any of the intermediates in the reaction were stable
compounds, then some of those compounds could end
up in the waste gas. But the reaction of hydrogen with
water is clean, and the waste gas would contain only
leftover hydrogen or oxygen.
A.
4.
B.
C.
Since both gases are at STP, the ideal gas equation guarantees that the two gases have the same
number of moles per unit volume, hence N is the same.
Neon has 10 electrons per gas particle, and helium has
2 per particle. So neon has a Z which is 5 times larger
and removes 5 times as much energy per unit distance.
6. A.
When the gases are introduced in the chamber,
the ideal gas law is P,,AL = n,,$?T,,, where n,,,, is the
number of total moles of both gases. Only one third of
those molecules are oxygen, so A is correct.
A.
1. C .
Passage 2
1. D.
If the collision is isolated, then there are no
external forces and momentum is conserved, so I is
true. The word "elastic" implies that kinetic energy is
conserved, so I1 is true. The total energy is always
conserved in an isolated collision, so 111 is true. Thus
the answer is D.
3. D.
On the right-hand side of the equation, everything
is constant except N, 2, and z. In this question we are
, the only difference is z. That
comparing 'H and ' ~ e so
is, 'H has z = 1 and 'He has z = 2. Since z is doubled,
' ~ must
e lose 4 times as much energy per unit distance.
2.
D.
The energy of motion is turned into heat, so the
equation is
3. C .
Solutions
4.
B.
3.
. . . . . . ...... . . .. . .. ............
Chapter
C.
+ EPI = EKZ + 4 7 . r
5. A.
The car initially has kinetic energy 1/2 M,v'. Of
this, only energy 112 ahi,vZ is placed into the flywheel.
And of this, only energy 112 a b ~ , , vis~transferred back
into kinetic energy, which is 112 M , , V , ~ . Thus
Passage 4
1. C .
EK2= M g H L .
2.
C.
7.
C.
A glance at the answers shows expression which
look like centripetal force and gravity. The force
diagram is shown.
The
2.
3.
4.
B.
B.
A.
The energy in the motor certainly starts out as
electrical. If the bumpers dissipate the energy, then the
energy ends up as heat. In the ride itself, energy is
sloshed back and forth from kinetic to potential.
Passage 5
+ EpI =
1. C .
A.
The rate of reaction does depend on surface area,
concentration, and temperature, but grain size affects
only the surface area. So A is the answer. Only a
catalyst could reduce the activation energy.
8. C .
9.
C.
+ Ep2
1
-mv2 = mgh,
2 .
h = -v 2
2g '
Solutions . . . . . . .. . .. .. . . . . . . . .. . .. . . . . Chapter
Chapter 10 Solutions
IO
1. C.
FB = mg,
A diagram of a human head is shown.
5.
C.
The buoyancy force is FB= p,,V,, g, but V,,, is
the volume of the man dp,,. Thus F, = (p,, I p-) mg
= 1.2 x
(686 N) = 0.8 N.
7.
D.
First, we draw a force diagram, as shown.
3
V, according to the problem.
4
-3
4'
8. D.
We apply the formula
P = Pa, + pgh
= (1.0 x 10'
+ lo3 - i 0 . 5 j Pa
9.
A.
10. A,
11. A.
The questions here are a bit confusing, so let us
think about what the muscles which expand the lungs
do. During normal breathing conditions, the pressure
inside your lungs is similar to the pressure outside, so
these forces balance. To breathe in, you exert a small
force to expand the push out the chest (and pull down
the diaphragm). A force diagram for the chest is
shown, where Pi, is the pressure of gas inside the lung,
and P, is the pressure of fluid outside your lung.
Po = Ps + pHgg(h1 - hz)
13. A.
This is a simple application of Pascal's law.
14. A.
Since the pressures are the same P, = P,, we can
substitute an expression for force P = FIA. Thus
F,IA, = FiIA,, and F, = (A,IA,)F,.
Solutions
. .. . . .. ... . . . . . . . . .. . . . . . . . .. Chapter
10
20. B.
15. B.
The long way to do this is to realize that a change
of volume on one side of the press is the same as a
change of volume on the other side of the press. If the
piston on the right moves Ax, and the load moves Ax2,
then
AV = A&, =
But
W , = F,Ax, =PA,&, = PAV,
and
W2= F2Ax2= PA2&, = PAV.
The short way to see this is to realize that the flow
of energy is from the piston doing the work on the fluid
to the fluid doing work on the load, increasing its
potential energy. We assume no energy is lost to
friction, so we must have W , = W2.
16. B.
P, = Pa,
+ p,,,g(h2
-h,)
which becomes
- Pam
= ~Ruidg(~2
-hl)
'
Compare this problem with the needle in Example 2 of Section E. The circumference about the
needle turned out to be twice the length of the needle.
Here we have bent the needle into a rectangle, so the
length L that goes into F,, = yL will be L =
2 (l+w+l+w). This is not like a rectangular bug foot
(see Example 1 of Section E) in the water. The circumference for the thread has to go all around the outside as
well as all around the inside.
19. A.
The relevant length for the wire circle is L,, =
2(2nr),just like Example 2 in Section E. The coin's
relevant length is 2 m , just like Example 1.
25. C.
31. C .
27. B.
Increasing the flow rate increases the likelihood
of turbulence, so A is incorrect. Making the joints
smooth does not seem to do anything at first glance, but
perhaps this would remove obstacles that would create
turbulence, so B is a possibility. Reynolds number says
nothing about temperature, so C is also incorrect.
Increasing the radius of the pipe increases the likelihood of turbulence, so D is incorrect, and the answer
must be B.
32. B.
This is a question about hydrostatic equilibrium,
=
pdh3 -
Pbottom
33. B.
Loolung at the answers, we see that the question
writer intends for us to apply Bernoulli's equation. A
streamline goes from the reservoir (pressure P,,
velocity 0) to the pipe 2 with no gravity gradient, so
28. B.
We can consider a streamline which goes from
the reservoir (pressure P,, v = 0) to the nozzle (pressure Pa,, velocity desired). Since gravity plays no role,
the pgh terms drop out. Bernoulli's equation gives
+ 0 = Pa, + -1 p,2,
2
so B is correct.
34. B.
Relating the velocity'vl tb velocity v2 is a matter
of continuity. The flow f has to be the same all along
the flow, so Alv, = A2v2.Thus B is correct.
29. A.
35. C .
I
30. B.
The viscosity depends only on the substance and
not on the situation, so A is incorrect. The water does
develop turbulence, and this would disqualify the flow
from Bernoulli's principle, so B is a possibility. C is
irrelevant, as well as D, so B is correct. The energy of
the flow goes into turbulence and eventually into heat.
36. A.
38. C.
Even if the pump draws a perfect vacuum, the
column will not rise above 10 m. Consider the situation
shown, which turns out to be impossible.
beverage
tubeC
We write the force equation
The pressure at the top of the column is P,,,, and we
write
'sun = Plop
D.
This problem is similar to other problems we
have done. In order to achieve force balance, the sink
must exert a force mg - pw,,Vdi,g. The problem gives
all the information except V,,. The capacity of the cup
is not the same as the volume the cup displaces when it
is under water.
+ ~bevexa~eg~
*
39. B.
We can obtain the height of the water column by
setting P,, to 0. Thus, P,, = P,, + pgh becomes
h = PJpg. Using a different straw clearly has no
effect, but decreasing p will increase h.
40. C.
We have to careful of units in this problem.
Pressure is the connection between gas data and
information about force. We do not need PV = nRT,
because we already know P = 2 atm. In order to
calculate force, however, we need Pascals, so P is
approximately 2 x 10' Pa. Using F = PA, where A =
0.01. m2 (the area of one face), we obtain that F is
approximately 2000 N.
The
41. A.
pw,4
'boy
- PamA - F b o y = 0
= (Prater - Pam)'
44. A.
This reminds us of a Bernoulli question (especially with the explicit "ignore viscosity"), and we can
draw a streamline as shown. The pressures at points 1
and 2 are both Pa,. The velocity at point 1 is essentially 0. If the top of the ocean is the standard height,
then the height of point 2 is -1 m. Bernoulli's equation
becomes
The pressure of the water can be obtained by hydrostatic equilibrium: P,,,, = Pam + pgh, where h is 1 m.
Solutions . .. . . . . . . . . . .. . . . .. . . .. . . . . . . Chapter
Passage 1
Fsu6= mg ,
2my= d h p g ,
h=-. 2~
rPg
From this we see that height h increases proportionally
as r decreases.
C.
Certainly there are pressure forces, but these
balance (both are F = P,,ar2). The surface tension
pulls the column up, balancing the force of gravity.
Passage 2
1.
3.
C.
2.
C.
The length of the line of contact is the circumference of the straw, so L = 2 m
6.
D.
According to this equation, the height h depends
only on the atmospheric pressure, the density of the
fluid, and the acceleration due to gravity.
5.
B.
In this model a maximum height is obtained by
setting the pressure at the top of the column to zero.
= Ptap+ pgh becomes Pa, = 0 + pgh. Thus
Thus, Pbollom
h = P,,/pg = lo5 Pa / (lo3 kg/m3) (10 rn/s2) = 10 m.
C.
The pressure at the top of the column is given by
F = PA = ~ , , s r r ~ .We use the air pressure since it is the
air that exerts the downward force on the column.
4.
10
B.
The force diagram for the column of water is
shown.
= 72 meters.
h = 2ylrpg =
(2 x 10-~m)(10~
3)(10~)
4.
A.
Passage
D.
7.
B.
The table in the passage allows us to calculate the
heat released per mole of quinone produced. We need
to know the concentration of quinone to know how
much heat per quantity of solution is produced. Thus A
is incorrect: not enough information. Once we know
the heat available per quantity of solution, then we can
use the heat capacity to get the temperature change. B
is correct. As for choice C, the heat of reaction is
necessary but it is derivable from information in the
passage. As for choice D, the volume of the chamber is
unnecessary information, since the question is about
intensive properties of the solution and not about
extensive properties. That is, the question is not about
how much stuff there is, but about quantities that do not
depend on volume (such as temperature).
1. B.
Option A seems good, except it is the purpose of
thejlter screen to filter out byproducts, so this is not a
likely possibility. The rupture film would certainly
keep in the reactants, increasing the concentration and
speeding the reaction. B is a likely possibility. The
rupture film does keep the reactants dry, but that is
important during the days and years before the accident,
not the 1-5 rns during the reaction, so C is incorrect.
There is no catalyst involved, so D is incorrect.
B.
A catalyst affects activation energy but has no
effect on the heat of reaction.
5.
A.
Choice A reminds us of Bernoulli's principle,
which might give an estimate of the pressure inside the
outer chamber, even if some energy is dissipated as
heat. Indeed there is no reason to assume the flow is
not turbulent and not viscous. But if Bernoulli's
principle applies, then A is a good estimate. Choices B,
C and D definitely remind us of an ideal gas, but the
passage clearly stated that the spray is a liquid, so these
choices are definitely incorrect.
2.
A.
In the absence of other information about entropy,
it is reasonable to assume that the reaction which
produces the most gas has the largest entropy increase.
At a given temperature gases tend to have much more
entropy than liquids and solids.
3. B.
Using P V = nRT, we note that 1000 / (0.0821)
(300) is the number of moles df gas we desire. Multiplying by 619 (because of the reaction coefficients)
yields the number of moles of sodium azide required.
Multiplying by the molar mass (65 gramslmol) yields
the number of grams required.
I
Having the bag deflate will not affect the temperature appreciably, so A is incorrect. There are a
number of ways to ensure that a bag will not burst
(safety valve, and so on) that are better than having it
deflate, so B is incorrect. The energy of collision or,
rather, the work that must be done on the driver to stop
him is a constant (equal to the negative of his kinetic
energy before the collision), so C is incorrect. The
language of choice D reminds us of W,o,= FAx. And,
indeed, the presence of an airbag increases the distance
over which the decelerating force acts: not all at once at
the steering wheel, but "gradually" over 0.3 meters.
This decreases the force and reduces grievous bodily
harm.
5.
6.
Passage 5
C.
It is not clear how a larger area of the bag could
affect the flow rate. A larger area of the nozzle could
increase the flow rate, but the question is asking about
the airbag, so A and B are incorrect. Option C reminds
us of the formula P = FIA. If A is increased, and F is
the same, then pressure is reduced, hence the cushioning effect Thus C is correct. D makes a correct
statement, but doesnot lend advantage to the driver, so
it is incorrect.
IO
2.
D.
The pressure all along the flow is the same,
except for the tiny region where Barometer 2 disturbs
the flow. Thus the pressure measured by Barometer 1 is
the same as the upstream pressure. If we take the
streamline shown in the figure, then upstream we have
pressure PI and desired velocity v , and at the point in
front of the barometer we have pressure P2 and v = 0.
Bernoulli's equation becomes
D.
Option A, chemical to heat, misses the point. If
this were a reaction that generated a lot of heat that then
dissipated, then A would be a possibility. The point of
energy release in an airbag is that the airbag is actually
inflated. So A is incorrect, as well as B. Option C
seems like a possibility, since the bag seems to gain
kinetic energy, which might later be turned to heat.
Option D is a better description; however, since the true
energy flow involves pushing back the atmosphere and
not in moving the bag (which has little mass).
3. C .
Since the flow must go through a smaller area, the
velocity must increase to maintain the same flow rate
f = Av.
4.
A.
Bernoulli's equation (neglecting the gravity
terms) is
The
5.
+ heat energy
volume
Chapter 1 1 Solutions
II
1. D.
4.
B.
The extension x is given by
x = F,,,Bk = (10 N)/(50 Nlm) = 0.2 m. Since the radius
of the circle of revolution is 2 m and since the spring is
pulling, the resting length of the spring must be 1.8 m.
5.
A.
We simply use
1
E,= -kr2 = (0.5) (50 Nlm) (0.2 m)' = 1 J. Of course,
2
on the real MCAT, the answer to a problem would not
depend on the answer to a previous problem.
A.
12. C.
EKI+ E P I= En +EPZv
15. A.
16. B.
The relationship between frequency and spring
constant is given by
11. C.
The change in gravitational energy is -mgL. The
1
change in spring potential energy is -kr2. These should
2
add to zero, since energy is conserved (there is no
change from begin point to end point in kinetic energy):
17. D.
There is no relationship between amplitude and
spring constant. Any amplitude may be chosen for the
oscillation.
11
18. B.
In an oscillating system, the energy is going back
and forth between two or more forms (so A is incorrect).
It cannot dissipate as heat, so D is incorrect. Gravitational potential energy plays no role in the problem, so C
is also incorrect. Indeed, spring potential and kinetic are
the two forms of energy.
23. A.
The acceleration is the greatest when the net force
is the greatest, that is, when the spring is compressed or
extended. The net force is to the right when the spring
is extended. Some readers confuse acceleration with
velocity (or speed) and think that the block must be
moving when the acceleration is great. This is not so.
24. D.
During the collision, most of the initial kinetic
energy gets dissipated as heat. The choices involving
chemical energy would also be possibilities (since the
tearing and grasping of velcro is chemical), but any
answer must also involve heat, so D is the best answer.
The question is a little vague on purpose, since some
MCAT questions are like that.
25. B.
26. C.
The wavelength is the distance from peak to peak.
27. A.
28. A.
22. A.
If we try to think of forces, we will get confused,
because the spring force keeps changing. Again this is an
energy problem, in which the kinetic energy (after the
collision) gets completely converted to potential energy:
30. D.
The
MCAT Physics
Book
31. A.
The passage stated that p = pA. Thus if the
diameter is decreased by a factor of 2, then the radius is
decreased by a factor of 2, the area is decreased by a
factor of 4, and the linear density is decreased by a
factor of 4.
32. C.
We have a node.
Since we want to keep v the same and T the same,
we therefore (since vZ= Tlp) want to keep p the same.
Thus we have
pA = const,
39. C.
If the waves are out of phase, the combination has
its minimum amplitude of (0.5 - 0.3) Pa = 0.2 Pa. If the
waves are in phase, the combination has its maximum
amplitude 0.8 Pa.
40. C.
second
harmonic
41. A.
Since the frequency is inversely related to the
wavelength (from v = ilf),the ratio f1& is 112. We could
also write
Solutions
42. C.
...... .. ............... . . . . . . C h d p t e r
11
Passage 1
1. C.
A.
3.
C.
44. B.
C.
-.
?he sixth harmonic has five nodes (not including the ends). This is shown in the figure. The
wavelength is d = 113 (0.65 m) = 0.22 m.
=1.5x104 Hz.
= 2.25 x lo4Hz.
Passage 3
1. B.
1. D.
D.
3.
D.
Choice A is incorrect, since we certainly do not
want to convert light energy to anything else. Choice B
is incorrect, since the situation more closely resembles
paragraphs 1 and 2, the coating providing a gradual
transition. Visible light waves have a small wavelength,
so a thin coating would suffice. This would decrease the
reflectivity.
5. C.
Choice A is incorrect, since the point is that the
.
snow is not reflecting sound, resulting in your not
hearing it. Choice B would be a good explanation, if the
coat of snow were thick compared with the wavelength
of sound (paragraph 2). This is not the case. It must be
the case that the snow absorbs the sound energy, turning
it to heat.
6. B
Choice A is incorrect, because the transition from
air to wood is abrupt. Choice B seems likely. There is
nothing to indicate C is correct. Choice D is definitely
incorrect, since the frequency of a wave stays constant
as it travels from place to place. There is nothing in the
passage indicating a change of frequency.
A.
The lowest frequency is 20 Hz, so T = I&=
1420 Hz) = 0.05 s.
I
I
5-
n.
Frequency is not directly connected to intensity.
6.
D.
Choices A and B are irrelevant. doncerning
choice C, the problem is not energy reflecting off the
organ but being absorbed by it. This is an example of
resonance, and there must be a weak coupling between
two oscillators: the sound is one and the oscillating
organ is the other.
7. D.
Chapter
1 2 Solutions
12
5. C .
A.
3. A.
The 40 W goes out in all directions. If you stand
6 m away from the sound, then you can imagine a ball
around the speaker of radius 6 m. The total surface area
of that ball is 4z 3. (If you have forgotten this formula,
then take a minute to memorize it.) Since intensity is
power per area, we can write
6. D.
In the last problem we determined that the power
produced by one mosquito was 1.3 x lo-" W. To power
a 10-W bulb, we would need (10 W)/(1.3 x lo-'' W) =
10110-'I = 1oL2.
7. B.
8. B.
Moving from 30 to 3 m away decreases the
distance by a factor of 10, and this increases the
intensity by a factor of 100 (two factors of lo), and this
adds 10 to twice, so = (20 + 10 + 10) decibels. You
can work this problem by explicitly using the formula,
but that is more difficult.
The
9.
14. D.
D.
10. A.
15. C.
B.
16. B.
17. B.
12. A.
18. A.
The beat frequency in this case is f,, = 29.1 Hz 27.5 Hz = 1.6 Hz.Thus the period is T = Ilf= 0.6 s.
19.
13. A.
D.
The problem states that the beat frequency is
twice a second, or 2 Hz. This means that the E string
frequency differs from true 660 Hz by 2 Hz.Thus it
could be 658 Hz or 662 Hz.
21. C.
Choice A is tempting since the mini-passage is
about beats, but we hear beats only at the end of the
procedure. For choice B, interference is the addition of
two waves in the same medium, which is what happens
when waves from both strings combine, but that is not
the excitation of the C, string. Choice C is when energy
gets transferred from one oscillator to another of similar
frequency by a weak coupling, so C is the answer,
Choice D, dispersion, is the spreading of waves due to
the dependence of wave speed on frequency (which we
have not discussed).
= 490 Hz.
25. C.
This time the detector is moving, and we choose
the positive sign in the numerator to obtain an answer
greater than 420 Hz (again they are approaching). Thus
,,,
f,= 350
(420 Hz)
8
7
= -(420
Hz)
= 480 Hz.
26. B.
,
The beat frequency between the notes is f
f, -f,= (784.87 - 783.99) HZ= 0.88 HZ.
1P
= 367.5 Hz.
23. B.
The figure shows the fundamental and the third
harmonic for the C , string. The fundamental frequency
is given by f,= vl(2L). The frequency of the third
harmonic is given by f, = vl(2l3 L) = 3 f,.Thus to get
the frequency of the fundamental, we need to divide
784.87 Hz by 3.
27. D.
If the car were to hear the frequency, the frequency it would hear is given by
,,,
f ,= 350
(42 kHz)
24. D.
The emitter is moving, and we choose the minus
sign in the denominator because we want the answer to
be greater than 420 Hz.Thus we have
= 56 kHz.
This is the equivalent of sending a signal and having it
reflectfrom a moving target. There are two Doppler
shifts.
6. B.
1. B.
B.
Since the insect and the bat are moving in the
same direction at the same speed, there is no Doppler
shift, and the detected frequency is simply 30 kHz. If
you want to make sure, you can work this problem like
problem 27 above.
4.
D.
If the tree could detect a frequency, it would
detect
f=
,,,_,,
343
50 kHz.
D.
In the upper figure we see the beat, so T refers to
the beat period, which is l/j-,. The reason that the
power never reaches zero is that the outgoing wave has
a larger amplitude than the incoming wave.
C.
Higher harmonic content refers to higher frequencies being present in addition to the fundamental. These
higher harmonic frequencies thus have shorter wavelength. This would not really aid in distance measurement, so choice A is incorrect. Stunning the insect?
Choice B is also incorrect. There is a sentence in
paragraph 1 which indicates that C is plausible, since
shorter wavelengths are more likely to be reflected if
the wavelength of the fundamental is too large. If the
harmonics are Doppler shifted, then so is the fundamental, so choice D is incorrect.
5.
3.
4.
A.
D.
Because the detected frequency is lower, the
vehicle is receding, but there is no way to know from
the passage whether it is doing so directly away or at an
angle. The Doppler effect happens in three dimensions,
and we have discussed only the one dimension parallel
to the motion (which is the important one).
Passage 3
1. C.
$+
I
Passage 4
1.
B.
This is the definition of interference. Diffraction
is the spreading of waves. Beats is a particular phenomenon which occurs when waves of similar frequency
interfere. Difference tones have to do with the way the
ear processes sound.
2.
C.
The passage says that a note of average frequency
turns on and off. Thus the perceived frequency is (30.87
Hz + 32.70Hz)/2 = 3 1.79Hz.
node
7
antinode.
3. B.
This question asks for the beat frequency (times
per second), sof
, = 32.70Hz - 30.87Hz = 1.83 Hz.
4.
C.
Choices A, B, and C all share the property that
the difference is the desired frequency 110 Hz, but
choice D is excluded. Choices A and B include frequencies lower than 110 Hz, which cannot possibly be
harmonics, so A and B are incorrect.
A.
We usef = VIA.
6.
C.
According to paragraph 3, sound of two frequencies f, and f, enter the ear. Choice A is definitely wrong,
especially with a wavelength drawn onto the graph. For
that the horizontal axis must be a space coordinate like
x. Choice B might show a portion of the power spectrum after some processing, but the frequencyf, -f,
does not enter the ear. The ear constructs the difference
tone later. Choice C is correct. Choice D might have
been correct if time were the horizontal coordinate.
Since Alice is an equal distance from the speakers, and the speakers are emitting sound waves in phase,
wave crests arrive at her location in phase. Thus she is
at an antinode. Bob has moved to a position of relative
silence, which must be a node.
2.
C.
The waves arrive out of phase where Bob is
sitting, because the wave fiom the left speaker takes a
bit longer to arrive. When a crest from the right speaker
is arriving, the corresponding crest from the left is still
in transit. By the time it arrives, a trough is arriving
from the right speaker.
3. D.
The sum of the two distances is not significant
and cannot be derived from the information.
4.
D.
The key here is that, for Bob, the waves arrive out
of phase. When crest is coming from the left speaker,
trough comes from the right. The difference is half a
wavelength.
5.
C.
Alice is positioned equidistant from the speakers,
so for her the waves will still be in phase, so A and D
are incorrect. Bob's location at a node depends on the
wavelength of the sound, which we change when we
change the frequency.
6. C .
Experiment 2 is the prescription for creating
beats.
Solutions
Chdpter
13
Chapter 13 Solutions
--.
lass
glass
The
6. C .
11. D.
Oi = 60".
n, sin 8, = ni sin 8,,
2
8, = sin-' -.
3
1
sin 6 , = 2'
12. A.
For the critical angle, we must have the refracted
angle be 90":
ni sin 8,,, = n, sin 90,
3
sine,, = -,
4
3
0cnt. =sin-' -.
4
9.
D.
= n, sin 90".
14. B.
19. B.
20. C .
17. C.
(object
di
do
21. B.
The
22. D.
26. C.
If the object is an infinite distance away, then the
focus is on the focal plane, 12 m in front of the mirror.
To see this in the equation, we write
Ii
Z image
The figure shows the ray diagram for the candle
6 m from the mirror. From the diagram we can see that
the image is about 12 m behind the mirror. The equation becomes
30. B.
image
'
25. C.
13
32. B.
1.
B.
The light emerging from the first (left) polarizer
has half the intensity of the original source, since the
horizontal component has been taken out of it. All of
the resulting light passes through the second (right)
polarizer, since it is already vertically polarized. Thus
the final beam has half the intensity of the original.
1 - 1
---
f,,
2m
+-, 1
4m
D.
The passage states (paragraph 1) that radiation is
emitted perpendicular to the wire. For the antenna in
question 5, most of the radiation is emitted east, west,
up, down, and so on, but none is emitted north and
south.
37. D.
The power of the combination is the sum of the
powers.
The figure shows the ray diagram for this problem. The diagram is not too much help in this problem,
so we calculate
D = 10m.
You must stand 10 m away.
3. A.
The subtended angle can be calculated from
information in the third paragraph, that is, the ratio of
the spatial separation of the top and bottom of the moth
to the distance from the moth to the eye. Thus the angle
is 0.01 m10.25 m = 0.04 radians, which is about 2.3".
4.
D.
The desired power of the combination of corrective lens plus eye lens is 110.025 m = 40 D. Since PC,,,
= Peye+ Pmmt, we must have PC,,=
5 D.
14
Chapter 1 4 Solutions
Sections A-D
1. A.
At first it seems as if 111can be definitely concluded. Then we remember that a charged object can
attract a neutral object if it induces a charge, so D is the
correct answer.
D.
Both charges increase by a factor of 2. Both
contribute an increase of a factor of 2 to the force.
5.
B.
By Coulomb's law, the force exerted by one
charge on the other has the same magnitude as the force
the other exerts on the first. This is an example of
Newton's thud law.
Q*
7. B.
The positive charges will repel each other and,
being free to move, will move as far apart as possible.
Thus they will end up distributed as in choice B.
The
13. C .
16. A.
First we draw a diagram, showing the electric
field from the various charges.
Note that because point P is closer to the negative
charge, the vector to the left is longer. The sum will be
to the left, so the answer is A.
If we want to calculate the numbers, we calculate
the magnitude
17. A.
First we draw electric field vectors into the
diagram.
14
20. B.
First we draw a diagram, showing the electric
field from the various charges.
F
m'
a=-
m
2 2 A.
The electron has a charge of opposite sign, E is constant, and m is 2000 times smaller. Therefore, the
acceleration of the electron is of opposite direction and
2000 times greater.
28. A.
Then, we have
w =%(v, - Y )
29. C.
30. B
The clue here is the combination of energy and
charge, so the equation to use is W = qAV. Although the
problem specifies that the path is straight across, the
energy is path independent, so we do not need that
information. Thus
w = q(V, - VA)
33. A.
31. D
Again we use
W = qAV = q(v, - v,)
34. A.
Whenever we see a problem about work or energy
with charges, we know to write W = QAV. If we try to
apply W = FAxcos I$,we are dead in the water, because,
as the charge moves, F changes. In this case the charge
q starts at point A, where the potential due to Q is VA=
kQ /rl = 2 x lo4V, where we use r, = 5 m. It moves to
point 8, which is r, = 1 m away from Q, so V, =
kQlr2= 10' V. The work required is
W=qAV
32. D.
Whenever we see a problem about work or energy
with charges, we know to write W =Q AV. Let's
consider the charge q, to be fixed and move charge q,
from initial distance rito final distance r, (see figure).
36. A.
37. A.
If we work out the electric potential at points A
and B, we obtain
41. B.
2
energy. Thus if v is increased by a factor of 4, then the
initial kinetic energy is increased by a factor of 16, as is
the final electrostatic energy.
43. A.
= 4 x 105 volts,
since r, = r4= 5 m. (Use the Pythagorean theorem.)
39. B.
The
44. D.
B.
If the current is up, then the electrons (being
negative) must be flowing down.
46. D.
53. A.
47. C.
54. B.
Passage 1
1. A.
49. C .
3. B.
The magnetic field is perpendicular to the
direction of propagation (northlsouth) and to the
electric field (upldown). So the magnetic field must
point eastlwest.
51. A.
D.
4.
B.
6. A.
Passage
14
1. A.
5.
Passage
C.
409
The
MCAT Physics
Booic
Chapter 15 Solutions
1. C.
D.
Height gives a measure of the energy per mass
required to place water at that point, just as electric
potential is a measure of the energy per charge required
to place a charge at that point
6.
B.
See the above explanation.
3. C.
C.
8.
B.
See above explanation.
Solutions .............................
Chapter 1 5
14. C.
11. C.
D.
In this case, Ohm's law is A y = IR, since I is
constant. The light bulb with the higher resistance has a
proportionally higher potential drop.
The
18. B.
Solutions
............................. Chapter
15
28. C.
The power dissipated (that is,-the energy per unit
time) is constant. If the question had asked, "What is
the energy dissipated look like as a function of time?',
then D would have been the answer. B might have been
the answer if the source were alternating current.
29. B.
34.
D.
See the diagram above.
36. A.
We apply the equation for resistors in parallel:
1
RT
Rl
R2
--r=-+-
37. D.
Voltage does not flow. The question makes as
much sense as asking what height is flowing in a river.
Solutions
I5
38. A.
44. C.
The circuit diagram is simply that shown above.
Since the power is given by P = IAV, we obtain the
current from I = 300 Wl120 V = 2.5 A. The resistance is
120 V12.5 A = 48 a.
1. C.
There is an electric field E, between the plates,
and the force on a charged particle is given by F,, =
2.
4.
A.
In Experiment 2 the increase in distance results in
a decrease in capacitance by a factor of 3 (see equation). Since AV is the same in Experiment 2, the charge
(Q = CAV) decreases by a factor of 3.
3.
B.
A.
1. A.
Once the wires are removed from the plates, no
more charge may be transferred, so the charge must
stay the same. This is a situation in which the question
becomes easy if we visualize the charges and think
about the experiment. The question is difficult or
intractable if we rely on rote memorization of equations.
5.
Passage 2
1. A.
D.
The capacitance does not change, so the charge
must increase by a factor of 4, since we have Q = CAV.
3.
2.
D.
The electric field is E = AVld, so the electric field
increases by a factor of 4.
B.
The formula AV = EAx works here, where Ax is
the separation between the Earth's surface and the
ionosphere. The Earth and the ionosphere are like a
parallel-plate capacitor which has been bent intoa
sphere.
C.
The capacitance is decreased by a factor of 2
since the distance is increased by a factor of 2. The
charge is still Q,since there is no way for it to.change.
Thus the potential between the plates must change. The
only equation we have relating voltage and charge is
AV= QIC, so AV increases by a factor of 2.
2.
3.
A.
The work required is given by W = qAV =
(-1.6 x 10-I9C) (9 x 105J/C) =-1.4 x 10-l3J.But we
are unsure about the sign. The Earth has a negative
charge, so taking an electron away from it is easy: a
downhill ride, so to speak. Thus the change in potential
energy is negative.
4.
D.
The electric field points away from a positive charge
and toward a negative one. Thus it points toward the
Earth and away from the ionosphere. The other two
pictures are reminiscent of the Earth's magnetic field.
5. A.
Since the passage said that the potential difference between the ionosphere and the Earth's surface is
9 x lo5volts, we can exclude C and D. For a proton,
going from the surface to the ionosphere is uphill, that
is, going to greater potential energy away from the
Earth's negative charge. Therefore A is the correct
answer.
Passage 4
1. A.
A sodium ion (Na') is positively charged, and the
negatively charged central wire will attract it.
2.
15
A.
It does not matter what kind of molecule this is.
The only thing that matters is its neutral charge. The
passage states that it will be attracted to the wire.
Passage 5
1. B.
We have several formulas involving electric field:
E = kQlr for a point charge, E = AVIAx, and F = qE.
We are not given the charge of the cloud in the passage
(and even if we were, it would be difficult to get E from
it), nor are we given a force on a charge. But we are
told the potential AV between the cloud and the Earth
and the distance Ax between them. Thus E =
(10' J/C)/(2 x 10' m) = 5 x 10' NIC. (To get the units,
recall J = Nm.)
3. D.
Since this question is about energy and charges,
our guess is that we will use W = qAV somewhere. A
fluoride ion near the negative wire has high potential
energy, so it gains kinetic energy as it moves away from
the wire. The change in potential energy is qAV =
(-1.6 x 10-l9C)(5 x 10' JIC) = -8 x lo-" J. (Recall: a
fluoride ion has one extra electron, so its charge is the
same as an electron.) The increase in kinetic energy is
thus 8 x 10-ISJ, since energy is being conserved. Now
some energy may be lost to heat, and so on, so this is
actually the maximum energy available.
2.
6. D.
B.
5. C.
The only thing we can do with the new piece of
information is to combine it with the information in the
third paragraph, to obtain the power usage of P =
(300 Ym3) (100 m31s) = 3 x lo4Watts. The question
asks for current, and the only connection we know of
between power and current is potential difference. Aha!
I = PIAV = (3 x lo4Jls)I(5 x lo4JIC) = 0.8 Us.
A.
4.
A.
This is easy if we remember that current is the
rate at which charge moves, I = Qldt. Thus dt = QII =
(4 c)1(2 x loqUS) = 2 x lo4 S.
D.
The attraction of neutral particles to a charged
wire is like a charged comb attracting neutral pieces of
The
6. B.
Visible light results from the electrical transition of electrons in atoms and molecules. Dissociation of molecules may produce some light, but
excitation of molecules is a better answer. (Ionization
andrecombination would have been another good
answer, if it had been a choice.) Heating and expansion of air is lower on the energy scale, producing
merely thunder.
Passage 6
1. C.
2.
A.
The cells are stacked in series. Thus the current
through all the cells is the same, and the total current
is simply 30 rnilliarnps. Remember, the currents add
up if the components of the current are in parallel.
The voltages add up in series.
D.
To obtain the magnetic field, we need to
consider the current. The main current during the
activated state is along the fish from tail to head. The
right-hand rule applied to this implies a magnetic
field circling the body in the direction shown in
choice D. The secondary currents outside the fish in
the fourth figure reinforce this magnetic field. On
Solutions
Chapter 16 Solutions
16
EK~+?
1. D.
2.
C.
The force on a particle is simply F = qE,. The
charge on the deuteron (orie proton and one neutron)
is the opposite of the charge on the electron. Thus the
forces on the two are the same in magnitude.
3. A.
The symbol 'OF- represents an ion with a
nucleus of 9 protons (hence F) and 11 neutrons
(hence mass number 20) and 10 electrons outside the
nucleus (hence the overall charge). For this problem,
the overall charge is all that is needed. Since F = qEo,
the magnitudes of the forces are the same.
C.
We have a = qE,jm from Problem 4. Now 'H
has one half the charge of He (decreasing the
acceleration by a factor of 2) and one fourth the mass
(increasing the acceleration by a factor of 4). The
result is an increase by a factor of 2, as expressed in
answer C.
6. B.
The nuclei have the same charge (both have
one proton). The forces on both are the same.
8.
B.
The reactants are i ~ + e He. Thus the superscripts must add to 6, so C is not the answer. The
subscripts must add to 4, so A and D are eliminated.
9.
C.
The left side of the reaction is represented by
19. D.
+ :He.
20. C.
We write the nuclear reaction
238
,,U+
13.
i ~ e -ye+
+
-ye+ ; H e + ? .
B.
The lower limit for visible light is 4.0 x 1014Hz,
which corresponds to E,,,,, = hf = (4.14 x
ev s)
(4.0 x 1014Hz) = 1.7 ev, and the upper limit corresponds to E,,,,, = (4.14 x lo-'' ev s) (7.5 x 1014Hz) =
3.1 ev. ?he only transition to fall in these limits is from
-2.5 ev to 0 ev.
21. C.
A.
:H+ :H-+ ? ~ + y
B.
C.
D.
+v
-ye- + V
17. C.
18. B.
We write the nuclear reaction
200(+]
= 30,
Solutions
25. C.
.. . . ..... .......... . . . . . . . . . .
Chdpter 16
Passage 1
1. B.
3.
26. A.
We calculate m,=
(7.01601 + 1.00783 2 (4.00260)) amu.
28. C.
The original ' ~ must
e have more mass than the
final products in order for the reaction to occur. The
mass deficit is converted into the kinetic energy of the
resulting products.
29. C.
30. B.
Choices C and D are nonsensical. Choice A is
possible, but in fact the principle of massenergy
conservation is completely general. The energy of a
reaction must be in the form of mass before the reaction
occurs. It is, however, quite small for chemical reactions, about a million (lo6) times smaller than for
nuclear reactions.
A.
4.
C.
Both momentum and energy are conserved
because there are no external forces in this scenario.
Energy is converted from one form to another: from the
energy of an excited state to photon energy and (much
less) kinetic energy of the recoiling atom.
The
Passage
3.
1. C.
Since the radiation is blocked by several centimeters of aluminum, it is not gamma radiation. But it is
not alpha radiation, since a sheet of metal foil fails to
block it. Is it normal beta radiation or positron radiation? The information from the magnetic field indicates
the particles are positive. n u s the decay is positron
decay.
A.
5.
6. D.
Each 9.5 hours sees a decrease by a factor of 2 in
radioactivity. Therefore, the sample will never have zero
radioactivity. (However, there must be a last atom todecay, so the answer "indefinitely" is better than
"forever".)
5. A.
The Sun emits photons of all sorts of frequencies;
that is what makes the colors of the rainbow. And many
photons make it through the atmosphere.
A.
After 19 hours, the radioactivity of the sample
decreases by a factor of 4, that is, two halflives. Its
halflife is 9.5 hours, which is what the question asks
for.
C.
Passage 4
1. D.
2. C .
The only information we need here is that the
alpha particles are positive. In an electric field which
points up, they will experience a force up.
2.
B.
According to the passage, K-capture happens
under the same conditions which promote positron
decay, that is, proton-rich nuclei. That is to say, nuclei
with more protons than neutrons have a negative N - Z.
3. B.
According to the passage, the possibility of
electron capture is nonzero only if there is some overlap
of the electron wavefunction and the nucleus, so if the
orbital has vanishing amplitude near the nucleus, the
probability of capture is small.
4.
C.
4.
16
D.
The reaction is
z
~ +fi ~ + e';:~m.
5. D.
The 7.8 years represents 3 halflives, so the.
original sample decreased by a factor of 2 three times.
The original sample must have been 0.08 moles.
6. B.
5.
A.
Passage 5
Passage 6
1. D.
1. B.
Most of the energy goes into the ionization of
molecules. The energy comes from nuclear energy.
2.
D.
We write the reaction
:H+ : : ~ e - + ':F+?.
Choice A is nonsensical. Choice B misses the
point, since the question asks why these neutrons do
not tend to ionize the tissue. The passage indicates
(paragraph 2) that a charged particle with large
energy is ideal for ionizing tissue, so a neutron with
no charge and little energy will do little harm. Choice
C is a likely possibility. Choice D is excluded since
neutrons are an elementary particle.
3. B.
Choices C and D do not have the superscripts
and subscripts balanced. Choice A does not satisfy
the description in the passage, in that there is no
gamma ray in the products.
':
4.
C.
The
5.
D.
The 8 minutes is about 4 halflives, so the activity
must have decreased by a factor of 2 four times. Its
initial activity was 160 mCi.
6.
A.
Index
Index
Acceleration 1 7-1 8
centripetal 6 5
uniform 21, 22-23
conservation of
Continuity 1 6 6
Airbag 181-182
Alpha decay 3 0 9
,4mplitude 1 9 0
Antinode 1 9 2
Archimedes' priniple 160
11 3, 1 5 9
alternating 290-991
electric 263, 279, 290
Atom 3 0 5
Atomic mass 3 0 6
Atomic number 3 0 6
Atomic weight 3 0 6
D
DC cell 9 7 9
Density 1 59-1
Attitude 3-4
60
Dielectric 250, 2 9 0
69
Differencetone 2 2 1
Diffraction 2 4 6
Dimensions 2, 5
Diopters 2 3 8
Dispersion 2 3 8
Doppler shift 2 1 3-2 1 5
Drag 83-85
Dust mite 9 8 9
E
Eficiency 1 29-1 3 0
Charge 249-950
conservation 01 2 5 0
electric
949
induced 9 5 0
Circular motion. See Motion: circular
Closet 3 1 7
Collision
elastic 1 2 9
inelastic 1 2 9
color 225, 9 3 8
Components 34
Conductor 2 5 0
Elastic limit 1 0 2
Elasticity 1 0 2
Uectric charge. See Charge: electric
Electric circuits 279-991
Electric dipole P 57, 2 5 8
Electric eel 3 0 2
El-c
fidd
953-958,
964-965,
989
168, 1 8 6
K-capture 3 1 9
OW
rate 1 6 6
Lens 230-234
69
aberations 2 3 9
uorescence 308
combination 2 3 8-239
,cal length 2 3 0
convergng
xce 13
diverging 2 3 0
160-1 6 1
65
bouyant
power 2 3 8
centripetal
thin 2 3 9
conservative 126
36-3
diagrams
meter
Lever arm 9 6
electric 252-253,
230
Light 225-239,
253-258
161
ultraviolet
reefall 47-48
254, 264-265
speed of 2 2 5
226
visible 2 2 6
Lightning rod 3 0 1
riction 77-85
kinetic 80-89
static 77-80
264-265
3arnma decay. 3 1 0
Jamma rays 2 2 6
Mass number 3 0 6
Mass-energy conversion ' 3 1 1
Jauge pressure 1 6 4
Microwaves 925-226
Jraphs 18-90
Mirrors 2 3 5-23 7
zravitation, Law of 5, 4 5
Molecule 2 4 9
Sravity 45-5 3
Moment of inertia 9 3
Sround 2 5 1, 9 9 0
Momentum 1 1 1-1 1 5
conservation of 1 1 2-1 1 4
-.-
impulse 1 1 4
Motion
Harmonic 195,
2 10-9 1 2
66-69
circular 64-66,
Halflife 3 1 0
1 85
first law
3 1-3
notmal mode 1 9 4
second law
3 3-3 5
6
equilibrium
Neutron 249,
Impulse 1 1 4
305
Newton's
Inclined planes 6 3 - 6 4
cradle 1 1 1
Index of refraction 2 2 6 1 3 0 , 23 8
Inertia 3 1
insulator
250, 990
199
Intensity 208-2 1 0
Node
Interference 1 9 1
Nonconductor
constructive
destructive
192
250,
951,
990
305
Index
1 87-1 88
226-230
Sound 207-21 5
beats 21 2, 22 1
intensity 208-2 10
pipes 2 1 0-2 12
pitch 21 0
production 207
speed 208
Specific gravity 1 59
Speed 16-17
Sphygmomanometer 1 64
Springs 1 85-1 96, 1 86
28 1-284
305
Ohm's Law
Orbital
P
1 88
93, 1 88
Periodic motion 1 87-1 88
Phase 192
Phosphorescence 3 17
Photon 264, 306
Pendulum
Period
Snell's law
Polarization
Positron decay
Strain
Potential
1 01
101
Stress
259
electric 2 58-263
terminal 285
absolute electric
Potential difference
260
Power 1 30-1 31
electric 286-2 87
Pressure 1 59, 1 59-1 60, 162-1 64
Projectile 47-48, 52
Proton 249, 305
Pulleys 131-133
electrical
Superposition
Surface tension
T
301
291, 297
93-102
Thunderclouds
Toaster
Torque
electromagnetic
264-96 5, 974
929
307-309
1 66-1 67
Ultrasound
Radiation
192
1 64-1 65
205
2-3
Units
309
309-3 1 2
Ray-tracing diagram 231, 23 5
Real image 232
Reflection 226-230
Refraction 226-9 30
Radioactive decay
Radioactivity
981
internal 285
Resistivity 286
Resistor 981-984
ddinition 979
in parallel 989, 983
in series 981, 989
Resolution 946
Resonance 189
Reynolds number 89
Rocket engine 1 1 9
Root mean square 291
Resistance
Scattering
308
Shear modulus
101
v
253
1 4-1 5
Velocity 1 6-1 7
Virtual image 232
Viscosity 1 66-1 67
Vector field
Vectors
Visible
spectrum
926
Voltage
Wave
199
1 88
frequency 1 88
fundamental 195
infrared 225-926
intensity 208-9 10
longitudinal 190
radio 995-996
standing 194-1 96
addition
amplitude
'ouns's modulus
I OI